CAF7-Financial Accounting and Reporting II_Questionbank

September 28, 2017 | Author: Evan Jones | Category: Depreciation, Expense, Balance Sheet, Fixed Asset, Dividend
Share Embed Donate


Short Description

financial reporting question book for ca students...

Description

2015

FINANCIAL ACCOUNTING AND REPORTING II QUESTION BANK

CAF-07

ICAP

Question Bank

P

Financial accounting and reporting II

Second edition published by Emile Woolf Limited Bracknell Enterprise & Innovation Hub Ocean House, 12th Floor, The Ring Bracknell, Berkshire, RG12 1AX United Kingdom Email: [email protected] www.emilewoolf.com

© Emile Woolf International, February 2015 All rights reserved. No part of this publication may be reproduced, stored in a retrieval system, or transmitted, in any form or by any means, electronic, mechanical, photocopying, recording, scanning or otherwise, without the prior permission in writing of Emile Woolf Publishing Limited, or as expressly permitted by law, or under the terms agreed with the appropriate reprographics rights organisation. You must not circulate this book in any other binding or cover and you must impose the same condition on any acquirer.

Notice Emile Woolf International has made every effort to ensure that at the time of writing the contents of this study text are accurate, but neither Emile Woolf International nor its directors or employees shall be under any liability whatsoever for any inaccurate or misleading information this work could contain.

© Emile Woolf International

ii

The Institute of Chartered Accountants of Pakistan

Certificate in Accounting and Finance Financial accounting and reporting II

C Contents Page

Question and Answers Index

v

Questions Section A

Questions

1

Answers

93

Answers Section B

© Emile Woolf International

iii

The Institute of Chartered Accountants of Pakistan

Financial accounting and reporting II

© Emile Woolf International

iv

The Institute of Chartered Accountants of Pakistan

Certificate in Accounting and Finance Financial accounting and reporting II

I

Index to questions and answers Question page

Answer page

CHAPTER 2 – IAS 1: PRESENTATION OF FINANCIAL STATEMENTS 2.1

LARRY

2

94

2.2

MINGORA IMPORTS LIMITED

3

95

2.3

BARRY

4

97

2.4

OSCAR INC

6

99

2.5

CLIFTON PHARMA LIMITED

7

101

2.6

SARHAD SUGAR LIMITED

8

103

2.7

BSZ LIMITED

10

105

2.8

YASIR INDUSTRIES LIMITED

12

108

2.9

SHAHEEN LIMITED

14

112

2.10

MOONLIGHT PAKISTAN LIMITED

15

114

2.11

FIGS PAKISTAN LIMITED

17

115

CHAPTER 3 – IAS 7: STATEMENTS OF CASH FLOWS 3.1

KLEA

19

119

3.2

STANDARD INC

21

121

3.3

FALLEN

22

124

3.4

BIN QASIM MOTORS LIMITED

24

126

3.5

ITTEHAD MANUFACTURING LTD

27

129

3.6

WASEEM INDUSTRIES LIMITED

29

131

3.7

JALIB INDUSTRIES LIMITED

31

134

© Emile Woolf International

v

The Institute of Chartered Accountants of Pakistan

Financial accounting and reporting II

Question page

Answer page

3.8

APOLLO INDUSTRY LIMITED

32

136

3.9

MARVEL ENGINEERING LIMITED

34

137

CHAPTER 4 – CONSOLIDATED ACCOUNTS: STATEMENTS OF FINANCIAL POSITION – BASIC APPROACH 4.1

HALL

36

139

4.2

HASSLE

37

140

4.3

HYMN

37

141

4.4

HANG

38

143

4.5

HASH

38

144

CHAPTER 5 – CONSOLIDATED ACCOUNTS: STATEMENTS OF FINANCIAL POSITION - COMPLICATIONS 5.1

HAIL

39

146

5.2

HAIRY

40

148

5.3

HARD

41

150

5.4

HALE

42

152

5.5

HELLO

42

153

5.6

HASAN LIMITED

43

155

CHAPTER 6 – CONSOLIDATED ACCOUNTS: STATEMENTS OF COMPREHENSIVE INCOME 6.1

HARRY

45

159

6.2

HORNY

46

161

6.3

HERON

47

163

6.4

HANKS

48

164

CHAPTER 7 – PROPERTY, PLANT AND EQUIPMENT 7.1

ROONEY

50

168

7.2

EHTISHAM

51

170

7.3

CARLY

51

172

7.4

ADJUSTMENTS LIMITED

52

173

7.5

FAM

53

175

7.6

IMRAN LIMITED

54

177

7.7

HUMAYUN CHEMICALS LIMITED

55

179

7.8

FARADAY PHARMACEUTICAL LIMITED

55

180

© Emile Woolf International

vi

The Institute of Chartered Accountants of Pakistan

Index to questions and answers

Question page

Answer page

7.9

SPIN INDUSTRIES LIMITED

56

182

7.10

SCIENTIFIC PHARMA LIMITED

56

183

7.11

QURESHI STEEL LIMITED

57

184

7.12

GRANITE CORPORATION

58

185

CHAPTER 8 – IAS 38: INTANGIBLE ASSETS 8.1

FAZAL

59

186

8.2

HENRY

59

186

8.3

TOBY

60

187

8.4

BROOKLYN

60

188

8.5

ZOUQ INC

61

189

8.6

STAR-BRIGHT PHARMACEUTICAL LIMITED

62

190

8.7

RAISIN INTERNATIONAL

62

191

CHAPTER 9 – IAS 17: LEASES 9.1

DAWOOD

64

192

9.2

FINLEY

64

192

9.3

FABIAN

64

193

9.4

XYZ INC

65

194

9.5

SNOW INC

65

197

9.6

MIRACLE TEXTILE LIMITED

66

199

9.7

SHOAIB LEASING LIMITED

66

200

9.8

NEPTUNE LIMITED

67

202

9.9

QUARTZ AUTO LIMITED

67

204

9.10

LODHI TEXTILE MILLS LIMITED

68

205

9.11

NOMAN ENGINEERING LIMITED

68

206

CHAPTER 10 – IAS 37: PROVISIONS CONTINGENT LIABILITIES AND CONTINGENT ASSETS AND IAS 10: EVENTS OCCURRING AFTER THE REPORTING DATE 10.1

BADAR

69

207

10.2

GEORGINA

69

207

10.3

EARLEY INC

70

209

10.4

ACCOUNTING TREATMENT

70

210

© Emile Woolf International

vii

The Institute of Chartered Accountants of Pakistan

Financial accounting and reporting II

Question page

Answer page

10.5

J-MART LIMITED

71

211

10.6

AKBER CHEMICALS LIMITED

72

212

10.7

QALLAT INDUSTRIES LIMITED

72

213

10.8

SKYLINE LIMITED

73

213

10.9

WALNUT LIMITED

74

214

10.10

ATTOCK TECHNOLOGIES LIMITED

75

215

CHAPTER 11 – IAS 8: ACCOUNTING POLICIES, CHANGES IN ACCOUNTING ESTIMATES AND ERRORS 11.1

WONDER LIMITED

76

216

11.2

DUNCAN

77

217

11.3

MOHANI MANUFACTURING LIMITED

78

218

CHAPTER 12 – IAS 12: INCOME TAXES 12.1

FRANCESCA

79

219

12.2

SHEP (I)

79

220

12.3

SHEP (II)

80

221

12.4

SHEP (III)

81

222

12.5

SHEP (IV)

82

224

12.6

WAQAR LIMITED

82

225

12.7

SHAKIR INDUSTRIES

83

227

12.8

MARS LIMITED

84

228

12.9

BILAL ENGINEERING LIMITED

85

230

12.10

GALAXY INTERNATIONAL

85

231

12.11

APRICOT LIMITED

86

232

CHAPTER 13 – RATIO ANALYSIS 13.1

WASIM

87

233

13.2

AMIR AND MO

88

233

CHAPTER 14 – ETHICAL ISSUES IN FINANCIAL REPORTING 14.1

ETHICAL ISSUES

90

235

14.2

SINDH INDUSTRIES LTD

90

236

© Emile Woolf International

viii

The Institute of Chartered Accountants of Pakistan

SECTION

Certificate in Accounting and Finance Financial accounting and reporting II

A Questions

© Emile Woolf International

1

The Institute of Chartered Accountants of Pakistan

Financial accounting and reporting II

CHAPTER 1 – LEGAL BACKGROUND TO THE PREPARATION OF FINANCIAL STATEMENTS There are no questions specific to chapter one. This is because the learning outcomes in this area concern the preparation of financial statements and these questions have given in relation to chapter 2 in this question bank.

CHAPTER 2 – IAS 1: PRESENTATION OF FINANCIAL STATEMENTS 2.1

LARRY The trial balance of Larry at 31 December 2015 is as follows.

Administration charges Bank account Cash Payables’ ledger Accumulated amortisation on patents at 31 December 2015 Accumulated depreciation at 31 December 2015 Receivables’ ledger Distribution expenses Property, plant and equipment at cost Interest received Issued share capital Loan Patents at cost Accumulated profits Purchases Sales Inventories at 31 December 2014

Rupees in million Dr Cr 342 89 2 86 5 918 189 175 2,830 20 400 18 26 1,562 2,542 3,304 118 ––––– ––––– 6,313 6,313 ════ ════

The following information is also relevant. (1)

Inventories on 31 December 2015 amounted to Rs. 127 million.

(2)

Current tax of Rs. 75 million is to be provided.

(3)

The loan is repayable by equal annual instalments over three years.

Required Prepare an statement of profit or loss (analysing expenses by function) for the year ended 31 December 2015 and a statement of financial position as at that date.

© Emile Woolf International

2

The Institute of Chartered Accountants of Pakistan

Questions

2.2

MINGORA IMPORTS LIMITED The trial balance of Mingora Imports Limited at 31 December 2015 is as follows.

Patent rights Work-in-progress, 1 January `2015 Leasehold buildings at cost Ordinary share capital Sales Staff costs Accumulated depreciation on buildings, 1 January 2015 Inventories of finished games, 1 January 2015 Consultancy fees Directors’ salaries Computers at cost Accumulated depreciation on computers, 1 January 2015 Dividends paid Cash Receivables Trade payables Sundry expenses Accumulated profits, 1 January 2015

Rupees in million Dr Cr 60 125 300 600 1,740 260 60 155 44 360 50 20 125 440 420 92 294 121

––––––

––––––

2,633 ════

2,633 ════

The following information is also relevant. (1)

Closing inventories of finished games are valued at Rs. 180 million. Work in progress has increased to Rs. 140 million.

(2)

The patent rights relate to a computer program with a three year lifespan.

(3)

On 1 January 2015 buildings were revalued to Rs. 360 million. This has not yet been reflected in the accounts. Computers are depreciated over five years. Buildings are now to be depreciated over 30 years. An allowance for bad debts (irrecoverable debts) of 5% is to be created.

(4) (5)

There is an estimated bill for current tax of Rs. 120 million which has not yet been recognised.

Required Prepare an statement of profit or loss (analysing expenses by nature for the year ended 31 December 2015 and a statement of financial position as at that date.

© Emile Woolf International

3

The Institute of Chartered Accountants of Pakistan

Financial accounting and reporting II

2.3

BARRY Barry has prepared the following draft financial statements for your review Barry: Statement of profit or loss for year to 31st August 2015 Rs. in ‘000 Sales revenue Raw materials consumed Manufacturing overheads Increase in inventories of work in progress and finished goods Staff costs Distribution costs Depreciation Interest payable

30,000 (9,500) (5,000) 1,400 (4,700) (900) (4,250) (350) ––––––

6,700 ════ Statement of financial position as at 31st August 2015 Rs. in ‘000 Assets Non-current Freehold land and buildings Plant and machinery Fixtures and fittings

Rs. in ‘000

20,000 14,000 5,600

–––––––

39,600 Current assets Prepayments Trade receivables Cash at bank Inventories

200 7,400 700 4,600

–––––––

12,900

–––––––

Total assets

52,500

–––––––

Equity and liabilities Equity shares of Rs. 1 each Accumulated profit Share premium

21,000 14,000 2,000

–––––––

Total equity Revaluation surplus Current liabilities

37,000 5,000 5,300

Non-current liabilities 8% Debentures 2019

5,200

–––––––

Total equity and liabilities

© Emile Woolf International

52,500 ═════

4

The Institute of Chartered Accountants of Pakistan

Questions

Additional information 1

Income tax of Rs. 2.1 million has yet to be provided for on profits for the current year. An unpaid under-provision for the previous year’s liability of Rs. 400,000 has been identified on 5th September 2015 and has not been reflected in the draft accounts.

2

There have been no additions to, or disposals of, non-current assets in the year but the assets under construction have been completed in the year at an additional cost of Rs. 50,000. These related to plant and machinery. The cost and accumulated depreciation of non-current assets as at 1st September 2014 were as follows:

Freehold land and buildings (land element Rs. 10 million) Plant and machinery Fixtures and fittings Assets under construction

Cost

Depreciation

Rs. in ‘000 19,000

Rs. in ‘000 3,000

20,100 10,000 400

4,000 3,700 -

3

There was a revaluation of land and buildings during the year, creating the revaluation surplus of Rs. 5 million (land element Rs. 1 million). The effect on depreciation has been to increase the buildings charge by Rs. 300,000. Barry adopts a policy of transferring the revaluation surplus included in equity to retained earnings as it is realised.

4

Staff costs comprise 70% factory staff, 20% general office staff and 10% goods delivery staff

5

An analysis of depreciation charge shows the following: Rs. in ‘000 Buildings (50% production, 50% administration) Plant and machinery Fixtures and fittings (30% production, 70% administration)

1,000 2,550 700

Required Prepare the following information in a form suitable for publication for Barry’s financial statements for the year ended 31st August 2015. Statement of profit or loss Statement of financial position Reconciliation of opening and closing property, plant and equipment

© Emile Woolf International

5

(25)

The Institute of Chartered Accountants of Pakistan

Financial accounting and reporting II

2.4

OSCAR INC The following trial balance has been extracted from the books of accounts of Oscar Inc as at 31 March 2015. Rs. in ‘000 Dr Cr Administrative expenses Share capital Receivables Bank overdraft Income tax (overprovision in 2014) Provision Distribution costs Non-current investments Investment income Plant and machinery At cost Accumulated depreciation (at 31 March 2015) Retained earnings (at 1 April 2014) Purchases Inventory (at 1 April 2014) Trade payables Sales revenue Interim dividend paid

210 600 470 80 25 180 420 560 75 750 220 180 960 140 260 2,010 120 3,630

3,630

Additional information (1)

Inventory at 31 March 2015 was valued at Rs. 150,000.

(2)

The income tax charge based on the profits on ordinary activities is estimated to be Rs. 74,000.

(3)

The provision is to be increased by Rs. 16,000.

(4)

There were no purchases or disposals of fixed assets during the year.

Required Prepare the company’s statement of profit or loss for the year to 31 March 2015 and a statement of financial position as at that date in accordance with IAS 1. (18)

© Emile Woolf International

6

The Institute of Chartered Accountants of Pakistan

Questions

2.5

CLIFTON PHARMA LIMITED The following trial balance relates to Clifton Pharma Limited, a public listed company, at 30 September 2015. Rs. in ‘000 Dr Cr Cost of sales Operating expenses Loan interest paid (see note (1)) Rental of vehicles (see note (2)) Revenue Investment income Leasehold property at cost (see note (4)) Plant and equipment at cost Accumulated depreciation at 1 October 2014: - leasehold property - plant and equipment Investments Share capital Share premium Retained earnings at 1 October 2014 Loan notes (see note (1)) Deferred tax balance at 1 October 2014 (see note (5)) Inventory at 30 September 2015 Trade receivables Trade payables Bank

134,000 35,000 1,500 8,600 338,300 2,000 250,000 197,000 40,000 47,000 92,400 280,000 20,000 19,300 50,000 20,000 23,700 76,400 14,100 12,100 830,700

830,700

The following notes are relevant (1)

The effective interest rate on the loan notes is 6% per year.

(2)

There are two separate contracts for rental of vehicles. A recent review by the finance department of these contracts has reached the conclusion that Rs. 7 million of the total rental cost of vehicles relates to a finance lease rather than an operating lease or rental arrangement. The finance lease was entered into on 1 October 2014 which was when the Rs. 7 million was paid: the lease agreement is for a four-year period in total, and there will be three more annual payments in advance of Rs. 7 million, payable on 1 October in each year. The vehicles in the finance lease agreement had a fair value of Rs. 24 million at 1 October 2014 and they should be depreciated using the straight line method to a nil residual value. The interest rate implicit in the lease is 10% per year. The other contract for vehicle rental is an operating lease and the rental payment should be charged to operating expenses. (Note: You are not required to calculate the present value of the minimum lease payments for the finance lease.)

(3)

Other plant and equipment is depreciated at 20% per year by the reducing balance method. All depreciation of property, plant and equipment should be charged to cost of sales.

© Emile Woolf International

7

The Institute of Chartered Accountants of Pakistan

Financial accounting and reporting II

(4)

The leasehold property has a 25-year life and is amortised at a straight-line rate. On 30 September 2015 the leasehold property was re-valued to Rs. 220 million and the directors wish to incorporate this re-valuation in the financial statements.

(5)

The provision for income tax for the year ended 30 September 2015 has been estimated at Rs. 18 million. At 30 September 2015 there are taxable temporary differences of Rs. 92 million. The rate of income tax on profits is 25%.

Required (a)

Prepare an statement of profit or loss for Clifton Pharma Limited for the year to 30 September 2015 (8)

(b)

Prepare a statement of financial position (balance sheet) for Clifton Pharma Limited as at 30 September 2015 (17) (25)

2.6

SARHAD SUGAR LIMITED The following trial balance relates to Sarhad Sugar Limited at 30 September 2015:

Leasehold property – at valuation 1 October 2014 (note (i)) Plant and equipment – at cost (note (i)) Plant and equipment – accumulated depreciation at 1 October 2014 Capitalised development expenditure – at 1 October 2014 (note (ii)) Development expenditure – accumulated amortisation at 1 October 2014 Closing inventory at 30 September 2015 Trade receivables Bank Trade payables and provisions (note (iii)) Revenue (note (i)) Cost of sales Distribution costs Administrative expenses (note (iii)) Interest on bank borrowings Equity dividend paid Research and development costs (note (ii)) Share capital Retained earnings at 1 October 2014 Deferred tax (note (v)) Revaluation surplus (Leasehold property)

Rs. in ‘000 Dr Cr 50,000 76,600 24,600 20,000 6,000 20,000 43,100 1,300 23,800 300,000 204,000 14,500 22,200 1,000 6,000 8,600 70,000 24,500 5,800 10,000 466,000

466,000

The following notes are relevant: (i)

Non-current assets – tangible: The leasehold property had a remaining life of 20 years at 1 October 2014. The company’s policy is to revalue its property at each year end and at 30 September 2015 it was valued at Rs. 43 million.

© Emile Woolf International

8

The Institute of Chartered Accountants of Pakistan

Questions

On 1 October 2014 an item of plant was disposed of for Rs. 2·5 million cash. The proceeds have been treated as sales revenue by Sarhad Sugar Limited. The plant is still included in the above trial balance figures at its cost of Rs. 8 million and accumulated depreciation of Rs. 4 million (to the date of disposal). All plant is depreciated at 20% per annum using the reducing balance method. Depreciation and amortisation of all non-current assets is charged to cost of sales. (ii)

Non-current assets – intangible: In addition to the capitalised development expenditure (of Rs. 20 million), further research and development costs were incurred on a new project which commenced on 1 October 2014. The research stage of the new project lasted until 31 December 2014 and incurred Rs. 1·4 million of costs. From that date the project incurred development costs of Rs. 800,000 per month. On 1 April 2015 the directors became confident that the project would be successful and yield a profit well in excess of its costs. The project is still in development at 30 September 2015. Capitalised development expenditure is amortised at 20% per annum using the straight-line method. All expensed research and development is charged to cost of sales.

(iii)

Sarhad Sugar Limited is being sued by a customer for Rs. 2 million for breach of contract over a cancelled order. Sarhad Sugar Limited has obtained legal opinion that there is a 20% chance that Sarhad Sugar Limited will lose the case. Accordingly Sarhad Sugar Limited has provided Rs. 400,000 (Rs. 2 million x 20%) included in administrative expenses in respect of the claim. The unrecoverable legal costs of defending the action are estimated at Rs. 100,000. These have not been provided for as the legal action will not go to court until next year.

(iv)

The directors have estimated the provision for income tax for the year ended 30 September 2015 at Rs. 11·4 million. The required deferred tax provision at 30 September 2015 is Rs. 6 million.

Required (a)

Prepare the statement of profit or loss for the year ended 30 September 2015. (10)

(b)

Prepare the statement of financial position as at 30 September 2015.

(10)

Note: notes to the financial statements are not required.

(20)

© Emile Woolf International

9

The Institute of Chartered Accountants of Pakistan

Financial accounting and reporting II

2.7

BSZ LIMITED The post-closing trial balance of BSZ Limited, a listed company, as at June 30, 2015 is given below: Debit Credit Rs. in million Cash at banks – current accounts Cash at banks – in saving accounts Stocks in trade – closing Accounts receivable Provision for bad debts Advances to suppliers Advances to staff Short term deposits Prepayments Sales tax receivable Freehold land – at revalued amount Furniture and fixtures - cost Accumulated depreciation – Furniture and fixtures Machines - cost Accumulated depreciation – Machines Building on freehold land – cost Accumulated depreciation – Building Computer software – cost Accumulated amortization – Computer software Deferred taxation Short term loan Accounts payable Accrued liabilities Provision for taxation Issued, subscribed and paid up capital (Rs. 10 each) Surplus on revaluation of fixed assets Accumulated profits

7 22 90 60 3 16 6 11 4 12 375 27 8 85 27 150 26 10

875

2 40 85 75 7 17 400 120 65 875

Additional Information (i)

The first revaluation of freehold land was carried out in 2011 and resulted in a surplus of Rs. 120 million. The valuation was carried out under market value basis by an independent valuer, Mr. Dee, Chartered Civil Engineer of M/s SSS Consultants (Pvt.) Ltd., Islamabad.

(ii)

The details relating to additions, disposal and depreciation/amortization of fixed assets, during the year 2015 are given below: 

The company uses the straight line method for charging depreciation and amortization. The building is depreciated at a rate of 5% whereas 10% is charged on machines, furniture and fixtures and computer software.



Construction on third floor of the building commenced on March 1, 2015 and is expected to be completed on September 30, 2015. The cost incurred during the year i.e. Rs. 20 million was capitalised on June 30, 2015.

© Emile Woolf International

10

The Institute of Chartered Accountants of Pakistan

Questions



Furniture and fixtures worth Rs. 8 million were purchased on April 1, 2015.



A machine was sold on February 28, 2015 to NJ Enterprise at a price of Rs. 13 million. At the time of disposal, the cost and written down value of the machine was Rs. 15 million and Rs. 10 million respectively.

(iii)

50% of the accounts receivable were secured and considered good. 10% of the unsecured accounts receivable were considered doubtful. Bad debts expenses for the year amounted to Rs. 1.0 million. An amount of Rs. 1.4 million was written off during the year.

(iv)

All advances given to suppliers are considered good and include an amount of Rs. 4.0 million paid for goods which will be supplied on December 31, 2016.

(v)

Cash at banks in saving accounts carry interest / mark-up ranging from 3% to 7% per annum.

(vi)

The authorised share capital of the company is Rs. 500 million.

Required Prepare the statement of financial position as at June 30, 2015 along with the relevant notes showing all possible disclosures as required under the International Accounting Standards and the Companies Ordinance, 1984. (Comparative figures and the note on accounting policies are not required.)(22)

© Emile Woolf International

11

The Institute of Chartered Accountants of Pakistan

Financial accounting and reporting II

2.8

YASIR INDUSTRIES LIMITED The following trial balance related to Yasir Industries Limited (YIL) for the year ended June 30, 2015:

Ordinary share capital (Rs. 10 each) Retained earnings Sales Purchases Production labour Manufacturing overheads Inventories (July 1, 2014) Administrative expenses Distribution expenses Financial charges Cash and bank Trade creditors Accrued expenses 10% redeemable preference shares Debentures Deferred tax (July 1, 2014) Suspense account Leasehold property - at cost Machines – at cost Software – at cost Acc. depreciation – Leasehold property (June 30, 2015) Acc. depreciation – Machines (June 30, 2015) Acc. amortization – Software (June 30, 2015) Trade receivables

Dr Cr Rs. in million 120.00 10.20 472.40 175.70 61.00 39.00 38.90 40.00 19.80 0.30 13.25 30.40 16.20 40.00 80.00 6.00 30.00 230.00 168.60 20.00 40.25 48.60 12.00 66.00 889.30

889.30

Additional Information (i)

Sales include an amount of Rs. 27 million, made to a customer under sale or return agreement. The sale has been made at cost plus 20% and the expiry date for the return of these goods is July 31, 2015.

(ii)

The value of inventories at June 30, 2015 was Rs. 42 million.

(iii)

A fraud of Rs. 30 million was discovered in October 2014. A senior employee of the company who left in June 2014, had embezzled the funds from YIL’s bank account. The chances of recovery are remote. The amount is presently appearing in the suspense account.

(iv)

On January 1, 2015 YIL issued debenture certificates which are repayable in 2020. Interest is paid on these at 12% per annum.

(v)

Financial charges comprise bank charges and bank commission.

(vi)

The provision for current taxation for the year ended June 30, 2015 after making all the above adjustments is estimated at Rs. 16.5 million.

© Emile Woolf International

12

The Institute of Chartered Accountants of Pakistan

Questions

(vii) The carrying value of YIL’s net assets as on June 30, 2015 exceeds their tax base by Rs. 30 million. The income tax rate applicable to the company is 30%. (viii) On July 1, 2014, the leasehold property having a useful life of 40 years was revalued at Rs. 238 million. No adjustment in this regard has been made in the books. (ix)

Depreciation of leasehold property is charged using the straight line method. 50% of depreciation is allocated to manufacturing, 30% to administration and 20% to selling and distribution.

Required In accordance with the requirements of the Companies Ordinance, 1984 and International Accounting Standards, prepare the: (a)

statement of financial position as of June 30, 2015.

(b)

statement of profit or loss for the year ended June 30, 2015.

(20)

(Comparative figures and notes to the financial statements are not required.)

© Emile Woolf International

13

The Institute of Chartered Accountants of Pakistan

Financial accounting and reporting II

2.9

SHAHEEN LIMITED Following is the trial balance of Shaheen Limited (SL) as at June 30, 2015: Rs. in ‘000 Dr Cr 200,000 100,000 35,000 30,000 23,000 5,000 2,000 6,000 86,000

Sales revenue Manufacturing costs Selling and distribution costs Administrative costs Opening inventories Interest on borrowings Provision for income tax Advance income tax paid Property, plant and equipment Accumulated depreciation on property, plant and equipment Export licence Trade receivables Cash and bank balances Other receivable and prepayments Trade payables Provisions for litigation Long term borrowings Deferred tax Share capital (Rs. 10 each and fully paid) Retained earnings

12,000 6,000 37,800 4,725 14,000 12,000 5,000 31,525 5,000 60,000 20,000 347,525

347,525

Additional information (i)

Sales last year (year ended 30 June 2014) included goods invoiced at Rs 10 million which were sent to a customer on June 25, 2014 under a sale or return agreement, at cost plus 20%. The goods were returned on August 25, 2014. No correction has been made for the return.

(ii)

The export licence has been obtained for exporting a new product and is effective for five years up to December 31, 2019. However, the exports commenced from July 1, 2015.

(iii)

Closing inventories are valued at Rs. 30 million.

(iv)

Details of property, plant and equipment are as follows: Land Cost as at June 30, 2014 Fully depreciated amounts included in cost Estimated useful life at the date of purchase

20,000

Plant and Buildings equipment Rs in ‘000 36,000 30,000 3,000 20 years 10 years

The company uses straight line method for charging depreciation. Depreciation is allocated to manufacturing, distribution and administrative costs at 75%, 15% and 10% respectively. (v)

Rs. 6 million of the long term borrowings is of current maturity (i.e. will be repaid within 12 months).

© Emile Woolf International

14

The Institute of Chartered Accountants of Pakistan

Questions

(vi)

During the year Rs. 5 million was paid in full and final settlement of income tax liability against which a provision of Rs. 7.0 million had been made in the previous year. Current year’s taxable income exceeds accounting income by Rs. 5 million of which 0.8 million are permanent differences. Applicable tax rate for the company is 35%.

(vii) On July 30, 2015 the board of directors proposed a final dividend at 15% for the year ended June 30, 2015 (2014: at 20%) Required In accordance with the requirements of the Companies Ordinance, 1984 and International Financial Reporting Standards, prepare: (a)

The statement of financial position as of June 30, 2015

(b)

The statement of profit or loss for the year ended June 30, 2015

(c)

The statement of changes in equity for the year ended June 30, 2015.

(Comparative figures and notes to the financial statements are not required) (25)

2.10

MOONLIGHT PAKISTAN LIMITED Following is the summarised trial balance of Moonlight Pakistan Limited (MPL), a listed company, for the year ended December 31, 2015:

Land and buildings - at cost Plants – at cost Trade receivables Stock in trade at December 31, 2015 Cash and bank Cost of sales Selling expenses Administrative expenses Financial charges Accumulated depreciation as on January 1, 2015 – Buildings Accumulated depreciation as on January 1, 2015 – Plants Ordinary shares of Rs. 10 each fully paid Retained earnings as at January 1, 2015 12% Long term loan Provision for gratuity Deferred tax on January 1, 2015 Trade payables Right subscription received Revenue

© Emile Woolf International

15

Rs. in million Debit Credit 2,600 2,104 702 758 354 1,784 220 250 210 400 670 1,200 510 1,600 8 22 544 420 3,608 8,982 8,982

The Institute of Chartered Accountants of Pakistan

Financial accounting and reporting II

Additional Information (i)

The land and buildings were acquired on January 1, 2011. The cost of land was Rs. 600 million. On January 1, 2015 a professional valuation firm valued the buildings at Rs. 1,840 million with no change in the value of land. The estimated life at acquisition was 20 years and the remaining life has not changed as a result of the valuation. 60% of depreciation on buildings is allocated to manufacturing, 25% to selling and 15% to administration.

(ii)

Plant is depreciated at 20% per annum using the reducing balance method.

(iii)

On March 31, 2015 MPL made a bonus issue of one share for every six held. The issue has not been recorded in the books of account.

(iv)

Right shares were issued on September 1, 2015 at Rs. 12 per share.

(v)

The interest on long term loan is payable on the first day of July and January. No accrual has been made for the interest payable on January 1, 2013.

(vi)

MPL operates an unfunded gratuity scheme for all its eligible employees. The provision required as on December 31, 2015 is estimated at Rs. 23 million. Rs. 3 million were paid during the year and debited to the provision for gratuity account. Cost of gratuity is allocated to production, selling and administration expenses in the ratio of 60% : 20% : 20%.

(vii) The tax charge for the current year after making all related adjustments is estimated at Rs. 37 million. The timing differences related to taxation are estimated to increase by Rs. 80 million, over the last year. The applicable income tax rate is 35%. Required In accordance with the requirements of Companies Ordinance, 1984 and International Financial Reporting Standards, prepare the following: (a)

Statement of Financial Position as of December 31, 2015.

(b)

Statement of profit or loss for the year ended December 31, 2015.

(22)

(Comparative figures and notes to the financial statements are not required)

© Emile Woolf International

16

The Institute of Chartered Accountants of Pakistan

Questions

2.11

FIGS PAKISTAN LIMITED Figs Pakistan Limited is a listed company engaged in the business of manufacturing and marketing of personal care and food products. Following is an extract from its trial balance for the year ended 31 December 2015: Debit

Credit

Rs. in million Sales - Manufactured goods Sales - Imported goods Scrap sales Dividend income Return on savings account Sales tax - Imported goods Sales tax - Manufactured goods Sales discount Raw material stock as on 1 January 2015 Work in process as on 1 January 2015 Finished goods (manufactured) as on 1 January 2015 Finished goods (imported) as on 1 January 2015 Purchases - Raw material Purchases - Imported goods Stores and spares consumed Salaries, wages and benefits Utilities Depreciation and amortization Stationery and office expenses Repairs and maintenance Advertisement and sales promotion Outward freight and handling Legal and professional charges Auditor's remuneration Donations Workers Profit Participation Fund Worker Welfare Fund Loss on disposal of property, plant and equipment Financial charges on short term borrowings Exchange loss Financial charges on lease

56,528 1,078 16 12 2 53 10,201 2,594 1,751 73 1,210 44 22,603 658 180 2,367 734 1,287 230 315 4,040 1,279 71 13 34 257 98 10 133 22 11

Additional information (i)

The position of inventories as at 31 December 2015 was as follows: Raw material Work in process Finished goods (manufactured) Finished goods (imported)

© Emile Woolf International

17

Rs. m 2,125 125 1,153 66

The Institute of Chartered Accountants of Pakistan

Financial accounting and reporting II

(ii)

The basis of allocation of various expenses among cost of sales, distribution costs and administrative expenses are as follows:

Salaries, wages and benefits Depreciation and amortization Stationery and office expenses Repairs and maintenance / Utilities

Cost of Distribution Administrative sales costs expenses % % % 55 30 15 70 20 10 25 40 35 85 5 10

(iii)

Salaries, wages and benefits include contributions to provident fund (defined contribution plan) and gratuity fund (defined benefit plan) amounting to Rs. 54 million and Rs. 44 million respectively.

(iv)

Auditor’s remuneration includes taxation services and out-of-pocket expenses amounting to Rs. 4 million and Rs. 1 million respectively.

(v)

Donations include Rs. 5 million given to Dates Cancer Foundation (DCF). One of the company’s directors, Mr. Peanut is a trustee of DCF.

(vi)

The tax charge for the current year after making all related adjustments is estimated at Rs. 1,440 million. Taxable temporary differences of Rs. 3,120 originated in the year million, over the last year. The applicable income tax rate is 35%.

(vii) 274 million ordinary shares were outstanding as on 31 December 2015. (viii) There is no other comprehensive income for the year. Required Prepare the statement of profit or loss for the year ended 31 December 2015 along with the relevant notes showing required disclosures as per the Companies Ordinance, 1984 and International Financial Reporting Standards. Comparatives are not required. (24)

© Emile Woolf International

18

The Institute of Chartered Accountants of Pakistan

Questions

CHAPTER 3 – IAS 7: STATEMENTS OF CASH FLOWS 3.1

KLEA The statement of financial position and statement of profit or loss for Klea for the year to 31st March 2015 are provided below. Statement of financial position as at 31st March 2015 2015 2014 Rs. in ‘000 Assets Non-current assets Intangible assets Property, plant and equipment Financial assets

300 3,450 400

––––––

4,150

Current assets Inventory Trade receivables Cash and cash equivalents

3,200 2,400 32

2,000 2,000 580

––––––

9,782

––––––

4,580

––––––

6,580

––––––

––––––

3,000 838 910

2,000 560 354

––––––

Total equity

2,000

––––––

5,632

Equity and liabilities Equity Issued share capital Share premium account Retained earnings

––––––

––––––

––––––

Total assets

200 1,600 200

4,748

––––––

2,914

––––––

––––––

Revaluation surplus

1,000

-

Non-current liabilities Interest-bearing loans and liabilities

1,600

2,000

Current liabilities Bank overdraft Trade payables Taxation

414 1,600 420

1,266 400

––––––

2,434

––––––

Total liabilities

4,034

Total equity and liabilities

© Emile Woolf International

19

––––––

1,666

––––––

3,666

––––––

––––––

9,782 ════

6,580 ════

The Institute of Chartered Accountants of Pakistan

Financial accounting and reporting II

Statement of profit or loss for the year ended 31st March 2015 Rs. in ‘000 Revenue 10,000 Other income 100 Change in inventory of finished goods and WIP 1,300 Raw materials and consumables used 4,000 Employee benefits costs 3,000 Depreciation and amortisation expense 800 Other expenses 1,724 ––––––

Total expenses

(9,524) ––––– 1,876 (320) 50 ––––– 1,606 (650) ––––– 956 ════

Finance costs Finance income Profit before tax Income tax expense Profit for the year Additional information (i)

Rs. in ‘000 2014

Non-current assets 2015 Intangible assets Property, plant and equipment

Cost

Deprec’n

Cost

Deprec’n

700

400

400

200

5,000

1,550

3,000

1,400

(ii) (iii)

At 1 April 2014 land was revalued from Rs. 1million to Rs. 2 million. During the year, plant and machinery costing Rs. 600,000 and depreciated by Rs. 500,000 was sold for Rs. 150,000.

(iv)

The interest bearing loans relate to debentures which were issued at their nominal value. Rs. 400,000 of these debentures were redeemed at par during the year.

(v)

Ordinary shares were issued for cash during the year.

(vi)

Rs. 100,000 of current asset investments held as cash equivalents were sold during the year for Rs. 94,000.

(vii) Dividends paid in the year were Rs. 200,000 relating to the 2014 proposed dividend and a Rs. 200,000 interim dividend for 2015. Required Prepare a statement of cash flows for Klea for the year ended 31 March 2015 in accordance with IAS 7 using the indirect method. (25)

© Emile Woolf International

20

The Institute of Chartered Accountants of Pakistan

Questions

3.2

STANDARD INC The summarised statements of financial position of Standard Inc at 31 December 2014 and 2015 are as follows. 2015 2014 Rs. in ‘000 Rs. in ‘000 Issued share capital 150,000 100,000 Share premium 35,000 15,000 Retained earnings 41,000 14,000 Long-term loans 30,000 70,000 Payables 48,000 34,000 Bank overdraft – 14,000 Tax payable 33,000 21,500 Proposed dividends 15,000 7,500 Depreciation Plant and machinery 54,000 45,000 Fixtures and fittings 15,000 13,000 ———— ———— 421,000 334,000 ———— ———— Freehold property at cost Plant and machinery at cost Fixtures and fittings at cost Inventories Trade receivables Long-term investments Cash at bank

130,000 151,000 29,000 51,000 44,000 4,600 11,400 ———— 421,000 ————

110,000 120,000 24,000 37,000 42,800 – 200 ———— 334,000 ————

The following information is relevant: (a) (b)

There had been no disposal of freehold property in the year. A machine tool which had cost Rs. 8,000,000 (in respect of which Rs. 6,000,000 depreciation had been provided) was sold for Rs. 3,000,000, and fixtures which had cost Rs. 5,000,000 (in respect of which depreciation of Rs. 2,000,000 had been provided) were sold for Rs. 1,00,0000. Profits and losses on those transactions had been dealt with through the statement of profit or loss.

(c)

The statement of profit or loss charge in respect of tax was Rs. 22,000,000.

(d)

The premium paid on redemption of the long-term loan was Rs. 2,000,000, which has been written off to the statement of profit or loss.

(e)

The proposed dividend for 2014 had been paid during the year.

(f)

Interest received during the year was Rs. 450,000. Interest charged in the statement of profit or loss for the year was Rs. 6,400,000. Accrued interest of Rs. 440,000 is included in payables at 31 December 2014 (nil at 31 December 2015).

(g)

The government stock is a long term investment.

Required Prepare a cash flow statement for the year ended 31 December 2015, together with notes as required by IAS 7. (20)

© Emile Woolf International

21

The Institute of Chartered Accountants of Pakistan

Financial accounting and reporting II

3.3

FALLEN Fallen has prepared the following rough draft accounts for the year ended 31 December 2015. Statement of profit or loss Revenue Cost of sales Gross profit Distribution costs Administration expenses Interest payable Operating profit before tax Taxation (35%) including deferred tax Profit after tax Dividends Retained profit

Rs. in ‘000 11,563 (5,502) ——— 6,061 (402) (882) (152) ——— 4,625 (1,531) ——— 3,094 (700) ——— 2,394 ———

Statements of financial position

Leasehold premises (net) Plant, machinery and equipment (net) Investments at cost Inventories Receivables Bank

© Emile Woolf International

22

31 December 2015 2014 Rs. in ‘000 Rs. in ‘000 6,600 5,700 5,040 3,780 2,406 2,208 2,880 1,986 2,586 1,992 – 576 ——— ——— 19,512 16,242 ——— ———

The Institute of Chartered Accountants of Pakistan

Questions

31 December 2015 2014 Rs. in ‘000 Rs. in ‘000 2,280 1,800 2,112 1,800 9,108 6,714 202 138 1,240 1,800 1,202 1,016 1,026 702 222 –

Share capital Share premium Profit and loss account Deferred taxation Long-term loan (10%) Provision for deferred repairs Payables Overdraft Taxation Corporation tax Proposed dividends

1,730 390 ——— 19,512 ———

2,038 234 ——— 16,242 ———

The following data is relevant. (1)

The 10% long-term loan were redeemed at par.

(2)

Plant and equipment with a written down value of Rs. 276,000 was sold for Rs. 168,000. New plant was purchased for Rs. 2,500,000.

(3)

Leasehold premises costing Rs. 1,300,000 were acquired during the year.

(4)

The investments are highly liquid securities held for the short term.

Required Prepare the cash flow statement and supporting notes in accordance with IAS 7 for Fallen Inc for 2015. (20)

© Emile Woolf International

23

The Institute of Chartered Accountants of Pakistan

Financial accounting and reporting II

3.4

BIN QASIM MOTORS LIMITED The summarised financial statements of Bin Qasim Motors Limited for the year to 30 September 2015, together with a comparative balance sheet, are: Statement of profit or loss

Rs. 000

Sales revenue

7,482

Cost of sales

(4,284)

Gross profit

3,198

Operating expenses

(1,479)

Interest payable

(260)

Investment income

120

Profit before tax

1,579

Income tax

(520)

Profit for the period

1,059

Statement of financial position as at 30 September

Assets Non-current assets Property, plant and equipment Investment Current assets Inventory Trade accounts receivable Short term treasury bills Bank Total assets Total equity and liabilities Equity: Share capital Reserves: Share premium Retained earnings At beginning of the year Net profit for period Dividends At end of the year

© Emile Woolf International

24

2015 Rs. in ‘000

2014 Rs. in ‘000

2,344 690 3,034

1,908 nil 1,908

1,046 935 120 nil 2,101 5,135

785 824 50 122 1,781 3,689

1,400

1,000

460

60

192 1,059 (180) 1,071 2,931

147 65 (20) 192 1,252

The Institute of Chartered Accountants of Pakistan

Questions

Revaluation surplus Non-current liabilities Deferred tax Deferred income 10% Convertible loan stock Current liabilities Trade accounts payable Accrued interest Provision for negligence claim Provision for income tax Deferred income Overdraft Total equity and liabilities

90

40

439 275 nil 714

400 200 400 1,000

644 40 nil 480 100 136 1,400 5,135

760 25 120 367 125 nil 1,397 3,689

The following information is relevant (i)

Non-current assets Rs in ‘000

Property, plant and equipment is analysed as follows: 30 September 2015

30 September 2014

Cost/

Cost/

Valuation Depreciation

NBV

Valuation Depreciation

NBV

Land and buildings

2,000

760

1,240

1,800

680

1,120

Plant

1,568

464

1,104

1,220

432

788

3,568

1,224

2,344

3,020

1,112

1,908

On 1 October 2014 Bin Qasim Motors Limited recorded an increase in the value of its land of Rs. 150,000. During the year an item of plant that had cost Rs. 500,000 and had accumulated depreciation of Rs. 244,000 was sold at a loss (included in cost of sales) of Rs. 86,000 on its carrying value. (ii)

Deferred income Bin Qasim Motors Limited sells servicing contracts on certain types of machinery. Payments are received in advance for a service which Bin Qasim Motors Limited must provide over a number of following years. Income that relates to these contracts is deferred and recognised in P&L as the period of service passes. A credit of Rs. 125,000 for the current year’s recognition of deferred income has been included revenue in this period.

(iii)

Share capital and loan stocks The increase in the share capital during the year was due to the following events: (1)

© Emile Woolf International

On 1 January 2015 there was a bonus issue (out of the revaluation surplus) of one bonus share for every 10 shares held.

25

The Institute of Chartered Accountants of Pakistan

Financial accounting and reporting II

(iv)

(2)

On 1 April 2015 the 10% convertible loan stock holders exercised their right to convert to ordinary shares. The terms of conversion were 25 ordinary shares of Rs. 1 each for each Rs. 100 of 10% convertible loan stock.

(3)

The remaining increase in the ordinary shares was due to a stock market placement of shares for cash on 12 August 2015.

Provision for negligence claim In June 2015 Bin Qasim Motors Limited made an out of court settlement of a negligence claim brought about by a former employee. The dispute had been in progress for two years and Bin Qasim Motors Limited had made provisions for the potential liability in each of the two previous years. The unprovided amount of the claim at the time of settlement was Rs. 30,000 and this was charged to operating expenses.

Required Prepare a statement of cash flows for Bin Qasim Motors Limited for the year to 30 September 2015 in accordance with IAS 7 Statement of Cash Flows. (25)

© Emile Woolf International

26

The Institute of Chartered Accountants of Pakistan

Questions

3.5

ITTEHAD MANUFACTURING LTD The financial statements of Ittehad Manufacturing Ltd for the year to 30 September 2015, together with the comparative statement of financial position (balance sheet) for the year to 30 September 2014 are shown below: Rs. in million Sales revenue Cost of sales (note 1) Gross profit for period Operating expenses (note 1)

3,820 (2,620) 1,200 (280) 920 (30) 890 (270) 620

Interest – Loan note Profit before tax Taxation Net profit for the period Statement of financial position as at 30 September: 2015 Rs. in million Non-current assets Property, plant and equipment Intangible assets (note 2) Current assets Inventory Accounts receivable Cash Total assets Equity and liabilities Ordinary shares of Rs. 1 each Reserves Share premium Revaluation Retained earnings Non-current liabilities (note 3) Current liabilities (note 4) Total equity and liabilities

2016 Rs. in million

1,890

1,830

670 2,560

300 2,130

1,420 990 70 2,480 5,040

940 680 nil 1,620 3,750

750

500

350 190 1,860 3,150 610 1,280 5,040

100 nil 1,600 2,200 240 1,310 3,750

Extract from statement of changes in equity 2015 2014 Rs. in million 1,600 1,000 620 800 (320) (200) (50) 10 – 1,860 1,600

Retained earnings – brought forward Profit for the year Dividends Bonus issue Transfer from revaluation surplus Retained earnings – carried forward

© Emile Woolf International

27

The Institute of Chartered Accountants of Pakistan

Financial accounting and reporting II

Notes to the financial statements: (1) Cost of sales includes depreciation of property, plant and equipment of Rs. 320 million and a loss on the sale of plant of Rs. 50 million. 2015

2014

Rs. in million

(2)

(3)

(4)

Intangible non-current assets: Deferred development expenditure

470

100

Goodwill

200

200

670

300

10% loan note

300

100

Deferred tax

310

140

610

240

875

730

Bank overdraft

nil

115

Accrued loan interest

15

5

Deferred income

260

300

Taxation

130

160

1,280

1,310

Non-current liabilities:

Current liabilities: Accounts payable

The following additional information is relevant: (i)

Intangible non-current assets:

(ii)

The company successfully completed the development of a new product during the current year, capitalising a further Rs. 500 million before amortisation charges for the period. Property, plant and equipment/revaluation surplus:

(iii)



The company revalued its buildings by Rs. 200 million on 1 October 2014. The surplus was credited to a revaluation surplus.



New plant was acquired during the year at a cost of Rs. 250.



Rs. 10 million has been transferred from the revaluation surplus to retained earnings as a year-end adjustment in respect of the additional depreciation created by the revaluation.



The remaining movement on property, plant and equipment was due to the disposal of obsolete plant.

Share issues: On 1 October 2014 a bonus issue of 1 new share for every 10 held was made from retained earnings. Ittehad Manufacturing Ltd made a further issue of ordinary shares for cash during the year.

Required (a)

A statement of cash flows for Ittehad Manufacturing Ltd for the year to 30 September 2015 prepared in accordance with IAS 7 Statement of Cash Flows. (20)

(b)

Comment briefly on the financial position of Ittehad Manufacturing Ltd as portrayed by the information in your statement of cash flows.

(5) (25)

© Emile Woolf International

28

The Institute of Chartered Accountants of Pakistan

Questions

3.6

WASEEM INDUSTRIES LIMITED The following statements of financial position relate to Waseem Industries Limited for the years ended December 31:

ASSETS Non-current assets Fixed assets Property, plant and equipment Capital work-in-progress Long term investments Long term deposits Total non-current assets

2015

2014

Rs. in million

Rs. in million

242 20 262 75 13 350

182 18 200 100 13 313

Current assets Stocks-in-trade Trade debts Advances, prepayments and other receivables Cash and bank balances Total current assets

55 51

48 38

37 11 154

40 20 146

TOTAL ASSETS

504

459

150 55 85 290

125 80 50 255

94 16 110

118 12 130

25 13 66 104

22 6 46 74

504

459

EQUITY AND LIABILITIES Shareholders' equity Share capital Share premium Unappropriated profit Non-current liabilities Long term finances - Secured Deferred liability - Gratuity (unfunded) Current liabilities Current portion of long term finances Short term finances Trade and other payables TOTAL EQUITY AND LIABILITIES

© Emile Woolf International

29

The Institute of Chartered Accountants of Pakistan

Financial accounting and reporting II

Other relevant information is as follows: (i)

An interim bonus issue of one for five ordinary shares was made during the year out of share premium. The company also approved final cash dividend of 10% (2014: 8%), in its annual general meeting.

(ii)

During the year, the company provided Rs. 17 million (2014: Rs. 13 million) on account of depreciation. The details relating to disposal of property, plant and equipment are as follows: Carrying amount Sale proceeds Rs. m Plant and machinery Vehicles

20 3

Rs. m 22 4

(iii)

Advances, prepayments and other receivables include advance tax of Rs. 10 million (2014: Rs. 7 million).

(iv)

In 2015, the company paid Rs. 6 million on account of gratuity.

(v)

Accrued mark-up on long term finances amounting to Rs. 7 million (2014: Rs. 9 million) is included in trade and other payables. Financial charges included in the profit and loss account are Rs. 16 million (2014 : Rs. 14 million).

(vi)

Income tax expense for the year 2015 amounted to Rs. 19 million (2014: Rs. 13 million).

Required Prepare a cash flow statement in accordance with the requirements of IAS 7 Cash Flow Statement” using the indirect method. (20)

© Emile Woolf International

30

The Institute of Chartered Accountants of Pakistan

Questions

3.7

JALIB INDUSTRIES LIMITED Jalib Industries Limited is a listed company. The relevant information contained in the financial statements for the year ended December 31, 2015 is as follows: Statement of Financial Position 2015

2014

Rupees in million

Non-current assets Property, plant and equipment Capital work in progress Current assets Stock in trade Trade debts Advances and other receivables Cash and bank

Equity Issued, subscribed and paid-up capital Share premium Unappropriated profit Non-current liabilities Deferred liabilities Long term loans Current liabilities Current portion of long term loans Creditors, accrued and other liabilities Dividend payable

129.40 22.50 151.90

100.60 37.00 137.60

531.80 28.50 37.40 12.00 609.70 761.60

451.00 24.70 42.00 3.00 520.70 658.30

396.00 45.00 142.60 583.60

300.00 12.00 163.00 475.00

40.80 80.00 120.80

27.50 100.00 127.50

18.00 36.20 3.00 57.20 761.60

20.00 34.40 1.40 55.80 658.30

Statement of profit or loss 2015 Rupees in million

Sales Cost of goods sold Gross profit Operating expenses Financial charges Loss on sale of fixed assets

2,535.00 (1,774.50) 760.50 (554.00) (10.50) (4.60) (569.10) 191.40 (104.60) (2.20) (106.80) 84.60

Profit before tax Tax expense - Current - Deferred Profit after tax

© Emile Woolf International

31

The Institute of Chartered Accountants of Pakistan

Financial accounting and reporting II

The following supporting information is available: (i)

During the year, an amount of Rs. 42 million was transferred from capital work in progress to property, plant and equipment.

(ii)

The company sold property, plant and equipment having book value of Rs. 15 million for Rs. 10.4 million.

(iii)

Depreciation for the year amounted to Rs. 27.7 million.

(iv)

Trade debts written off during the year amounted to Rs. 1 million. It is the policy of the company to maintain the provision for doubtful debts at 5% of trade debts.

(v)

Advances and other receivables include advance tax of Rs. 3.6 million (2014: Rs. 2.2 million).

(vi)

Deferred liabilities include deferred tax and provision for gratuity. There was no deferred tax liability at the beginning of the year. Provision for gratuity made during the year amounted to Rs. 15.5 million.

(vii) Creditors, accrued and other liabilities include accrued financial charges amounting to Rs. 5 million (2014: Rs. 6 million). (viii) On January 15, 2016, the company declared final dividend for the year ended December 31, 2015 comprising 7.5% (2014: 25%) cash dividend and 12.5% (2014:10%) bonus shares, for its ordinary shareholders. Required Prepare a statement of cash flow for the year ended December 31, 2015 in accordance with the requirements of International Accounting Standards. Show all necessary workings. (23)

3.8

APOLLO INDUSTRY LIMITED Following are the relevant extracts from the financial statements of Apollo Industry Limited, a listed company, for the year ended December 31, 2015. Statement of financial position as at December 31, 2015

Issued, subscribed and paid up capital Unappropriated profit Surplus on revaluation of property, plant & equipment Non-current liabilities Staff gratuity Deferred tax liability- net Trade and other payables

© Emile Woolf International

32

2015

2014

Rs. 000

Rs. 000

25,000 20,900 45,900

20,000 22,000 42,000

7,000

8,000

1,400 590 1,990 4,200 59,090

1,190 1,190 6,250 57,440

The Institute of Chartered Accountants of Pakistan

Questions

Property, plant and equipment Capital work in progress Intangible assets Deferred tax asset- net Long term deposits and prepayments Current Assets Tax refundable Other current assets Cash and bank balances

2015

2014

Rs. 000

Rs. 000

35,000 5,500 1,100 41,600 400 42,000

25,500 10,000 1,140 36,640 350 300 37,290

950 15,700 440 17,090 59,090

800 12,125 7,225 20,150 57,440

Statement of comprehensiveiIncome for the year ended December 31, 2015 2015 Rs. 000 146,700 (127,500) 19,200

Sales Cost of sales Gross profit Operating expenses Financial charges Other income

(15,000) (500) 2,800 (12,700) 6,500 (4,660) (940) (5,600) 900

Profit before tax Tax expense - current - deferred Tax for the year Profit after tax Other relevant information is as under: (i)

During the year, the company has issued 10% bonus shares.

(ii)

Depreciation and amortization for the year amounted to Rs. 7 million.

(iii)

WDV of assets disposed off during the year amounted to Rs. 1.2 million. (The assets had not been revalued)

(iv)

Other income includes interest earned on short term placements, amounting to Rs. 1 million. The remaining amount represents gain on disposal of property, plant and equipment.

(v)

Gratuity of Rs. 0.3 million was paid to outgoing employees.

(vi)

Intangible assets worth Rs. 50 thousand were acquired during the year.

Required Prepare the Statement of Cash Flows for the year ended December 31, 2015 in accordance with the requirements of IAS - 7 (Statement of Cash Flows) using ‘indirect method’. (22)

© Emile Woolf International

33

The Institute of Chartered Accountants of Pakistan

Financial accounting and reporting II

3.9

MARVEL ENGINEERING LIMITED Following are the extracts from the draft financial statements of Marvel Engineering Limited (MEL), a listed company, for the year ended 30 June 2015: Statement of Financial Position Rs. in million 2015 Non current assets Property, plant and equipment Long term investments

130

Share capital and reserves 410 Share capital (Rs. 10 each) 100 Share premium

763

510 Retained earnings

97

68 Non current liabilities 57 Long term loans 120 Gratuity payable 39 Deferred taxation 284 Current liabilities Trade and other payables Tax payable - net Dividend payable

633

Current assets Stock-in-trade Trade debts Other receivables Cash at bank

2014

133 100 31 361

1,124

794

2015

2014

494

440

8

-

133 635

110 550

330 55 15 400

110 50 21 181

73

56

12 4 89 1,124

5 2 63 794

Statement of profit or loss 2015 Rs. in million 654 (458) 196 (68) (75) 35 (108) 88 (21) 67

Revenue Cost of sales Gross profit Operating expenses Financial charges Other income Profit before tax Income tax expense Profit after tax Additional information: (i)

During the year, the company recognised a provision for impairment in respect of one of its plant, amounting to Rs. 11 million. Total depreciation for the year amounted to Rs. 50 million.

(ii)

It is the policy of the company to maintain a provision for doubtful debts at 5% of trade debts. During the year, trade debts amounting to Rs. 6 million (2014: Rs. 2 million) were written off.

(iii)

Trade and other payables include accrued financial charges amounting to Rs. 7 million (2014: Rs. 3 million).

© Emile Woolf International

34

The Institute of Chartered Accountants of Pakistan

Questions

(iv)

On 15 July 2015, MEL’s board of directors proposed a final dividend of 10% for the year ended 30 June 2015 (2014: 5% cash dividend and 5% bonus declared on 20 July 2014).

(v)

Other income comprises of the following: Dividend income Gain on sale of vehicles (carrying value of Rs. 5 million) Gain on sale of investments (carrying value of Rs. 10 million)

(vi)

Rs. m 30 2 3 35

Gratuity paid during the year amounted to Rs. 6 million.

Required Prepare the statement of cash flows for Marvel Engineering Limited for the year ended 30 June 2015. (24)

© Emile Woolf International

35

The Institute of Chartered Accountants of Pakistan

Financial accounting and reporting II

CHAPTER 4 – CONSOLIDATED ACCOUNTS: STATEMENTS OF FINANCIAL POSITION – BASIC APPROACH 4.1

HALL Statements of financial position at 31 December 2015

Assets Non-current assets Property, plant and equipment Investment in Stand Current assets

Equity and liabilities Capital and reserves Share capital Retained earnings

Non-current liabilities 8% Debenture loans Current liabilities

Hall Rs. 000

Stand Rs. 000

35,000 12,000

20,000 –

16,000 ——— 63,000 ———

14,000 ——— 34,000 ———

10,000 13,000 ——— 23,000

4,000 12,000 ——— 16,000

20,000

9,000

20,000 ——— 63,000 ———

9,000 ——— 34,000 ———

On 1 January 2013 Hall acquired 75% of Stand for Rs. 12,000,000. At that date the balance on Stand’s retained earnings was Rs. 8,000,000. Required Prepare the consolidated statement of financial position of Hall as at 31 December 2015. (6)

© Emile Woolf International

36

The Institute of Chartered Accountants of Pakistan

Questions

4.2

HASSLE Statements of financial position at 31 December 2015 Hassle Rs. Investment in Strife 60,000 Sundry assets 247,500 ———– 307,500 ———– Share capital Retained earnings Liabilities

Strife Rs. – 226,600 —–—— 226,600 ——–—

120,000 87,500 100,000 ———– 307,500 ———–

50,000 70,000 106,600 ——–— 226,600 ——–—

Hassle bought 80% of Strife when the balance on Strife’s retained profit was Rs. 50,000. Required Prepare the consolidated statement of financial position at 31 December 2015.

4.3

(8)

HYMN The following are the summarised statements of financial position of a group of companies as at 31 December 2015. Hymn Rs. Assets Non-current assets Property, plant and equipment Investment Current assets

Psalm Rs.

105,000 65,000 85,000 220,000 55,000 ———– ———– 410,000 120,000 ———– ———–

Equity and liabilities Equity Share capital Retained earnings

100,000 155,000 ———– 255,000 155,000 ———– 410,000 ———–

Current liabilities

50,000 49,000 ———– 99,000 21,000 ———– 120,000 ———–

Hymn purchased 80% of Psalm’s shares on 1 January 2015 when there was a credit balance on that company’s retained earnings of Rs. 20,000. Required Prepare the Hymn group consolidated statement of financial position as at 31 December 2015.

© Emile Woolf International

37

(6)

The Institute of Chartered Accountants of Pakistan

Financial accounting and reporting II

4.4

HANG On 31 December 2012, Hang acquired 60% of Swing for Rs. 140,000. At that date Swing had a retained earnings balance of Rs. 50,000 and a share premium account balance of Rs. 49,000. The following statements of financial position have been prepared as at 31 December 2015. Hang Rs. Assets Non-current assets Property, plant and equipment Investment in Swing Current assets

Equity and liabilities Equity Share capital Share premium Retained earnings Current liabilities

Swing Rs.

240,000 140,000 250,000 ———– 630,000 ———–

180,000

200,000 25,000 180,000 ———– 405,000 225,000 ———– 630,000 ———–

90,000 49,000 80,000 ———– 219,000 157,000 ———– 376,000 ———–

196,000 ———– 376,000 ———–

Required Prepare the consolidated statement of financial position of Hang and its subsidiary as at 31 December 2015. (6)

4.5

HASH Statements of financial position at 31 December 2015 Hash Rs. 000 Investment in Stash (80%) 100,000 Sundry assets 207,500 ———– 307,500 ———– Share capital Retained earnings Liabilities

120,000 87,500 100,000 ———– 307,500 ———– Hash purchased the shares in Stash on 30th September 2015.

Stash Rs. 000 – 226,600 —–—— 226,600 ——–— 50,000 70,000 106,600 ——–— 226,600 ——–—

Stash’s retained profit for the year ended 31st December 2015 was Rs. 24,000,000. Required Prepare the consolidated statement of financial position at 31 December 2015.

© Emile Woolf International

38

(8)

The Institute of Chartered Accountants of Pakistan

Questions

CHAPTER 5 – CONSOLIDATED ACCOUNTS: STATEMENTS OF FINANCIAL POSITION – COMPLICATIONS 5.1

HAIL The following are the draft statements of financial position of Hail and its subsidiary Snow as at 31 December 2015.

Assets Non-current assets Property, plant and equipment Investments Current assets Cash Trade receivables Snow current account Inventory

Equity and liabilities Shareholders’ equity Share capital Retained earnings Share premium Capital reserve

-

Current liabilities Hail current account

Hash Rs. 000

Stash Rs. 000

161,000 68,000

85,000

7,700 92,500 15,000 56,200 ———– 400,400 ———–

25,200 45,800 36,200 ——–— 192,200 ——–—

100,000 185,400 20,000 ———– 285,400 115,000 ———– 400,400 ———–

50,000 41,200 5,000 ——–— 116,200 68,000 8,000 ——–— 192,200 ——–—

Notes (1)

Snow has 50,000 shares in issues. Hail acquired 45,000 of these on 1 January 2012 for a cost of Rs. 65,000,000 when the balances on Snow’s reserves were Rs. 000 5,000

Share premium account Capital reserve Retained earnings

– 10,000

(2)

Hail declared a dividend of Rs. 3,000,000 before the year end and Snow declared one of Rs. 2,000,000. These transactions have not been accounted for.

(3)

The current account difference is due to cash in transit.

Required Prepare the consolidated statement of financial position as at 31 December 2015 of Hail. (12)

© Emile Woolf International

39

The Institute of Chartered Accountants of Pakistan

Financial accounting and reporting II

5.2

HAIRY The summarised statements of financial position of Hairy and Spider as at 31 December 2015 were as follows. Hairy Spider Rs. 000 Rs. 000 Assets Non-current assets Property, plant and equipment Investments

120,000 55,000

Current assets Cash Investments Trade receivables Current account – Hairy Inventory

60,000 –

11,000 4,000 – 3,000 72,600 19,100 – 3,200 17,000 11,000 ———– ———– 275,600 100,300 ———– ———–

Equity and liabilities Share capital Share premium Capital reserve Retained earnings Trade payables Current account – Spider

100,000 60,000 20,000 – 23,000 16,000 91,900 7,300 38,000 17,000 2,700 – ———– ———– 275,600 100,300 ———– ———–

The following information is relevant. (1)

On 31 December 2012, Hairy acquired 48,000 shares in Spider for Rs. 55,000,000 cash. Spider has 60,000 shares in total.

(2)

The inventory of Hairy includes Rs. 4,000,000 goods from Spider invoiced to Hairy at cost plus 25%.

(3)

The difference on the current account balances is due to cash in transit.

(4)

The balance on Spider’s retained earnings was Rs. 2,300,000 at the date of acquisition. There has been no movement in the balance on Spider’s capital reserve since the date of acquisition.

Required Prepare the consolidated statement of financial position of Hairy and its subsidiary Spider as at 31 December 2015. (12)

© Emile Woolf International

40

The Institute of Chartered Accountants of Pakistan

Questions

5.3

HARD On 31 December 2011, Hard acquired 60% of the ordinary share capital of Soft for Rs. 110 million. At that date Soft had a retained earnings balance of Rs. 50 million and a share premium account balance of Rs. 10 million. The following statements of financial position have been prepared as at 31 December 2015.

Assets Non-current assets Property, plant and equipment Investments in Soft Current assets

Equity and liabilities Capital and reserves Share capital Share premium Retained earnings

Current liabilities

Hard Rs. 000

Soft Rs. 000

225,000 110,000

175,000

271,000 ———– 606,000 ———–

157,000 ———– 332,000 ———–

100,000 15,000 260,000 ———– 375,000 231,000 ———– 606,000 ———–

100,000 10,000 80,000 ———– 190,000 142,000 ———– 332,000 ———–

During the year to 31 December 2015 Hard sold a tangible asset to Soft for Rs. 50 million. The asset was originally purchased in the year to 31 December 2012 at a cost of Rs. 100 million and had a useful economic life of five years. Soft’s depreciation policy is 25% per annum based on cost. Both companies charge a full year’s depreciation in the year of acquisition and none in the year of disposal. Required Prepare the consolidated statement of financial position of Hard and its subsidiary as at 31 December 2015. (12)

© Emile Woolf International

41

The Institute of Chartered Accountants of Pakistan

Financial accounting and reporting II

5.4

HALE On 1 July 2012 Hale acquired 128,000 of Sowen’s 160,000 shares. The following statements of financial position have been prepared as at 31 December 2015. Hale Sowen Rs. 000 Rs. 000 Property, plant and equipment Investment in Sowen Inventory at cost Receivables Bank balance

152,000 203,000 112,000 104,000 41,000 ———– 612,000 ═════

129,600 – 74,400 84,000 8,000 ———– 296,000 ═════

Hale Sowen Rs. 000 Rs. 000 Share capital Retained earnings Payables

100,000 460,000 52,000 ———– 612,000 ═════

160,000 112,000 24,000 ———– 296,000 ═════

The following information is available. (1)

At 1 July 2012 Sowen had a debit balance of Rs. 11 million on retained earnings.

(2)

Property, plant and equipment of Sowen included land at a cost of Rs. 72 million. This land had a fair value of Rs. 100,000 at the date of acquisition.

(3)

The inventory of Sowen includes goods purchased from Hale for Rs. 16 million. Hale invoiced those goods at cost plus 25%.

Required Prepare the consolidated statement of financial position of Hale as at 31 December 2015. (12)

5.5

HELLO On 1 January 2012, Hello acquired 60% of the ordinary share capital of Solong for Rs. 110,000. At that date Solong had a retained earnings balance of Rs. 60,000. The following statements of financial position have been prepared as at 31 December 2015.

Assets Non-current assets Property, plant and equipment Investments in Solong Current assets

© Emile Woolf International

42

Hello Rs.

Solong Rs.

225,000 110,000

175,000

271,000 ———– 606,000 ———–

157,000 ———– 332,000 ———–

The Institute of Chartered Accountants of Pakistan

Questions

Equity and liabilities Capital and reserves Share capital Retained earnings

100,000 275,000 ———– 375,000 231,000 ———– 606,000 ———–

Current liabilities

100,000 90,000 ———– 190,000 142,000 ———– 332,000 ———–

The fair value of Solong’s net assets at the date of acquisition was determined to be Rs. 170,000. The difference between the book value and the fair value of the new assets at the date of acquisition was due to an item of plant which had a useful life of 10 years from the date of acquisition. Required Prepare the consolidated statement of financial position of Hello and its subsidiary as at 31 December 2015. (12)

5.6

HASAN LIMITED On 1 April 2014, Hasan Limited acquired 90% of the equity shares in Shakeel Limited. On the same day Hasan Limited accepted a 10% loan note from Shakeel Limited for Rs. 200,000 which was repayable at Rs. 40,000 per annum (on 31 March each year) over the next five years. Shakeel Limited’s retained earnings at the date of acquisition were Rs. 2,200,000. Statements of financial position as at 31 March 2015 Hasan Limited Rs. 000

Shakeel Limited Rs. 000

2,120 – 4,110 200

1,990 1,800 – –

65 6,495

210 4,000 560 328 –

Total assets

719 524 75 20 1,338 7,833

888 4,888

Equity and liabilities: Capital and reserves Equity shares of Rs. 1 each Share premium

2,000 2,000

1,500 500

Non-current assets Property, plant and equipment Intangible – software Investments – equity in Shakeel Limited Investments – 10% loan note Shakeel Limited Investments – others Current assets Inventories Trade receivables Shakeel Limited current account Cash

© Emile Woolf International

43

The Institute of Chartered Accountants of Pakistan

Financial accounting and reporting II

Retained earnings Non-current liabilities 10% Loan note from Hasan Limited Government grant Current liabilities Trade payables Hasan Limited current account Income taxes payable Operating overdraft Total equity and liabilities

2,900 6,900

1,955 3,955

– 230 230

160 40 200

475 – 228 – 703 7,833

472 60 174 27 733 4,888

The following information is relevant: (i)

Included in Shakeel Limited’s property at the date of acquisition was a leasehold property recorded at its depreciated historical cost of Rs. 400,000. The leasehold had been sub-let for its remaining life of only four years at an annual rental of Rs. 80,000 payable in advance on 1 April each year. The directors of Hasan Limited are of the opinion that the fair value of this leasehold is best reflected by the present value of its future cash flows. An appropriate cost of capital for the group is 10% per annum. The present value of a Rs. 1 annuity received at the end of each year where interest rates are 10% can be taken as: 3 year annuity

(ii)

Rs. 2.50

4 year annuity Rs. 3.20 The software of Shakeel Limited represents the depreciated cost of the development of an integrated business accounting package. It was completed at a capitalised cost of Rs. 2,400,000 and went on sale on 1 April 2013. Shakeel Limited’s directors are depreciating the software on a straight-line basis over an eight-year life (i.e. Rs. 300,000 per annum). However, the directors of Hasan Limited are of the opinion that a five-year life would be more appropriate as sales of business software rarely exceed this period.

(iii)

The inventory of Hasan Limited on 31 March 2015 contains goods at a transfer price of Rs. 25,000 that were supplied by Shakeel Limited who had marked them up with a profit of 25% on cost. Unrealised profits are adjusted for against the profit of the company that made them.

(iv)

On 31 March 2015 Shakeel Limited remitted to Hasan Limited a cash payment of Rs. 55,000. This was not received by Hasan Limited until early April. It was made up of an annual repayment of the 10% loan note of Rs. 40,000 (the interest had already been paid) and Rs. 15,000 of the current account balance.

(v)

The accounting policy of Hasan Limited for non-controlling interests (NCI) in a subsidiary is to value NCI at a proportionate share of the net assets.

(v)

An impairment test at 31 March 2015 on the consolidated goodwill concluded that it should be written down by Rs. 120,000. No other assets were impaired.

Required: Prepare the consolidated statement of financial position of Hasan Limited as at 31 March 2015. (Total: 25 marks)

© Emile Woolf International

44

The Institute of Chartered Accountants of Pakistan

Questions

CHAPTER 6 – CONSOLIDATED ACCOUNTS: STATEMENTS OF COMPREHENSIVE INCOME 6.1

HARRY The following are the statements of profit or loss for the year ended 31 December 2015 of Harry and its subsidiary Sally.

Revenue Cost of sales Gross profit Distribution costs Administration costs Operating profit Investment income Finance costs Profit before tax Income tax expense Profit for the year

Retained profit brought forward Profit for year Dividends paid and proposed Retained profit carried forward

Harry Rs. 000

Sally Rs. 000

1,120 (610) 510 (50) (55) 405 20 (18) 407 (140) 267

390 (220) 170 (40) (45) 85 4 (4) 85 (25) 60

Rs. 000

Rs. 000

100 267 (50) 317

45 60 (20) 85

The following information is relevant. (1)

Harry acquired 75% of Sally six years ago when Sally’s retained earnings were Rs. 9,000.

(2)

Harry made sales to Sally totalling Rs. 100,000 in the year. At the year end the statement of financial position of Sally included inventory purchased from Harry. Harry had taken a profit of Rs. 3,000 on this inventory.

(3)

Harry’s investment income includes Rs. 15,000 being its share of Sally’s dividends.

Required Prepare a consolidated statement of profit or loss and a working showing the movement on consolidated retained profit for the year ended 31 December 2015.(10)

© Emile Woolf International

45

The Institute of Chartered Accountants of Pakistan

Financial accounting and reporting II

6.2

HORNY Statements of profit or loss for the year ended 31 December 2015. Horny Rs. 000 304,900 (144,200) 160,700 (76,450) 84,250 10,500 94,750

Revenue Cost of sales Gross profit Operating costs Operating profit Investment income Profit before tax Income tax expense(42,900) Profit for the year

51,850

Smooth Rs. 000 195,300 (98,550) 96,750 (52,100) 44,650 2,600 47,250 (16,500) 30,750

Statement of changes in equity (extracts) for the year ended 31 December 2015. Horny Rs. 000 80,200 51,850 (20,000) 112,050

Retained earnings brought forward Profit for the year Proposed ordinary dividend

Smooth Rs. 000 31,000 30,750 61,750

The following information is also available. (1)

Horny acquired 75% of the share capital of Smooth on 31 August 2015.

(2)

Negative goodwill of Rs. 3.8 million arose on the acquisition.

(3)

Profits of both companies are deemed to accrue evenly over the year except for the investment income of Smooth all of which was received in November 2015.

(4)

Horny has bought goods from Smooth throughout the year at Rs. 2 million per month. At the year-end Horny does not hold any inventory purchased from Smooth.

Required Prepare the consolidated statement of profit or loss and a working showing the movement on consolidated retained profit for the year ended 31 December 2015.(10)

© Emile Woolf International

46

The Institute of Chartered Accountants of Pakistan

Questions

6.3

HERON Statements of financial position as at 30 June 2015 Assets Non-current assets Property, plant and equipment Investment in Stork ( 1,000 ordinary shares) Current assets

Heron Rs. 000

Stork Rs. 000

31,000 1,000 32,000 23,000 55,000

15,000 15,000 11,000 26,000

Shareholders’ equity and liabilities Share capital (Rs. 0001 ordinary shares) Share premium Retained earnings

10,000 1,500 5,000 – 20,000 18,500 35,000 20,000 Non-current liabilities 15,000 – Current liabilities 5,000 6,000 55,000 26,000 Heron acquired its shares in Stork when the balance on the retained earnings was Rs. 000nil. Statements of profit or loss for the year ended 30 June 2015 Heron Rs. 000 Revenue 30,000 Cost of sales (9,000)

Stork Rs. 000 25,000 (10,000)

Gross profit Distribution costs Administrative expenses Finance costs

21,000 (3,000) (1,000) (2,000)

15,000 (1,200) (2,800) –

Profit before tax Income tax expense

15,000 (3,000)

11,000 (3,000)

Profit for the period

12,000 ════

8,000 ════

Statement of changes in equity for the year ended 30 June 2015 (extract) Retained earnings brought forward Profit for the financial year

8,000 12,000 ——— 20,000 ════

Retained earnings carried forward

10,500 8,000 ——— 18,500 ════

Required Prepare Heron’s consolidated statement of profit or loss, consolidated statement of financial position and a working showing the movement on consolidated retained profit for Heron for the year ended 30 June 2015. (12)

© Emile Woolf International

47

The Institute of Chartered Accountants of Pakistan

Financial accounting and reporting II

6.4

HANKS Statements of financial position as at 31 December 2015

Assets Non-current assets Property, plant and equipment Investments Current assets Cash at bank and in hand Trade receivables Inventory

Equity and liabilities Share capital Share premium account Retained earnings

Current liabilities

Hanks Rs. 000

Streep Rs. 000

Scott Rs. 000

32,000 33,500 ———– 65,500

25,000 – ——— 25,000

20,000 – ——— 20,000

9,500 20,000 30,000 ———– 125,000 ———–

2,000 8,000 18,000 ——— 53,000 ———

4,000 17,000 18,000 ——— 59,000 ———

40,000 6,500 55,000 ———– 101,500

10,000 – 37,000 ——— 47,000

15,000 – 27,000 ——— 42,000

23,500 ———– 125,000 ═════

6,000 ——— 53,000 ═════

17,000 ——— 59,000 ═════

Statements of profit or loss for the year ended 31 December 2015 Hanks Rs. 000 125,000 (65,000) ———– 60,000 (21,000) (14,000) ———– 25,000 (10,000) ———– 15,000 ═════

Revenue Cost of sales Gross profit Distribution costs Administrative expenses Profit before taxation Income tax expense Profit after tax

Streep Rs. 000 117,000 (64,000) ———– 53,000 (14,000) (8,000) ———– 31,000 (9,000) ———– 22,000 ═════

Scott Rs. 000 82,000 (42,000) ——— 40,000 (16,000) (7,000) ——— 17,000 (5,000) ——— 12,000 ═════

Statement of changes in equity (extract) for the year ending 31 December 2015 Hanks Rs. 000 40,000 15,000 – ———– 55,000 ═════

Retained earnings brought forward Retained profit for the financial year Dividends Retained earnings carried forward

© Emile Woolf International

48

Streep Rs. 000 15,000 22,000 – ———– 37,000 ═════

Scott Rs. 000 15,000 12,000 – ——— 27,000 ═════

The Institute of Chartered Accountants of Pakistan

Questions

You are given the following additional information (1)

Hanks owns 80% of Streep’s shares. These were purchased in 2012 for Rs. 20.5 million cash, when the balance on Streep’s retained earnings stood at Rs. 7million.

(2)

In 2010 Hanks purchased 60% of the shares of Scott by the issue of shares with a nominal value of Rs. 6.5 million. These shares were issued at a premium of Rs. 6.5 million. At that date the retained earnings of Scott stood at Rs. 3 million and the fair value of the net assets of Scott was Rs. 24 million. It was agreed that any undervaluation of the net assets should be attributed to land. This land was still held at 31 December 2015.

(3)

Included in the inventory of Scott and Streep at 31 December 2015 are goods purchased from Hanks for Rs. 5.2 million and Rs. 3.9 million respectively. Hanks aims to earn a profit of 30% on cost. Total sales from Hanks to Scott and to Streep were Rs. 8 million and Rs. 6 million respectively.

(4)

Hanks and Streep each proposed a dividend before the year end of Rs. 2 million and Rs. 2.5 million respectively. No accounting entries have yet been made for these.

(5)

Hanks has carried out annual impairment tests on goodwill in accordance with IFRS 3 and IAS 36. The estimated recoverable amount of goodwill at 31 December 2012 was Rs. 5 million and at 31 December 2015 was Rs. 4.5 million.

Required Prepare the consolidated statement of profit or loss and consolidated statement of changes in equity for the year ended 31 December 2015 and the consolidated statement of financial position at that date. (20)

© Emile Woolf International

49

The Institute of Chartered Accountants of Pakistan

Financial accounting and reporting II

CHAPTER 7 – TANGIBLE NON-CURRENT ASSETS (IAS 16: PROPERTY, PLANT AND EQUIPMENT AND IAS 23: BORROWING COSTS) 7.1

ROONEY (a)

Rooney has recently finished building a new item of plant for its own use. The item is a press for use in the manufacture of industrial diamonds. Rooney commenced construction of the asset on 1st April 2013 and completed it on 1st April 2015. 1st January 2013, Rooney took out a loan to finance the construction of the asset. Interest is charged on the loan at the rate of 5% per annum. The annual interest must be paid in four equal instalments at the end of each quarter. Rooney capitalises interest on manufactured assets in accordance with the rules in IAS 23 Borrowing costs. The costs (excluding finance costs) of manufacturing the asset were Rs. 28 million. Required State the IAS 23 rules on the capitalisation of borrowing costs, calculate the cost of the asset on initial recognition and explain the amount of borrowing cost capitalised. (6)

(b)

The press comprises two significant parts, the hydraulic system and the ‘frame’. The hydraulic system has a three year life and the ‘frame’ has an eight year life. Rooney depreciates plant on a straight line basis. The cost of the hydraulic system is 30% of the total cost of manufacture. Rooney uses the IAS 16 revaluation model in accounting for diamond presses and revalues these assets on an annual basis. Revaluation surpluses or deficits are apportioned between the hydraulic system and the ‘frame’ on the basis of their year end book values before the revaluation. Required Explain the IAS 16 rules on accounting for significant parts of property, plant and equipment and show the accounting treatment of the diamond press in the financial statements for the financial years ending: (i) 31st March 2016 (assume that the press has a fair value of Rs. 21 million) (ii) 31st March 2017 (assume that the press has a fair value of Rs. 19.6 million). (13) (25)

© Emile Woolf International

50

The Institute of Chartered Accountants of Pakistan

Questions

7.2

EHTISHAM The following information relates to the financial statements of Ehtisham for the year to 31 March 2015. The head office of Ehtisham was acquired on 1 April 2012 for Rs. 1million. Ehtisham intend to occupy the building for 25 years. On 31 March 2014 it was revalued to Rs. 1.15 million. On 31 March 2015, a surplus of vacant commercial property in the area had led to a fall in property prices and the fair value was now only Rs. 0.8 million. Required Explain the correct accounting treatment for the above (with calculations if appropriate). (10)

7.3

CARLY The following is an extract from the financial statements of Carly on 31 December 2014. Property, plant and equipment Land and Plant and buildings equipment

Computers

Total

Rs.

Rs.

Rs.

1,500,000

340,500

617,800

2,458,300

600,000

125,900

505,800

1,231,700

900,000

214,600

112,000

1,226,600

Rs. Cost On 31 December 2014 Accumulated depreciation On 31 December 2014 Carrying amount On 31 December 2014

Accounting policies Depreciation Depreciation is provided at the following rates. On land and buildings

2% per annum straight line on buildings only

On plant and equipment

25% reducing balance

On computers

33.33% per annum straight line

During 2015 the following transactions took place. (1)

On 31 December the land and buildings were revalued to Rs. 1,750,000. Of this amount, Rs. 650,000 related to the land (which had originally cost Rs. 500,000). The remaining useful life of the buildings was assessed as 40 years.

(2)

A machine which had cost Rs. 80,000 and had accumulated depreciation of Rs. 57,000 at the start of the year was sold for Rs. 25,000 in the first week of the year.

© Emile Woolf International

51

The Institute of Chartered Accountants of Pakistan

Financial accounting and reporting II

(3)

A new machine was purchased on 31 March 2015. The following costs were incurred: Rs. Purchase price, before discount, inclusive of reclaimable sales tax of Rs. 3,000

20,000

Discount

(4)

1,000

Delivery costs

500

Installation costs

750

Interest on loan taken out to finance the purchase

300

On 1 January it was decided to change the method of providing depreciation on computer equipment from the existing method to 40% reducing balance.

Required Produce the analysis of property, plant and equipment as it would appear in the financial statements of Carly for the year ended 31 December 2015.

7.4

ADJUSTMENTS LIMITED Adjustments Limited has carried out a review of its non-current assets. (a)

A lathe was purchased on 1 January 2009 for Rs. 150,000. The plant had an estimated useful life of twelve years, residual value of nil. Depreciation is charged on the straight line basis. On 1 January 2015, when the asset’s net book value is Rs. 75,000, the directors decide that the asset’s total useful life is only ten years.

(b)

A grinder was purchased on 1 January 2012 for Rs. 100,000. The plant had an estimated useful life of ten years and a residual value of nil. Depreciation is charged on the straight line basis. On 1 January 2015, when the asset’s net book value is Rs. 70,000, the directors decide that it would be more appropriate to depreciate this asset using the sum of digits approach. The remaining useful life is unchanged.

(c)

The company purchased a fifty year lease some years ago for Rs. 1,000,000. This was being depreciated over its life on a straight line basis. On 1 January 2015, when the net book value is Rs. 480,000 and twenty-four years of the lease are remaining, the asset is revalued to Rs. 1,500,000. This revised value is being incorporated into the accounts.

Required Explain the effects of these changes on the depreciation for the year to 31 December 2015.

© Emile Woolf International

52

(15)

The Institute of Chartered Accountants of Pakistan

Questions

7.5

FAM Fam had the following tangible fixed assets at 31 December 2014. Cost Depreciation NBV Rs. 000 Rs. 000 Rs. 000 Land 500 – 500 Buildings 400 80 320 Plant and machinery 1,613 458 1,155 Fixtures and fittings 390 140 250 Assets under construction 91 – 91 ——— —— ——— 2,994 678 2,316 ════ ════ ════ In the year ended 31 December 2015 the following transactions occur. (1) (2) (3) (4) (5)

Further costs of Rs. 53,000 are incurred on buildings being constructed by the company. A building costing Rs. 100,000 is completed during the year. A deposit of Rs. 20,000 is paid for a new computer system which is undelivered at the year end. Additions to plant are Rs. 154,000. Additions to fixtures, excluding the deposit on the new computer system, are Rs. 40,000. The following assets are sold. Cost Rs. 000 277 41

Plant Fixtures (6)

(7) (8)

Depreciation brought forward Rs. 000 195 31

Proceeds Rs. 000 86 2

Land and buildings were revalued at 1 January 2015 to Rs. 1,500,000, of which land is worth Rs. 900,000. The revaluation was performed by Messrs Jackson & Co, Chartered Surveyors, on the basis of existing use value on the open market. The useful economic life of the buildings is unchanged. The buildings were purchased ten years before the revaluation. Depreciation is provided on all assets in use at the year end at the following rates. Buildings 2% per annum straight line Plant 20% per annum straight line Fixtures 25% per annum reducing balance

Required Show the disclosure under IAS 16 in relation to fixed assets in the notes to the published accounts for the year ended 31 December 2015. (14)

© Emile Woolf International

53

The Institute of Chartered Accountants of Pakistan

Financial accounting and reporting II

7.6

IMRAN LIMITED On January 1, 2015, Imran Limited started the construction of its new factory. The construction period is approximately 15 months and the cost is estimated at Rs. 80 million. The work has been divided into 5 phases and payment to contractor shall be made on completion of each phase. In the year the company had the following sources of finance available. (i)

Rights i s s u e o f shares amounting to Rs. 15 million on January 1, 2015. The company usually pays a dividend of 10% each year.

(ii)

Bank loan of Rs. 32 million carrying a mark-up of 13% was raised on March 1, 2015. (This loan was outstanding for 306 days in the year).

(iii)

On August 1, 2015, Rs. 10 million were borrowed from the bank. Interest thereon, is payable at the rate of 11%. (This loan was outstanding for 153 days in the year).

Investment income on temporary investment of the borrowings amounted to Rs. 0.5 million. The details of bills submitted by the contractor, during the year are as follows: Particulars On completion of 1st phase

Date of payment

Rupees

March 1, 2015

20,000,000

nd

April 1, 2015

18,000,000

rd

On completion of 3 phase

October 1, 2015

16,000,000

On completion of 4th phase

Payment not yet made

17,000,000

On completion of 2 phase

On June 1, 2015, the Building Control Authority issued instructions for stoppage of work on account of certain discrepancies in the completion plan. The company filed a petition in the Court and the matter was decided in the company’s favour on July 31, 2015. Work recommenced after a delay of 61 days. The following periods may be relevant: Period

Days

March 1 to December 31

306

April 1 to December 31

275

August 1 to December 31

153

October 1 to December 31

92

Required a)

Assuming that the loans were taken specifically for the project, calculate the amount of borrowing costs that s h o u l d be capitalised i n t h e p e r i o d e n d i n g December 31, 2015 in accordance with the requirements of IAS 23 Borrowing Costs.

b)

Assuming that the loans constituted general finance, calculate the amount of borrowing costs that s h o u l d be capitalised i n t h e p e r i o d e n d i n g December 31, 2015 in accordance with the requirements of IAS 23 Borrowing Costs.

© Emile Woolf International

54

The Institute of Chartered Accountants of Pakistan

Questions

7.7

HUMAYUN CHEMICALS LIMITED (a)

On July 1, 2013, Humayun Chemicals Limited acquired a machine at a cost of Rs. 10 million. The useful life of the machine and its salvage value was estimated at 5 years and Rs. 3.0 million respectively. The cost of machine is being depreciated under the straight line method. Based on the practice followed by similar type of companies, the company has determined that the remaining useful economic life of the machine is six years. It has also been established that the residual value at the end of the useful life will be equal to 10% of the cost of machine. Required Compute the depreciation expenses and other adjustments (if any) required to be made in the financial statements of the company for the year ended June 30, 2015 under each of the following assumptions: (i)

the review of useful life and residual value was carried out on June 30, 2015;

(ii)

the review of useful life and residual value was carried out on June 30, 2014 but in the financial statements for the year then ended the depreciation expense was erroneously recorded on the previous basis. (11)

(b)

7.8

Discuss the requirements of International Accounting Standard(s) in respect of estimation and revision of useful life of an item of property, plant and equipment. (04)

FARADAY PHARMACEUTICAL LIMITED Faraday Pharmaceutical Limited (FPL) acquired a building for Rs. 200 million on July 1, 2011. The following information relating to the building is available: (i)

It is being depreciated on the straight line basis, over 20 years.

(ii)

FPL uses the revaluation model for subsequent measurement of its property, plant and equipment and accounts for revaluations on the net replacement value method. The details of revaluation carried out by the independent valuers during the past years are as follows: Revaluation date July 1, 2012 July 1, 2013 July 1, 2014

Fair value Rupees in million 230 170 180

(iii)

FPL transfers the maximum possible amount from the revaluation surplus to retained earnings on an annual basis.

(iv)

There is no change in the useful life of the building.

Required Prepare the journal entries to record the above transactions from the date of acquisition of the building to the year ended June 30, 2015. (Ignore deferred tax)

© Emile Woolf International

55

(16)

The Institute of Chartered Accountants of Pakistan

Financial accounting and reporting II

7.9

SPIN INDUSTRIES LIMITED On September 1, 2014, Spin Industries Limited (SIL) started construction of its new office building and completed it on May 31, 2015. The payments made to the contractor were as follows: Date of Payment September 1, 2014 December 1, 2014 February 1, 2015 June 1, 2015

Rupees 10,000,000 15,000,000 12,000,000 9,000,000

In addition to the above payments, SIL paid a fee of Rs. 8 million on September 1, 2014 for obtaining a permit allowing the construction of the building. The project was financed through the following sources: (i)

On August 1, 2014 a medium term loan of Rs. 25 million was obtained specifically for the construction of the building. The loan carried mark up of 12% per annum payable semi-annually. A commitment fee @ 0.5% of the amount of loan was charged by the bank. Surplus funds were invested in savings account @ 8% per annum. On February 1, 2015 SIL paid the six monthly interest plus Rs. 5 million towards the principal.

(ii)

Existing running finance facilities of SIL 

Running finance facility of Rs. 28 million from Bank A carrying mark up of 13% payable annually. The average outstanding balance during the period of construction was Rs. 25 million.



Running finance facility of Rs. 25 million from Bank B. The mark up accrued during the period of construction was Rs. 3 million and the average running finance balance during that period was Rs. 20 million.

Required Calculate the amount of borrowing costs to be capitalised on June 30, 2015 in accordance with the requirements of International Accounting Standards. (Borrowing cost calculations should be based on number of months).

7.10

(18)

SCIENTIFIC PHARMA LIMITED Scientific Pharma Limited (SPL) is a manufacturer of pharmaceutical products. In January 2015, one of its plants suffered a major break down. It was repaired at a cost of Rs. 1.5 million but the production capacity was reduced significantly. The plant was ready for production on June 30, 2015. At that time the company’s engineers advised that the plant could be used at a reduced level for 3 years only. The factory was estimated to have a recoverable amount of Rs. 19,277,000 at June 30, 2015 Other related information is as under: (i)

The plant was imported at FOB price of US$ 800,000. The payment was made at the time of shipment on July 1, 2005 at Rs. 52 per US$. Other charges including installation cost amounted to Rs. 7 million. Installation of the plant was completed on December 31, 2005 and commercial production commenced from April 1, 2006.

© Emile Woolf International

56

The Institute of Chartered Accountants of Pakistan

Questions

(ii)

The company uses straight line method of deprecation. Depreciation is charged from the month the asset is available for use upto the month prior to disposal. At the time of purchase, the estimated useful life of the plant was estimated at 15 years whereas the salvage value was estimated at Rs. 2.0 million.

(iii)

Based on the report of a professional independent valuer, the plant was revalued on July 1, 2010 at Rs. 45 million. There was however, no change in estimated useful life of the plant.

(iv)

The factory remained closed from April 1, to June 30, 2012 due to law and order situation.

(v)

The salvage value has not changed since it was first estimated at the time of purchase.

Required Prepare accounting entries for the year ended June 30, 2015. Give all the necessary calculations. (Ignore taxation)

7.11

(20)

QURESHI STEEL LIMITED On July 1, 2014, Qureshi Steel Limited (QSL) signed an agreement with Pak Construction Limited for construction of a factory building at a cost of Rs. 100 million. It was agreed that the factory would be ready for use from January 1, 2016. The terms of payments were agreed as under: (i)

10% advance payment would be made on signing of the agreement. The advance paid would be adjusted at 10% of the quarterly progress bills.

(ii)

5% retention money would also be deducted from the progress bills. Retention money will be refunded one year after completion of the factory building.

(iii)

Progress bills will be raised on last day of each quarter and settled on 15th of the next month.

The under mentioned progress bills were received and settled by QSL as per the agreement: Invoice date September 30, 2014 December 31, 2014 March 31, 2015 June 30, 2015

Amount (Rs. ) 30 million 20 million 10 million 15 million

On April 30, 2015 an invoice of Rs. 1.5 million was raised by the contractor for damages sustained at the site, on account of rains. After negotiations, QSL finally agreed to make additional payment of Rs. 1.0 million to compensate the contractor. The amount was paid on May 15, 2015. It is expected that 75% of the payment would be recovered from the insurance company. The cost of the project has been financed through the following sources: (i)

Issue of right shares amounting to Rs. 15 million, on September 1, 2014. The company has been following a policy of paying dividend of 20% for the past many years.

© Emile Woolf International

57

The Institute of Chartered Accountants of Pakistan

Financial accounting and reporting II

(ii)

Bank loan of Rs. 25 million obtained on December 1, 2014. The loan carries a markup of 13% per annum. The principal is repayable in 5 half yearly equal instalments of Rs. 5 million each along with the interest, commencing from May 31, 2015. Loan processing charges of Rs. 0.5 million were deducted by the bank at the time of disbursement of loan. Surplus funds, when available, were invested in short term deposits at 8% per annum.

(iii)

Cash withdrawals from the existing running finance facility provided by a bank. Average running finance balance for the year was Rs. 60 million. Markup charged by the bank for the year was Rs. 9 million.

Required Compute cost of capital work in progress for the factory building as of June 30, 2015 in accordance with the requirements of relevant IFRSs. (Borrowing costs calculations should be based on number of months)

7.12

(18)

GRANITE CORPORATION On 1 March 2014, Granite Corporation (GC) started the construction of a new plant to meet the growing demand for its products. The new plant was completed at a cost of Rs. 100 million on 31 May 2015. GC financed the cost of the project from the following sources: (i)

On 1 March 2014, a 7-year loan of Rs. 70 million was obtained specifically for the construction of the plant. The loan carried mark-up @ 13% per annum payable semi-annually. An arrangement fee @ 1% of the loan amount was paid to the bank. Two instalments, each comprising of repayment of principal of Rs. 5 million with interest, were paid on 31 August 2014 and 28 February 2015.

(ii)

GC also has a running finance facility of Rs. 100 million carrying mark-up @ 14% per annum. Average utilization of this facility, prior to commencement of construction was Rs. 10 million. Any additional amount required for the project was provided through this facility.

(iii)

Surplus funds were used to reduce the running finance utilization or invested in savings account @ 8% per annum.

Payments made to the contractor were as follows: Payment date 01 March 2014 31 January 2015 30 September 2015

Rs. m 25 65 10

The construction work was suspended from 1 February 2015 to 28 February 2015. The suspension was caused due to delay in shipment of essential components for the installation of the plant. Required Calculate the amount of borrowing costs that may be capitalised during the years ended 30 June 2014 and 2015 in accordance with the requirements of International Financial Reporting Standards. (20)

© Emile Woolf International

58

The Institute of Chartered Accountants of Pakistan

Questions

CHAPTER 8 – IAS 38: INTANGIBLE ASSETS 8.1

FAZAL The following information relates to the financial statements of Fazal for the year to 31 March 2015. The IT division has begun a training course for all managers in a new programming language at a cost of Rs. 200,000. The consultants running the training course have quantified the present value of the training benefits over the next two years to be Rs. 400,000. The project cost has been included in the statement of financial position as a current asset. The accounting policy note identifies that the costs will be written off over the next two years to match the benefits. Required Explain the correct accounting treatment for the above (with calculations if appropriate). (3)

8.2

HENRY During 2015 Henry has the following research and development projects in progress. Project A was completed at the end of 2014. Development expenditure brought forward at the beginning of 2015 was Rs. 412,500 on this project. Savings in production costs arising from this project are first expected to arise in 2015. In 2015 savings are expected to be Rs. 100,000, followed by savings of Rs. 300,000 in 2016 and Rs. 200,000 in 2017. Project B commenced on 1 April 2015. Costs incurred during the year were Rs. 56,000. In addition to these costs a machine was purchased on 1 April 2015 for Rs. 30,000 for use on the project. This machine has a useful life of five years. At the end of 2015 there were still some uncertainties surrounding the completion of the project. Project C had been started in 2014. In 2014 the costs relating to this project of Rs. 36,700 had been written off, as at the end of 2014 there were still some uncertainties surrounding the completion of the project. Those uncertainties have now been resolved and a further Rs. 45,000 costs incurred during the year. Required Show how the above would appear in the financial statements (including notes to the financial statements) of Henry as of 31 December 2015.

© Emile Woolf International

59

The Institute of Chartered Accountants of Pakistan

Financial accounting and reporting II

8.3

TOBY Toby entered into the following transactions during the year ended 31 December 2015. The directors of Toby wish to capitalise all assets wherever possible. (1)

On 1 January Toby acquired the net assets of George for Rs. 105,000. The assets acquired had the following book and fair values.

Goodwill Patents Non-current assets Other sundry net assets

Book value

Fair value

Rs. 5,000 15,000 40,000 30,000

Rs. 5,000 20,000 50,000 25,000

–––––––

––––––––

90,000 100,000 ═════ ═════ The patent expires at the end of 2022. The goodwill arising from the above had a recoverable value at the end of 2015 of Rs. 7,000. (2) (3)

(4)

On 1 April Toby acquired a brand from a competitor for Rs. 50,000. The directors of Toby have assessed the useful life of the brand as five years. During the year Toby spent Rs. 40,000 on developing a new brand name. The development was completed on 30 June. The useful life of this brand has been assessed as eight years. The directors of Toby believe that there is total goodwill of Rs. 2 million within Toby and that this has an indefinite useful life.

Required Prepare the note to the financial statements for intangible assets as at 31 December 2015.

8.4

BROOKLYN Brooklyn is a bio-technology company performing research for pharmaceutical companies. The finance director has contacted your financial consulting company to arrange a meeting to discuss issues relevant to the preparation of the financial statements for the year to 30th June 2015. Your initial telephone conversation has provided the necessary background information. 1

On 1st August 2014 Brooklyn began investigating a new bio-process. On 1st September 2015, the new process was widely supported by the scientific community and the feasibility project was approved. A grant was then obtained relating to future work. Several pharmaceutical companies have expressed an interest in buying the ‘know how’ when the project completes in June 2016. The nominal ledger account set up for the project shows that the expenditure incurred between 1st August 2014 and 30th June 2015 was Rs. 300,000 per month.

2

In August 2015, an employee lodged a legal claim against the company for damage to his health as a result of working for the company for the two years through to 31st March 2014 when he had to retire due to ill health. He has argued that his health deteriorated as a result of the stress from his position in the organisation. Brooklyn has denied the claim and has appointed an employment lawyer to assist with contesting the case. The lawyer has advised that there is a 25% chance that the claim will be rejected, 50% chance that the damages will be Rs. 600,000 and 25% chance of Rs. 1 million. The company has an insurance policy that will pay 10% of any damages to the company.

© Emile Woolf International

60

The Institute of Chartered Accountants of Pakistan

Questions

The lawyer has said that the case could take until 30th June 2018 to resolve. The present value of the estimated damages discounted at 8% is Rs. 476,280 and Rs. 793,800 respectively. 3

Brooklyn owns several buildings, which include an administrative office in the centre of London. The company has revalued these on a regular basis every five years and the next valuation is due on 30th June 2017. Property prices have increased since the last review and particularly for the London premises. The cost of engaging a professionally qualified valuer is very expensive and so to reduce costs the finance director is proposing that the property manager, who is a professionally qualified valuer, should value the London property and that the increase in value should be included in the financial statements. The finance director is of the opinion that the property prices may fall next year.

Required Prepare notes for your meeting with the finance director which explain and justify the accounting treatment of these issues, preparing calculations where appropriate and identifying matters on which your require further information. (25)

8.5

ZOUQ INC Zouq Inc. is a multinational company. As part of its vision to expand its business in South Asia, it purchased a 90% share of a locally incorporated company, Momin Limited. Following are the brief details of the acquisition: January 1, 2014

Date of acquisition Total paid up capital of Momin Limited (Rs. 10 each) Purchase price per share

500,000,000 Rs. 30

Net assets of Momin Limited (as per 2013 audited financial statements) Fair value of net assets (other than intangible assets) of Momin Limited

650,000,000 1,100,000,000

Momin Limited has an established line of products under the brand name of “Badar”. On behalf of Zouq Inc., a firm of specialists has valued the brand name at Rs. 100 million with an estimated useful life of 10 years at January 1, 2014. It is expected that the benefits will be spread equally over the brand’s useful life. An impairment test of goodwill and brand was carried out on December 31, 2014 which indicated an impairment of Rs. 50 million in the value of goodwill. An impairment test carried out on December 31, 2015 indicated a decrease of Rs. 13.5 million in the carrying value of the brand. Required: (a)

What are the requirements of International Accounting Standards relating to amortization of intangible assets having finite life?

(b)

Prepare the ledger accounts for goodwill and the brand, showing initial recognition and all subsequent adjustments.

© Emile Woolf International

61

(15)

The Institute of Chartered Accountants of Pakistan

Financial accounting and reporting II

8.6

STAR-BRIGHT PHARMACEUTICAL LIMITED Star-Bright Pharmaceutical Limited (SPL), a listed company, purchased a brand on January 1, 2010 at a cost of Rs. 382 million. It has incurred a substantial amount on further development of the brand, in subsequent years. It is the policy of SPL to amortise the development expenditures which meet the recognition criteria as given in IAS-38 ‘Intangible Assets’, over a period of ten years. The amortization commences when the development expenditures first meet the recognition criteria. However, it was discovered during the year 2015 that the development expenditure incurred after acquisition had erroneously been written-off to the profit and loss account, details of which are as follows: Year ended

Rs. m

December 31, 2012

24

December 31, 2013

54

December 31, 2014

38

December 31, 2015

43

The draft financial statements (before correction of error) show that retained earnings as at December 31, 2015 was Rs. 1,950 million (2014: Rs. 1,785 million). Required In accordance with the requirements of International Financial Reporting Standards, prepare relevant extracts of the Statement of Financial Position along with the note on intangible assets after incorporating the required corrections. (Ignore tax) (16)

8.7

RAISIN INTERNATIONAL (a)

Discuss the criteria that should be used while recognizing intangible assets arising from research and development work. (05 marks)

(b)

Raisin International (RI) is planning to expand its line of products. The related information for the year ended 31 December 2015 is as follows: (i)

Research and development of a new product commenced on 1 January 2015. On 1 October 2015, the recognition criteria for capitalization of an internally generated intangible asset were met. It is estimated that the product would have a useful life of 7 years. Details of expenditures incurred are as follows: Rs. m Research work

4.50

Development work

9.00

Training of production staff

0.50

Cost of trial run

0.80

Total costs

© Emile Woolf International

14.80

62

The Institute of Chartered Accountants of Pakistan

Questions

(ii)

The right to manufacture a well-established product under a patent for a period of five years was purchased on 1 March 2015 for Rs. 17 million. The patent has an expected remaining useful life of 10 years. RI has the option to renew the patent for a further period of five years for a sum of Rs. 12 million.

(iii)

RI has acquired a brand at a cost of Rs. 2 million. The cost was incurred in the month of June 2015. The life of the brand is expected to be 10 years. Currently, there is no active market for this brand. However, RI is planning to launch an aggressive marketing campaign in February 2016.

(iv)

In September 2014, RI developed a new production process and capitalised it as an intangible asset at Rs. 7 million. The new process is expected to have an indefinite useful life. During 2015, RI incurred further development expenditure of Rs. 3 million on the new process which meets the recognition criteria for capitalization of an intangible asset.

Required In the light of International Financial Reporting Standards, explain how each of the above transaction should be accounted for in the financial statements of Raisin International for the year ended 31 December 2015. (11)

© Emile Woolf International

63

The Institute of Chartered Accountants of Pakistan

Financial accounting and reporting II

CHAPTER 9 – IAS 17: LEASES 9.1

DAWOOD The following information relates to the financial statements of Dawood for the year to 31 March 2015. On 1 October 2014, Dawood entered into a 5 year lease for a machine from Narbonne, agreeing to make payments every 6 months of Rs. 29,500 beginning on the 1 October. The cash price of the machine is Rs. 250,000 and the machine is believed to have a useful life of 5 years. Dawood has treated the arrangement as a finance lease. Any finance costs are to be treated using the sum-of-digits method. Required Explain the correct accounting treatment for the above (with calculations if appropriate). (7)

9.2

FINLEY On 1 January 2015, Finley entered into an agreement to lease a boat. The fair value of the boat was Rs. 36,000 and the term of the lease was four years. Annual lease payments of Rs. 10,000 are payable in advance. The interest rate implicit in the lease is 7.5%. Finley is responsible for insuring and maintaining the boat throughout the term of the lease. Required Show how this lease would be presented in the statement of profit or loss of Finley for the year ended 31 December 2015 and the statement of financial position as at that date. Detailed disclosure notes are not required.

9.3

FABIAN In the year ended 31 December 2015, Fabian leased two assets. (1)

A car was leased on 1 July 2015 via a three year lease agreement. Fabian paid a deposit of Rs. 7,500 followed by 36 monthly payments of Rs. 700 each on the 1st of each month. At the end of the three years Fabian will return the car. The car has a useful life of eight years.

(2)

A machine was leased on 1 January 2015 via a four year lease. The machine has a fair value of Rs. 130,000 and Fabian is responsible for its upkeep. Lease payments of Rs. 40,000 are payable in arrears annually. The interest rate implicit in the lease is 10% and the present value of the minimum lease payments is Rs. 126,760.

Required Show how the two lease agreements would be presented in the statement of profit or loss for 2015 and the statement of financial position at 31 December 2015. Notes to the financial statements are not required.

© Emile Woolf International

64

The Institute of Chartered Accountants of Pakistan

Questions

9.4

XYZ INC A lessor, ABC Inc, leases an asset, which it purchased for Rs. 4,400,000, to XYZ Inc under a finance lease. It estimates that its residual value after five years will be Rs. 400,000 and after seven years will be zero. The lease is for five years at a rental of Rs. 600,000 per half year in advance, with an option of two more years at nominal rental. The lease commences on 1 January 2015. The directors of XYZ Inc consider that the asset has a useful life of seven years. The finance charge is to be allocated using the sum of digits (“rule of 78”) method. Title to the asset will pass to XYZ at the end of seven years if the option is exercised. It is likely that it will be. Required (a) (b)

Show the relevant extracts from the accounts of XYZ Inc at 31 December 2015.

(5)

Show the allocation of the finance charge for XYZ Inc using the actuarial before tax method (using the interest rate implicit in the lease). Compare this with the sum of the digits allocation in (a) above. (14)

The rate of interest implicit in the lease is 7.68% per half year.

9.5

SNOW INC On 1 January 2015, Snow Inc entered into the following finance lease agreements. (a)

Snowplough To lease a snowplough for 3 years from Ice Inc. The machine had cost Ice Inc Rs. 35,000,000. A deposit of Rs. 2,000,000 was payable on 1 January 2015 followed by 6 half yearly instalments of Rs. 6,500,000 payable in arrears, commencing on 30 June 2015. Finance charges are to be allocated on a sum of digits basis.

(b)

Snow machine To lease a snow machine for 5 years from Slush Inc. The snow machine cost Slush Inc Rs. 150,000 and is estimated to have a useful life of 5 years. Snow Inc has agreed to make 5 annual instalments of Rs. 35,000,000, payable in advance, commencing on 1 January 2015. The interest rate implicit in the lease is 8.36%.

Required Show the relevant extracts from the accounts of Snow Inc for year ended 31 December 2015.

© Emile Woolf International

65

(15)

The Institute of Chartered Accountants of Pakistan

Financial accounting and reporting II

9.6

MIRACLE TEXTILE LIMITED On 1 July 2013, Miracle Textile Limited (MTL) acquired a machine on lease, from a bank. Details of the lease are as follows: (i)

Cost of machine is Rs. 20 million.

(ii)

The lease term and useful life is 4 years and 10 years respectively.

(iii)

Instalment of Rs. 5.80 million is to be paid annually in advance on 1 July.

(iv)

The interest rate implicit in the lease is 15.725879%.

(v)

At the end of lease term, MTL has an option to purchase the machine on payment of Rs. 2 million. The fair value of the machine at the end of lease term is expected to be Rs. 3 million.

MTL depreciates the machine on the straight line method to a nil residual value. Required Prepare relevant extracts of the statement of financial position and related notes to the financial statements for the year ended 30 June 2015 along with comparative figures. Ignore taxation (16)

9.7

SHOAIB LEASING LIMITED Shoaib Leasing Limited (the lessor) has entered into a three year agreement with Sarfaraz Limited (the lessee) to lease a machine with an expected useful life of 4 years. The cost of machine is Rs. 2,100,000. The following information relating to lease transaction is available: (i)

Date of commencement of lease is July 1, 2015.

(ii)

The lease contains a purchase bargain option at Rs. 100,000. At the end of the lease term, the value of the machine will be Rs. 300,000.

(iii)

Lease instalments of Rs. 860,000 are payable annually, in arrears, on June 30.

(iv)

The implicit interest rate is 12.9972%.

Required (a)

Prepare the journal entries for the years ending June 30, 2016, 2017 and 2018 in the books of lessor. Ignore tax.

(b)

Produce extracts from the statement of financial position including relevant notes as at June 30, 2016 to show how the transactions carried out in 2016 would be reflected in the financial statements of the lessor.

(Disclosure of accounting policy is not required.)

© Emile Woolf International

66

(20)

The Institute of Chartered Accountants of Pakistan

Questions

9.8

NEPTUNE LIMITED Neptune Limited (NL) had established its business in December 2014 as a supplier of plant and machinery. During the year ended December 31, 2015 the company sold two machines under lease arrangements. The details are as under: A January 1, 2015 6 years Rs. 2,000,000

Date of commencement of lease Lease period Lease instalments payable annually in advance

B January 1, 2015 3 years Rs. 4,000,000 (to be reduced annually by 5%)

Cost of machine Economic life

Rs. 6,963,448 6 years

Rs. 15,000,000 6 years

NL sells machines on cash at cost plus 25%. It depreciates its assets under straight line method with no residual value. Fair market annual interest rate is 15%. Required (a) (b)

Prepare journal entries to record the above transactions. Prepare notes to the financial statements for the year ended December 31, 2015 in accordance with the requirements of IAS - 17 (Leases). (19) (Ignore taxation and comparative figures)

9.9

QUARTZ AUTO LIMITED Quartz Auto Limited (QAL) is engaged in the business of manufacturing of trucks. Since a number of the prospective customers do not have adequate funds to purchase the vehicles against full payment, QAL provides lease financing facility to its customers. It expects to receive a return at the rate of 15% per annum on the amount of lease finance. On 1 July 2014, QAL sold seven trucks to Emerald Goods Transport Company (EGTC) on lease. The terms of the lease and related information are as follows: (i) (ii)

(iii) (iv) (v)

The lease period is 4 years, extendable up to the expected useful life of the trucks i.e. 5 years. EGTC has guaranteed a residual value of Rs. 360,000 for each truck, till the end of the fourth year. However, the guarantee would lapse if the lease term is extended to the fifth year. EGTC will return the truck at the end of the lease term. Lease rentals amount to Rs. 2,715,224 per annum and are payable in arrears i.e. on 30 June. The cost of each truck is Rs. 900,000. Price in case of outright sale is Rs. 1,350,000 per truck. The expected residual value of each truck at the end of the 4th and 5th year is Rs. 150,000 and Rs. 100,000 respectively.

Required Assuming that QAL and EGTC intend to extend the lease for a period of five years, prepare: (a) Journal entries to record the transactions for the year ended 30 June 2015. (08) (b) A note for inclusion in the financial statements, for the year ended 30 June 2015, in accordance with the requirements of IAS 17 ‘Leases’. (07)

© Emile Woolf International

67

The Institute of Chartered Accountants of Pakistan

Financial accounting and reporting II

9.10

LODHI TEXTILE MILLS LIMITED Lodhi Textile Mills Limited is facing severe financial difficulties. To improve the cash flows, the management has decided to sell and lease back three power generators of the company under three different sale and lease back arrangements which were signed on August 15, 2015. The company has assessed that all the leases shall qualify as finance leases. The related information as on August 15, 2015 is given below: Cost

Book Value

Fair Value Value in Amount of Use Financing

Rs. 000

Rs. 000

Rs. 000

Rs. 000

Rs. 000

Generator A

10,000

7,500

6,000

6,500

6,000

Generator B

12,000

6,000

4,000

5,000

6,000

Generator C

10,000

7,000

10,000

12,000

8,000

Required Prepare the accounting entries that should be recorded by the company on August 15, 2015 in respect of the above transactions.

(13)

Note: Ignore tax and deferred tax implications, if any.

9.11

NOMAN ENGINEERING LIMITED Noman Engineering Limited (NEL) manufactures auto parts. On July 1, 2014 it finalised a lease agreement with a bank for sale and leaseback of one of its plants costing Rs. 18.75 million. Relevant information is as under: (i)

Proceeds from the bank amounting to Rs. 20 million which represent the prevailing market value of such type and age of plant, were received on July 1, 2014.

(ii)

The plant had a book value of Rs. 15 million at the time of commencement of the lease.

(iii)

The remaining life of the plant on July 1, 2014 was estimated at 8 years.

(iv)

The lease period is 6 years. Lease instalments of Rs. 2.5 million each are payable semi-annually in arrears from December 31, 2014.

(v)

NEL has the option to purchase the plant at market value at the end of the lease term. No final decision has yet been made by NEL, in this regard.

(vi)

The rate of interest implicit in the lease is 13.731% per annum.

Required Pass journal entries in respect of the lease, for the year ended June 30, 2015.

© Emile Woolf International

68

(12)

The Institute of Chartered Accountants of Pakistan

Questions

CHAPTER 10 – IAS 37: PROVISIONS CONTINGENT LIABILITIES AND CONTINGENT ASSETS AND IAS 10: EVENTS OCCURRING AFTER THE REPORTING DATE 10.1

BADAR The following information relates to the financial statements of Badar for the year to 31 March 2015. The mining division of Badar has a 3 year operating licence from an overseas government. This allows it to mine and extract copper from a particular site. When the licence began on 1 April 2014, Badar started to build on the site. The cost of the construction was Rs. 500,000. The overseas country has no particular environmental decommissioning laws. In its past financial statements Badar has given information about the company’s environmental policy and has provided examples to demonstrate that it is a responsible company that believes in restoring mining sites at the end of the extraction period. The cost of removing the construction at the end of the three years is estimated to be Rs. 100,000. The cost of the site currently shown in the trial balance is Rs. 500,000. The company has a cost of borrowing of 10%. Required Explain the correct accounting treatment for the above (with calculations if appropriate). (6)

10.2

GEORGINA Georgina Company is preparing its financial statements for the year ended 30 September 2015. The following matters are all outstanding at the year end. (1) Georgina is facing litigation for damages from a customer for the supply of faulty goods on 1 September 2015. The claim, which is for Rs. 500,000, was received on 15 October 2015. Georgina’s legal advisors consider that Georgina is liable and that it is likely that this claim will succeed. On 25 October 2015 Georgina sent a counter-claim to its suppliers for Rs. 400,000. Georgina’s legal advisors are unsure whether or not this claim will succeed. (2) Georgina’s sales director, who was dismissed on 15 September, has lodged a claim for Rs. 100,000 for unfair dismissal. Georgina’s legal advisors believe that there is no case to answer and therefore think it is unlikely that this claim will succeed. (3) Although Georgina has no legal obligation to do so, it has habitually operated a policy of allowing customers to return goods within 28 days, even where those goods are not faulty. Georgina estimates that such returns usually amount to 1% of sales. Sales in September 2015 were Rs. 400,000. By the end of October 2015, prior to the drafting of the financial statements, goods sold in September for Rs. 3,500 had been returned. (4) On 15 September 2015 Georgina announced in the press that it is to close one of its divisions in January 2016. A detailed closure plan is in place and the costs of closure are reliably estimated at Rs. 300,000, including Rs. 50,000 for staff relocation. Required State, with reasons, how the above should be treated in Georgina’s financial statements for the year ended 30 September 2015.

© Emile Woolf International

69

The Institute of Chartered Accountants of Pakistan

Financial accounting and reporting II

10.3

EARLEY INC Earley Inc is finalising its accounts for the year ended 31 December 2014. The following events have arisen since the year end and the financial director has asked you to comment on the final accounts. (a)

At 31 December 2014 trade receivables included a figure of Rs. 250,000 in respect of Nedengy Inc. On 8 March 2015, when the current debt was Rs. 200,000, Nedengy Inc went into receivership. Recent correspondence with the receiver indicates that no dividend will be paid to unsecured creditors.

(b)

On 15 March 2015 Earley Inc sold its former head office building, Whitley Wood, for Rs. 2.7 million. At the year end the building was unoccupied and carried at a value of Rs. 3.1 million.

(c)

Inventories at the year-end included Rs. 650,000 of a new electric tricycle, the Opasney. In January 2015 the European Union declared the tricycle to be unsafe and prohibited it from sale. An alternative market, in Bongolia, is being investigated, although the current price is expected to be cost less 30%.

(d)

Stingy Inc, a subsidiary in Outer Sonning, was nationalised in February 2015. The Outer Sonning authorities have refused to pay any compensation. The net assets of Stingy Inc have been valued at Rs. 200,000 at the year end.

(e)

Freak floods caused Rs. 150,000 damage to the Southcote branch of Earley Inc in January 2015. The branch was fully insured.

(f)

On 1 April 2015 Earley Inc announced a 1 for 1 rights issue aiming to raise Rs. 15 million.

Required Explain how you would respond to the matters listed above.

10.4

(13)

ACCOUNTING TREATMENT You have been asked to advise on the appropriate accounting treatment for the following situations arising in the books of various companies. The year end in each case can be taken as 31 December 2015 and you should assume that the amounts involved are material in each case. (a)

At the year end there was a debit balance in the books of a company for Rs. 15,000, representing an estimate of the amount receivable from an insurance company for an accident claim. In February 2016, before the directors had agreed the final draft of the published accounts, correspondence with lawyers indicated that Rs. 18,600 might be payable on certain conditions.

(b)

A company has an item of equipment which cost Rs. 400,000 in 2012 and was expected to last for ten years. At the beginning of the 2015 financial year the book value was Rs. 280,000. It is now thought that the company will soon cease to make the product for which the equipment was specifically purchased. Its recoverable amount is only Rs. 80,000 at 31 December 2015.

(c)

On 30 November a company entered into a legal action defending a claim for supplying faulty machinery. The company’s solicitors advise that there is a 20% probability that the claim will succeed. The amount of the claim is Rs. 500,000.

© Emile Woolf International

70

The Institute of Chartered Accountants of Pakistan

Questions

(d)

An item has been produced at a manufacturing cost of Rs. 1,800 against a customer’s order at an agreed price of Rs. 2,300. The item was in inventory at the year-end awaiting delivery instructions. In January 2016 the customer was declared bankrupt and the most reasonable course of action seems to be to make a modification to the unit, costing approximately Rs. 300, which is expected to make it marketable with other customers at a price of about Rs. 1,900.

(e)

At 31 December a company has a total potential liability of Rs. 1,000,400 for warranty work on contracts. Past experience shows that 10% of these costs are likely to be incurred, that 30% may be incurred but that the remaining 60% is highly unlikely to be incurred.

Required For each of the above situations outline the accounting treatment you would recommend and give the reasoning of principles involved. The accounting treatment should refer to entries in the books and/or the year-end financial statements as appropriate. (12)

10.5

J-MART LIMITED (a)

Explain the terms “adjusting events” and “non-adjusting events” and give three examples of each. (05)

(b)

J-Mart Limited, a chain of departmental stores has distributed its operations into four Divisions i.e. Food, Furniture, Clothing and Household Appliances. The following information has been extracted from the records: (i)

The company allows the dissatisfied customers to return the goods within 30 days. It is estimated that 5% of the sales made in June 2015 will be refunded in July 2015.

(ii)

On June 2, 2015, three employees were seriously injured as a result of a fire at the company’s warehouse. They have lodged claims seeking damages of Rs. 2.0 million from the company. The company’s lawyers have advised that it is probable that the court may award compensation of Rs. 400,000.

(iii)

Under a new legislation, the company is required to fit smoke detectors at all the stores by December 31, 2015. The company has not yet installed the smoke detectors.

(iv)

On June 20, 2015, the board of directors decided to close down the Household Appliances Division. However, the decision was made public after June 30, 2015.

(v)

The company has a large warehouse in Lahore which was acquired under a three-year rent agreement signed on April 1, 2014. The agreement is non- cancellable and the company cannot sub-let the warehouse. However, due to operational difficulties, the company shifted the warehouse to a new location.

(vi)

A 15% cash dividend was declared on July 5, 2015.

Required Describe how each of the above issue should be dealt with in the financial statements for the year ended June 30, 2010. Support your point of view in the light of relevant International Accounting Standards. (15)

© Emile Woolf International

71

The Institute of Chartered Accountants of Pakistan

Financial accounting and reporting II

10.6

AKBER CHEMICALS LIMITED Akber Chemicals Limited is engaged in the business of manufacture and sale of different type of chemicals. The following transactions have not yet been incorporated in the financial statements for the year ended June 30, 2015: (a)

On June 15, 2015, one of its tankers carrying chemicals fell into a canal, thus polluting the water. The company has never faced such a situation before. The company has neither any legal obligation to clean the canal nor does it have any published environmental policy. In a meeting held on July 26, 2015 the Board of Directors decided to clean the canal, which is estimated to cost Rs. 5.5 million.

(b)

During the second week of July 2015, a significant decline in the demand for company’s products was observed which also led to a decrease in net realizable value of finished goods. It was estimated that goods costing Rs. 25 million as at June 30, 2015 would only fetch Rs. 23 million.

(c)

On June 21, 2015, a customer lodged a claim of Rs. 2 million with the company as a consignment dispatched on June 1, 2015 was not according to the agreed specifications. The company’s inspection team found that this defect arose because of inferior quality of raw materials supplied by the vendor. On June 28, 2015, the company lodged a claim for damages of Rs. 5.0 million, with its vendor, which include reimbursement of the cost of raw materials. The company anticipates that it will have to pay compensation to its customer and would be able to recover 50% of the amount claimed from the vendor.

Required Discuss how Akber Chemicals (Pvt.) Limited would deal with the above situations in its financial statements for the year ended June 30, 2015. Explain your point of view with reference to the guidance contained in the International Financial Reporting Standards. (13)

10.7

QALLAT INDUSTRIES LIMITED The following information pertains to Qallat Industries Limited (QIL) for its financial year ended June 30, 2015: (i)

QIL sells all its products on one-year warranty which covers all types of defects. Previous history indicates that 2% of the products contain major defects whereas 10% have minor defects. It is estimated that if major defects were detected in all the products sold, repair cost of Rs. 150 million would result. If minor defects were detected in all products sold, repair cost of Rs. 70 million would result. Total sales for the year are amounted to Rs. 830 million.

(ii)

QIL has two large warehouses, A and B. These were acquired under noncancellable lease agreements. Details are as follows:

Effective date of agreement Lease period Rental amount per month

© Emile Woolf International

72

Warehouse A

Warehouse B

July 1, 2010 10 years Rs. 450,000

January 1, 2013 8 years Rs. 300,000

The Institute of Chartered Accountants of Pakistan

Questions

On account of serious operating difficulties, QIL vacated both the warehouses on January 1, 2015 and moved to a warehouse situated close to its factory. On the same day QIL sub-let Warehouse A at Rs. 250,000 per month for the remaining lease period. Warehouse B was sub-let on March 1, 2015 for Rs. 350,000 per month for the remaining lease period. (iii)

On July 18, 2015, QIL was sued by an employee claiming damages for Rs. 6 million on account of an injury caused to him due to alleged violation of safety regulations on the part of the company, while he was working on the machine on June 15, 2015. Before filing the suit, he contacted the management on June 29, 2015 and asked for compensation of Rs. 4 million which was turned down by the management. The lawyer of the company anticipates that the court may award compensation ranging between Rs. 1.5 million to Rs. 3 million. However, in his view the most probable amount is Rs. 2 million.

(iv)

On November 1, 2014 a new law was introduced requiring all factories to install specialised safety equipment within four months. The Equipment costing Rs. 5.0 million was ordered on December 15, 2014 against 100% advance payment but the supplier delayed installation to July 31, 2015. On August 5, 2015 the company received a notice from the authorities levying a penalty of Rs. 0.4 million i.e. Rs. 0.1 million for each month during which the violation continued. QIL has lodged a claim for recovery of the penalty from the supplier of the equipment.

Required Describe how each of the above issues should be dealt with in the financial statements for the year ended June 30, 2015. Support your answer in the light of relevant International Accounting Standards and quantify the effect where possible. (14)

10.8

SKYLINE LIMITED The following information pertains to Skyline Limited (SL) for the financial year ended December 31, 2015: (i)

A customer who owed Rs. 1 million was declared bankrupt after his warehouse was destroyed by fire on February 10, 2016. It is expected that the customer would be able to recover 50% of the loss from the insurance company. (ii) An employee of SL forged the signatures of directors and made cash withdrawals of Rs. 7.5 million from the bank. Of these, Rs. 1.5 million were withdrawn before December 31, 2015. Investigations revealed that an employee of the bank was also involved and therefore, under a settlement arrangement, the bank paid 60% of the amount to SL on January 27, 2016. (iii) SL has filed a claim against one of its vendors for supplying defective goods. SL’s legal consultant is confident that damages of Rs. 1 million would be paid to SL. The supplier has already reimbursed the actual cost of the defective goods. (iv) A suit for infringement of patents, seeking damages of Rs. 2 million, was filed by a third party. SL’s legal consultant is of the opinion that an unfavourable outcome is most likely. On the basis of past experience he has advised that there is 60% probability that the amount of damages would be Rs. 1 million and 40% likelihood that the amount would be Rs. 1.5 million. Required Advise SL about the amount of provision that should be incorporated and the disclosures that are required to be made in the financial statements for the year ended December 31, 2015. (16)

© Emile Woolf International

73

The Institute of Chartered Accountants of Pakistan

Financial accounting and reporting II

10.9

WALNUT LIMITED Walnut Limited (WL) is engaged in the business of import and distribution of electronic appliances. The following events took place subsequent to the reporting period i.e. 31 December 2015: (i)

On 15 January 2016, one of WL’s competitors announced launching of an upgraded version of DVD players. WL’s inventories include a large stock of existing version of DVD players which are valued at Rs. 15 million. Because of the introduction of the upgraded version, the net realizable value of the existing version in WL’s inventory at 31 December 2015 has reduced to Rs. 12.5 million.

(ii)

On 20 December 2015, the board of directors decided to close down the division which imports and sells mobile sets. This decision was made public on 29 December 2015. However, the business was actually closed on 29 February 2016. Net costs incurred in connection with the closure of this division were as follows: Redundancy costs Staff training Operating loss from 1 July 2015 to closure of division Less: Profit on sale of remaining mobile sets

Rs. m 1.50 0.15 0.80 (0.50) 1.95

(iii)

On 16 January 2016, LED TV sets valuing Rs. 3 million were stolen from a warehouse. These sets were included in WL’s inventory as at 31 December 2015.

(iv)

WL owns 9,000 shares of a listed company whose price as on 31 December 2015 was Rs. 22 per share. During February 2016, the share price declined significantly after the government announced a new legislation which would adversely affect the company’s operations. No provision in this regard has been made in the draft financial statements.

(v)

On 31 January 2016, a customer announced voluntary liquidation. On 31 December 2015, this customer owed Rs. 1.5 million.

(vi)

On 15 February 2016, WL announced final dividend for the year ended 31 December 2015 comprising 20% cash dividend and 10% bonus shares, for its ordinary shareholders.

Required Describe how each of the above transactions should be accounted for in the financial statements of Walnut Limited for the year ended 31 December 2015. Support your answer in the light of relevant International Financial Reporting Standards. (16)

© Emile Woolf International

74

The Institute of Chartered Accountants of Pakistan

Questions

10.10 ATTOCK TECHNOLOGIES LIMITED Attock Technologies Limited (ATL) manufactures five hi-tech products, each on a different plant. It is in the process of preparing its financial statements for the year ended June 30, 2015. As the CFO of the company, the following matters are under your consideration: (i)

Inventory carried at Rs. 25 million on June 30, 2015 was sold for Rs. 15 million after it had been damaged in a flood, in July 2015.

(ii)

On July 5, 2015 one of ATL’s corporate customers declared bankruptcy. The liquidator announced on August 25, 2015 that 20% of the debt would be paid on liquidation.

(iii)

A new product introduced by a competitor on August 1, 2015 had caused a significant decline in the market demand of one of ATL’s major products. As a result, ATL is considering a reduction in price and a cut in production.

(iv)

On August 18, 2015 the government announced a retrospective increase in the tax rate applicable to the company.

(v)

The directors of ATL declared a dividend of Rs. 3 per share on August 28, 2015.

Required State how the above events should be treated in ATL’s financial statements for the year ended June 30, 2015. You may assume that all the above events are material to the company. (11)

© Emile Woolf International

75

The Institute of Chartered Accountants of Pakistan

Financial accounting and reporting II

CHAPTER 11 – IAS 8: ACCOUNTING POLICIES, CHANGES IN ACCOUNTING ESTIMATES AND ERRORS 11.1

WONDER LIMITED Wonder Limited (WL) is engaged in the manufacturing and sale of textile machinery. Following are the draft extracts of the statement of financial position and the statement of profit or loss for the year ended 30 June 2015: Statement of Financial Position 2015

2014

Rs. m

Rs. m

Property, plant and equipment

189

130

Retained earnings

166

108

45

27

2015

2014

Rs. m 90 32 58

Rs. m 120 42 78

Deferred tax liability Statement of profit or loss

Profit before taxation Taxation Profit after taxation

Following additional information has not been taken into account in the preparation of the above financial statements: (i)

Cost of repairs amounting to Rs. 20 million was erroneously debited to the machinery account on 1 October 2013. The estimated useful life of the machine is 10 years.

(ii)

On 1 July 2014, WL reviewed the estimated useful life of its plant and revised it from 5 years to 8 years. The plant was purchased on 1 July 2013 at a cost of Rs. 70 million.

Depreciation is provided under the straight line method. Applicable tax rate is 30%. Required Prepare relevant extracts (including comparative figures) for the year ended 30 June 2015 related to the following: (a)

Statement of financial position

(b)

Statement of profit or loss

(c)

Statement of changes in equity

(d)

Correction of error note

© Emile Woolf International

(20)

76

The Institute of Chartered Accountants of Pakistan

Questions

11.2

DUNCAN Duncan Company has previously written off any expenditure on borrowing costs in the period in which it was incurred. The company has appointed new auditors this year. They have expressed the view that the previous recognition of borrowing costs in the statement of profit or loss was in error. The company has decided to correct the error retrospectively in accordance with IAS 8. The financial statements for 2014 and the 2015 draft financial statements, both reflecting the old policy, show the following. Statement of changes in equity (extract)

Opening balance Profit after tax for the period Dividends paid

2014 Retained earnings Rs. 000 22,500 3,200 (1,750)

2015 Retained earnings Rs. 000 23,950 4,712 (2,500)

23,950 ═════

26,162 ═════

–––––––

Closing balance

–––––––

Borrowing costs written off were Rs. 500,000 in 2014 and Rs. 600,000 in 2015. The directors have calculated that borrowing costs, net of depreciation which should have been included in property, plant and equipment had the correct policy been applied, are as follows. Rs. 000 At 30 December 2013

400

At 31 December 2014

450

At 31 December 2015

180

Had the correct policy been in force depreciation of Rs. 450,000 would have been charged in 2014 and Rs. 870,000 in 2015. Required Show how the change in accounting policy must be reflected in the statement of changes in equity for the year ended 31 December 2015. Work to the nearest Rs. 000.

© Emile Woolf International

77

The Institute of Chartered Accountants of Pakistan

Financial accounting and reporting II

11.3

MOHANI MANUFACTURING LIMITED Mohani Manufacturing Limited is engaged in manufacturing of spare parts for motor car assemblers. The audited financial statements for the year ended December 31, 2014 disclosed that the profit and retained earnings were Rs. 21 million and Rs. 89 million respectively. The draft financial statements for the year show a profit of Rs. 15 million. However, following adjustments are required to be made: (i)

The management of the company has decided to change the method for valuation of raw materials from FIFO to weighted average. The value of inventory under each method is as follows: FIFO Rs. m 37.0 42.3 58.4

December 31, 2013 December 31, 2014 December 31, 2015 (ii)

Weighted Average Rs. m 35.5 44.5 54.4

In 2014, the company purchased a plant for Rs. 100 million. Depreciation on plant was recorded at Rs. 25 million instead of Rs. 10 million. This error was discovered after the publication of financial statements for the year ended December 31, 2014. The error is considered to be material.

Required Produce an extract showing the movement in retained earnings, as would appear in the statement of changes in equity for the year ended December 31, 2015. (11)

© Emile Woolf International

78

The Institute of Chartered Accountants of Pakistan

Questions

CHAPTER 12 – IAS 12: INCOME TAXES 12.1

FRANCESCA On 30 June 2014 Francesca Company had a credit balance on its deferred tax account of Rs. 1,340,600 all in respect of the difference between depreciation and capital allowances. During the year ended 30 June 2015 the following transactions took place. (1)

Rs. 45 million was charged against profit in respect of depreciation. The tax computation showed capital allowances of Rs. 50 million.

(2)

Interest receivable of Rs. 50,000 was reflected in profit for the period. However, only Rs. 45,000 of interest was actually received during the year. Interest is not taxed until it is received.

(3)

Interest payable of Rs. 32,000 was treated as an expense for the period. However, only Rs. 28,000 of interest was actually paid during the year. Interest is not an allowable expense for tax purposes until it is paid.

(4)

During the year Francesca incurred development costs of Rs. 500,600, which it has capitalised. Development costs are an allowable expense for tax purposes in the period in which they are paid.

(5)

Land and buildings with a net book value of Rs. 4,900,500 were revalued to Rs. 6 million.

The tax rate is 30%. Francesca has a right of offset between its deferred tax liabilities and its deferred tax assets. Required Calculate the deferred tax liability on 30 June 2015. Show where the increase or decrease in the liability in the year would be charged or credited.

12.2

SHEP (I) Shep was incorporated on 1 January 2015. In the year ended 31 December 2015 the company made a profit before taxation of Rs. 121,000 During the period Shep made the following capital additions. Rs. Plant Motor vehicles

48,000 12,000

During the period: Accounting depreciation Tax depreciation Tax is chargeable at a rate of 30%.

11,000 15,000

Required (a) (b) (c) (d)

Calculate the corporate income tax liability for the year ended 31st December 2015. Calculate the deferred tax balance that is required in the statement of financial position as at 31st December 2015. Prepare a note showing the movement on the deferred tax account and thus calculate the deferred tax charge for the year ended 31st December 2015 Prepare the statement of profit or loss note which shows the compilation of the tax expense for the year ended 31st December 2015.

© Emile Woolf International

79

The Institute of Chartered Accountants of Pakistan

Financial accounting and reporting II

12.3

SHEP (II) Continuing from the previous year. The following information is relevant for the year ended 31st December 2016. (a)

Capital transactions Rs. Depreciation charged Tax allowances

(b)

14,000 16,000

Interest payable On 1st April 2016 the company issued Rs. 25,000 of 8% convertible loan stock. Interest is paid in arrears on 30th September and 30th March. Assume that tax relief on interest expense is only given when the interest is paid.

(c)

Interest receivable On 1st April Shep purchased debentures having a nominal value of Rs. 4,000. Interest at 15% pa is receivable on 30th September and 30th March. Assume that interest income is not taxed until the cash is actually received.

(d)

Provision for warranty In preparing the financial statements for the year to 31st December 2016, Shep has recognised a provision for warranty payments in the amount of Rs. 1,200. This has been correctly recognised in accordance with IAS 37 and the amount has been expensed. Assume that tax relief on the warranty cost is only given when the expense is paid.

(e)

Fine During the period Shep has paid a fine of Rs. 6,000. The fine is not tax deductible.

(f)

Further information The accounting profit before tax for the year was Rs. 125,000.

Tax is chargeable at a rate of 30%. Required (a)

Calculate the corporate income tax liability for the year ended 31st December 2016.

(b)

Calculate the deferred tax balance that is required in the statement of financial position as at 31st December 2016.

(c)

Prepare a note showing the movement on the deferred tax account and thus calculate the deferred tax charge for the year ended 31st December 2016

(d)

Prepare the statement of profit or loss note which shows the compilation of the tax expense for the year ended 31st December 2016.

(e)

Prepare a note to reconcile the product of the accounting profit and the tax rate to the tax expense for year ended 31st December 2016.

© Emile Woolf International

80

The Institute of Chartered Accountants of Pakistan

Questions

12.4

SHEP (III) Continuing from the previous year. The following information is relevant for the year ended 31st December 2017. (a)

Interest payable/Interest receivable Shep still has Rs. 25,000 of 8% convertible loan stack in issue and still retains its holding in the debentures purchased in 2004.

(b)

Provision for warranty During the year Shep had paid out Rs. 500 in warranty claims and provided for a further Rs. 2,000.

(d)

Development costs During 2017 Shep has capitalised development expenditure of Rs. 17,800 in accordance with the provisions of IAS 38. Assume that tax relief on this expenditure is taken in full in the period in which it is incurred.

(e)

Further information Rs. Profit before taxation Depreciation charged Tax allowable depreciation

(f)

175,000 18,500 24,700

Entertainment Shep paid for a large office party during 2017 to celebrate a successful first two years of the business. This cost Rs. 20,000. Assume that this expenditure is not tax deductible.

Tax is chargeable at a rate of 30%. Required (a)

Calculate the corporate income tax liability for the year ended 31st December 2017.

(b)

Calculate the deferred tax balance that is required in the statement of financial position as at 31st December 2017.

(c)

Prepare a note showing the movement on the deferred tax account and thus calculate the deferred tax charge for the year ended 31st December 2017

(d)

Prepare the statement of profit or loss note which shows the compilation of the tax expense for the year ended 31st December 2017.

(e)

Prepare a note to reconcile the product of the accounting profit and the tax rate to the tax expense for year ended 31st December 2017.

© Emile Woolf International

81

The Institute of Chartered Accountants of Pakistan

Financial accounting and reporting II

12.5

SHEP (IV) Using the information provided in “Shep III” and assume that Shep is subject to a higher tax rate of 34% in 2017. Required

12.6

(a)

Calculate the corporate income tax liability for the year ended 31st December 2017.

(b)

Calculate the deferred tax balance that is required in the statement of financial position as at 31st December 2017.

(c)

Prepare a note showing the movement on the deferred tax account and thus calculate the deferred tax charge for the year ended 31st December 2017

(d)

Prepare the statement of profit or loss note which shows the compilation of the tax expense for the year ended 31st December 2017.

(e)

Prepare a note to reconcile the product of the accounting profit and the tax rate to the tax expense for year ended 31st December 2017.

WAQAR LIMITED Waqar Limited has provided you the following information for determining its tax and deferred tax expense for the year 2014 and 2015: (i)

During the year ended December 31, 2015, the company’s accounting profit before tax amounted to Rs. 40 million (2014: Rs. 30 million). The profit includes capital gains amounting to Rs. 10 million (2014: Rs. 8 million) which are exempt from tax.

(ii)

The accounting written down values of the fixed assets, as at December 31, 2013 were as follows: Accumulated Cost Rs. m Machinery Furniture and fittings

Depreciation Rs. m

Written down value Rs. m

200

25

175

50

10

40

No additions or disposals of fixed assets were made in the years 2014 and 2015. (iii)

Machinery was acquired on January 1, 2013 and is being depreciated on straight- line basis over its estimated useful life of 8 years. The tax base of machinery as at December 31, 2013 was Rs. 90 million.

(iv)

Furniture and fittings are also depreciated on the straight line basis at the rate of 10% per annum. The tax base of furniture and fittings as at December 31, 2013 was Rs. 40.5 million.

(v)

Normal rate of tax depreciation on both types of assets is 10% on written down value.

(vi)

The tax rates for 2013, 2014 and 2015 were 35%, 35% and 30% respectively.

© Emile Woolf International

82

The Institute of Chartered Accountants of Pakistan

Questions

Required For each year:

12.7

(a)

Calculate the corporate income tax liability for the year.

(b)

Calculate the deferred tax balance that is required in the statement of financial position as at the year end.

(c)

Prepare a note showing the movement on the deferred tax account and thus calculate the deferred tax charge for the year.

(d)

Prepare the statement of profit or loss note which shows the compilation of the tax expense.

(e)

Prepare a note to reconcile the product of the accounting profit and the tax rate to the tax expense. (25)

SHAKIR INDUSTRIES Given below is the statement of profit or loss of Shakir Industries for the year ended December 31, 2015: 2015 Rs. m 143.00 (96.60) 46.40 (28.70) 17.70 3.40 21.10 (5.30) 15.80

Sales Cost of goods sold Gross profit Operating expenses Operating profit Other income Profit before interest and tax Financial charges Profit before tax Following information is available: (i)

Operating expenses include an amount of Rs. 0.7 million paid as penalty to SECP on non-compliance of certain requirements of the Companies Ordinance, 1984.

(ii)

During the year, the company made a provision of Rs. 2.4 million for gratuity. The actual payment on account of gratuity to outgoing members was Rs. 1.6 million.

(iii)

Lease payments made during the year amounted to Rs. 0.65 million which include financial charges of Rs. 0.15 million. As at December 31, 2015, obligations against assets subject to finance lease stood at Rs. 1.2 million. The movement in assets held under finance lease is as follows: Rs. m 2.50 (0.7) 1.80

Opening balance – 01/01/2015 Depreciation for the year Closing balance – 31/12/2015

© Emile Woolf International

83

The Institute of Chartered Accountants of Pakistan

Financial accounting and reporting II

(iv)

(v)

(vi)

The details of owned fixed assets are as follows: Accounting Tax Rs. m Rs. m Opening balance – 01/01/2015 12.50 10.20 5.30 5.30 5.3 5.3 Purchased during the year (1.10) (1.65) (1.1) Depreciation for the year 5.30 5.30 16.70 13.85 Closing balance – 31/12/2015 (1.10) (1.65) Capital work-in-progress as on December 31, 2015 include financial charges of Rs. 2.3 million which have been capitalised in accordance with IAS-23 “Borrowing Costs”. However, the entire financial charges are admissible, under the Income Tax Ordinance, 2001. Deferred tax liability and provision for gratuity as at January 1, 2015 was Rs. 0.55 million and Rs. 0.7 million respectively.

(vii) Applicable income tax rate is 35%. Required Based on the available information, compute the current and deferred tax expenses for the year ended December 31, 2015.

12.8

(15)

MARS LIMITED Mars Limited (ML) is engaged in the manufacturing of chemicals. On July 1, 2014 it obtained a motor vehicle on lease from a bank. Details of the lease agreement are as follows: (i)

Cost of motor vehicle is Rs. 1,600,000.

(ii)

Instalments of Rs. 480,000 are to be paid annually in advance.

(iii)

The lease term and useful life is 4 years and 5 years respectively.

(iv)

The interest rate implicit in the lease is 13.701%.

ML follows a policy of depreciating the motor vehicles over their useful life, on the straight-line method. However, the tax department allows only the lease payments as a deduction from taxable profits. The tax rate applicable to the company is 30%. ML’s accounting profit before tax for the year ended June 30, 2015 is Rs. 4,900,000. There are no temporary differences other than those evident from the information provided above. Required (a)

Prepare journal entries in the books of Mars Limited for the year ended June 30, 2015 to record the above transactions including tax and deferred tax.

(b)

Prepare a note to the financial statements related to disclosure of finance lease liability, in accordance with the requirements of IFRS. (18)

(Ignore comparative figures.)

© Emile Woolf International

84

The Institute of Chartered Accountants of Pakistan

Questions

12.9

BILAL ENGINEERING LIMITED Bilal Engineering Limited earned profit before tax amounting to Rs. 50 million during the year ended December 31, 2015. The accountant of the company has submitted draft accounts to the Finance Manager along with the following information which he believes could be useful in determining the amount of taxation: (i)

Accounting deprecation for the year is Rs. 10 million which includes Rs. 1 million charged on the difference between cost and revalued amount.

(ii)

A motor vehicle costing Rs. 1 million was taken on lease in 2014. Related clauses of the lease agreement are as under: 

Annual instalment of Rs. 0.3 million is payable annually in advance.



The lease term and useful life is 4 years and 5 years respectively.



The interest rate implicit in the lease is 13.701% per annum.



Accounting depreciation on the leased vehicle is included in the depreciation referred to in para (i) above.

(iii)

Tax depreciation on the assets owned by the company is Rs. 7 million.

(iv)

Research and development expenses of Rs. 15 million were incurred in 2013 and are being amortised over a period of 15 years. For tax purposes research and development expenses are allowed to be written off in 10 years. However, 10% of these expenses were not verifiable and have not been claimed.

(v)

Expenses amounting to Rs. 0.25 million were disallowed in 2012. Out of these Rs. 0.15 million were allowed in appeal, during the current year. The company had initially expected that the full amount would be allowed but has decided not to file a further appeal.

(vi)

The applicable tax rate is 35%.

Required (a)

Prepare journal entries in respect of taxation, for the year ended December 31, 2015.

(b)

Prepare a reconciliation to explain the relationship between tax expense and accounting profit as is required to be disclosed under IAS 12 Income Taxes. (18)

12.10 GALAXY INTERNATIONAL The following information relates to Galaxy International (GI), a listed company, which was incorporated on January 1, 2014. (i)

The (loss) / profit before taxation for the years ended December 31, 2014 and 2015 amounted to (Rs. 1.75 million) and Rs. 23.5 million respectively.

(ii)

The details of accounting and tax depreciation on fixed assets is as follows: 2015 Rs. m 15 6

Accounting depreciation Tax depreciation (iii)

2014 Rs. m 15 45

In 2014, GI accrued certain expenses amounting to Rs. 2 million which were disallowed by the tax authorities. However, these expenses are expected to be allowed on the basis of payment in 2015.

© Emile Woolf International

85

The Institute of Chartered Accountants of Pakistan

Financial accounting and reporting II

(iv)

GI earned interest on Special Investment Bonds amounting to Rs. 1.0 million and Rs. 1.25 million in the years 2014 and 2015 respectively. This income is exempt from tax.

(v)

GI operates an unfunded gratuity scheme. The provision during the years 2014 and 2015 amounted to Rs. 1.7 million and Rs. 2.2 million respectively. No payment has so far been made on account of gratuity.

(vi)

The applicable tax rate is 35%.

Required Prepare a note on taxation for inclusion in the company’s financial statements for the year ended December 31, 2015 giving appropriate disclosures relating to current and deferred tax expenses including a reconciliation to explain the relationship between tax expense and accounting profit. (20)

12.11 APRICOT LIMITED The following information relates to Apricot Limited (AL), a listed company, for the financial year ended 31 December 2015: (i)

The profit before tax for the year amounted to Rs. 60 million (2014: Rs. 45 million).

(ii)

The accounting and tax written down value of fixed assets as on 31 December 2014 was Rs. 95 million and Rs. 90 million respectively. Accounting depreciation for the year is Rs. 10 million (2014: Rs. 9 million) whereas tax depreciation for the year is Rs. 8 million (2014: Rs. 7 million).

(iii)

During the year, AL sold a machine for Rs. 3 million and recognised a profit of Rs. 0.5 million. The tax written down value of the machine as on 31 December 2014 was Rs. 2 million. There were no other additions/disposals of fixed assets in 2014 and 2015.

(iv)

AL earned capital gain of Rs. 6 million (2014:Nil) on sale of shares of a listed company. This income is exempt from tax.

(v)

Bad debt expenses recognised during the year was Rs. 5 million (2014: Rs. 7 million).

(vi)

Bad debts written off during the year amounted to Rs. 3 million (2014: Rs. 4 million).

(vii) Deferred tax liability and provision for bad debts as on 31 December 2011 was Rs. 18.90 million and Rs. 9 million respectively. (viii) The company’s assessed brought forward losses up to 31 December 2011 amounted to Rs. 19.25 million. (ix)

Applicable tax rate is 35%.

Required Prepare a note on taxation for inclusion in AL’s financial statements for the year ended 31 December 2015 giving appropriate disclosures relating to current and deferred tax expenses including comparative figures for 2014 and a reconciliation to explain the relationship between 2015 tax expense and 2015 accounting profit. (21)

© Emile Woolf International

86

The Institute of Chartered Accountants of Pakistan

Questions

CHAPTER 13 – RATIO ANALYSIS 13.1

WASIM Wasim is an importer and retailer of vegetable oils. Extracts from the financial statements for this year and last are set out below. Income statements for the years ended 30 September Year 7 Rs.000 Revenue

Year 6 Rs.000 1,806

2,160 (1,755) –––––– 405 (130) (260) –––––– 15 (6) –––––– 9 ––––––

Cost of sales Gross profit Distribution costs Administrative expenses Profit before tax Income tax expense Profit for the period

(1,444) –––––– 362 (108) (198) –––––– 56 (3) –––––– 53 ––––––

Statements of financial position as of 30 September Year 7 Rs.‘000 Assets Non-current assets Property, plant and equipment Current assets Inventories Trade receivables Cash Total assets Equity and liabilities Equity Ordinaryshares Preference shares Share premium Revaluation reserve Retained earnings Current liabilities Bank overdraft Trade payables Current tax payable Total equity and liabilities

© Emile Woolf International

87

Year 6 Rs.‘000

78

72

106 316 ––––– 422 ––––– 500 –––––

61 198 6 ––––– 265 ––––– 337 –––––

110 23 15 20 78 246

85 11 20 74 190

49 198 7 ––––– 254 ––––– 500 –––––

142 5 ––––– 147 ––––– 337 –––––

The Institute of Chartered Accountants of Pakistan

Financial accounting and reporting II

Required Define and calculate the following ratios:

13.2

a)

Gross profit percentage.

b)

Net profit percentage

c)

Return on capital employed

d)

Asset turnover

e)

Current ratio

f)

Quick ratio

g)

Average receivables collection period

h)

Average payables period

i)

Inventory turnover

AMIR AND MO The income statements and statements of financial position of two manufacturing companies in the same sector are set out below. Amir Mo Rs. Rs. Revenue 150,000 700,000 Cost of sales (60,000) (210,000) –––––––– –––––––– Gross profit 90,000 490,000 Interest payable (500) (12,000) Distribution costs (13,000) (72,000) Administrative expenses (15,000) (35,000) –––––––– –––––––– Profit before tax 61,500 371,000 Income tax expense (16,605) (100,170) –––––––– –––––––– Profit for the period 44,895 270,830 –––––––– –––––––– Assets Non-current assets Property 500,000 Plant and equipment 190,000 280,000

Current assets Inventories Trade receivables Cash at bank

Total assets

© Emile Woolf International

88

––––––– 190,000

––––––– 780,000

12,000 37,500 500 ––––––– 50,000 ––––––– 240,000 –––––––

26,250 105,000 22,000 ––––––– 153,250 ––––––– 933,250 –––––––

The Institute of Chartered Accountants of Pakistan

Questions

Equity and liabilities Equity Share capital Retained earnings

Non-current liabilities Long-term debt Current liabilities Trade payables Total equity and liabilities

156,000 51,395 ––––––– 207,395

174,750 390,830 ––––––– 565,580

10,000

250,000

22,605 ––––––– 240,000 –––––––

117,670 ––––––– 933,250 –––––––

Required Define and calculate the following ratios for each company: a)

Gross profit percentage.

b)

Net profit percentage

c)

Return on capital employed

d)

Asset turnover

e)

Current ratio

f)

Quick ratio

g)

Average receivables collection period

h)

Average payables period

i)

Inventory turnover

© Emile Woolf International

89

The Institute of Chartered Accountants of Pakistan

Financial accounting and reporting II

CHAPTER 14 – ETHICAL ISSUES IN FINANCIAL REPORTING 14.1

ETHICAL ISSUES Waheed is a chartered accountant, recently employed by AA plc as deputy to the finance director, Arif (also a chartered accountant). AA plc is listed on the Lahore stock exchange. On Waheed’s first day on the job he met with Arif who said ‘Look, keep it to yourself but I’m having a second interview next week for a new job. The first thing that I need you to do is to review the financial statements before the auditors arrive. I qualified a few years ago and am not up to date on all of the little technicalities in IFRS. You should now these better than me and you’ll know more about what the auditors might focus on. We must do our best to present the financial statements in the most favourable light as the bonus paid to employees (including me) depends on profit being more than 10% bigger than last year’s and remember that you qualify for this too. Keep this in mind when you carry out the review as we do not really want to find anything. Do well at this and I might put in a good word for you when I leave as I’m sure you’ll be a great replacement for me.” Required Explain the ethical issues inherent in the above conversation and what Waheed should do about them.

14.2

SINDH INDUSTRIES LTD Jafar has recently been appointed as financial controller to Sindh Industries Ltd. Until a month ago, Sindh Industries had a finance director, who resigned suddenly, due to ill health. Since Jafar joined the company, he has learned that his resignation was related to stress caused by a series of disagreements with the managing director about the performance of the business.. The directors have not yet appointed a replacement. It is now March 2016 and you have been asked to finalise the financial statements for the year ended 31 December 2015. The draft statement of profit or loss extract and statement of financial position are shown below: Draft statement of profit or loss for the year ended 31 December 2015 Rs. 000 2,500

Profit before tax Draft statement of financial position at 31 December 2015 Property, plant and equipment Current assets Total assets

Rs. 000 12,000 3,500 15,500

Share capital Retained earnings Equity Non-current liabilities Current liabilities Total equity and liabilities

2,000 6,000 8,000 5,000 2,500 15,500

© Emile Woolf International

90

The Institute of Chartered Accountants of Pakistan

Questions

During the year ended 31 December 2015 Sindh Industries entered into the following transactions. (1)

Just before the year end Sindh Industries signed a contract to deliver consultancy services for a period of 2 years at a fee of Rs. 500,000 per annum. The full amount of this fee has been paid in advance and is nonrefundable.

(2)

Sindh Industries has constructed a new factory. The construction has been financed from the pool of existing borrowings. Land at a cost of Rs. 1.8 million was acquired on 1 February 2015 and construction began on 1 June 2015. Construction was completed on 30 September 2015 at an additional cost of Rs. 2.7 million. Although the factory was usable from that date, full production did not commence until 1 December 2015. Throughout the year the company’s average borrowings were as follows:

Amount Rs. 1,000,000 1,750,000 2,500,000

Bank overdraft Bank loan Debenture

Annual interest rate % 9.75 10 8

An amount of Rs. 450,000 has been included in property, plant and equipment in respect of borrowing costs relating to the construction of the factory. The useful life of the factory has been estimated at 20 years. No depreciation has been charged for the year. The reason for this is that the factory has only been in use for one month and that the depreciation charge would be immaterial. (3)

A blast furnace with a carrying amount at 1 January 2015 of Rs. 3.5 million has been depreciated in the draft financial statements on the basis of a remaining life of 20 years. In December 2015 the directors carried out a review of the useful lives of various significant items of plant and machinery, including the blast furnace and came to the conclusion that the useful life of the furnace was 20 years at 31 December 2015. The reasoning behind this judgement was that the lining of the furnace had been replaced in the last week of December 20X6 at a cost of Rs. 1.4 million. Provided that the lining is replaced every five years, the life of the furnace can be extended accordingly. You have found a report, commissioned by the previous finance director and prepared by a firm of asset valuation specialists, which assesses the remaining useful life of the main structure of the furnace at 1 January 2015 at 15 years and the lining of the furnace at 5 years. You have also found evidence that the managing director has seen this report. Jafar has had a conversation with the managing director who told him, “We need to make the figures look as good as possible so I hope you’re not going to start being difficult. The consultancy fee is non-refundable so there’s no reason why we can’t include it in full. I think we should look at our depreciation policies. We’re writing off our assets over far too short a period. As you know, we’re planning to go for a stock market listing in the near future and being prudent and playing safe won’t help us do that. It won’t help your future with this company either.”

© Emile Woolf International

91

The Institute of Chartered Accountants of Pakistan

Financial accounting and reporting II

Required (a)

Explain the required IFRS accounting treatment of these issues, preparing relevant calculations where appropriate. (17)

(b)

Prepare a revised draft of the statement of profit or loss extract for the year ended 31 December 2015 and the statement of financial position at that date. (6)

(c)

Discuss the ethical issues arising from your review of the draft financial statements and the actions that you should consider.

© Emile Woolf International

92

(5)

The Institute of Chartered Accountants of Pakistan

SECTION

Certificate in Accounting and Finance Financial accounting and reporting II

B Answers

© Emile Woolf International

93

The Institute of Chartered Accountants of Pakistan

Financial accounting and reporting II

CHAPTER 2 – IAS 1: PRESENTATION OF FINANCIAL STATEMENTS 2.1

LARRY Statement of profit or loss For the year ended 31 December 2015 Rs. in million Revenue Cost of sales (2,542 + 118 – 127) Gross profit Other income Distribution costs Administrative expenses Profit before tax Income tax expense

3,304 (2,533) 771 20 (175) (342) 274 (75)

Profit for the period

199

Statement of financial position As at 31 December 2015 Assets

Rs. in million

Non-current assets Property, plant and equipment (2,830 – 918) Intangible assets (26 – 5)

1,912 21 1,933

Current assets Inventories Trade and other receivables Cash (89 +2)

127 189 91 407

Total assets

2,340

Equity and liabilities Equity Share capital Retained earnings (1,562 + 199)

400 1,761 2,161

Non-current liabilities Long-term borrowings (18 x 2/3) Current liabilities Trade and other payables Current portion of long-term borrowing (18 ÷ 3) Current tax payable Total equity and liabilities

© Emile Woolf International

12 86 6 75 167 2,340

94

The Institute of Chartered Accountants of Pakistan

Answers

2.2

MINGORA IMPORTS LIMITED Statement of profit or loss for the year ended 31 December 2015 Rs. in million Revenue Change in inventories of finished goods and work-in-progress (W3) Staff costs (W3) Depreciation and other amortisation expense (W3) Other expenses (W3) Profit before tax Income tax expense

1,740 40 (620) (42) (359) 759 (120)

Profit for the period

639

Statement of financial position as at 31 December 2015 Rs. in million

Assets Non-current assets Property, plant and equipment (W1) Intangible assets (W2)

368 40 408

Current assets Inventories (180 + 140) Trade and other receivables (420 x 95%) Cash

320 399 440 1,159

Total assets

1,567

Equity and liabilities Equity Share capital Other reserves Retaind earnings

600 120 635 1,355

Current liabilities Trade and other payables Current tax payable

92 120 212

Total equity and liabilities

1,567

Statement of changes in equity for the year ended 31 December 2015 Share capital

Amounts in Rs. million Revaluati Retained on earnings reserve 121 (125) 120 -

Total

Balance at 31 December 2014 Dividends paid Net revaluation surplus in the year (360 – (300 – 60)) Profit after tax for the period

620

-

-

639

639

Balance at 31 December 2015

620

120

635

1,355

© Emile Woolf International

95

721 (125) 120

The Institute of Chartered Accountants of Pakistan

Financial accounting and reporting II

Workings (1)

Property, plant and equipment Rs. in million Cost brought forward Leasehold Computers Revaluation

300 50 60

Cost carried forward

410

Accumulated depreciation brought forward (60 + 20) Revaluation Charge for the year Leasehold (360 ÷ 30) Computers (50 ÷ 5)

80 (60) 12 10

Accumulated depreciation carried forward

42

Carrying amount carried forward (2)

368

Intangible assets Rs. in million Cost Amortisation (60 ÷ 3)

60 (20)

Carried forward (3)

40

Allocation of costs Amounts in Rs. million

Work-in-progress (140 – 125) Staff costs Finished goods (180 – 155) Consultancy fees Directors’ salaries Doubtful receivables (420  5%) Sundry Amortisation of patent (W2) Depreciation (12 + 10) (W1)

Change in inventori es (15)

Depreciat ion etc

Other expenses

260 (25) 44 360 21 294 20 22 (40)

© Emile Woolf International

Staff costs

96

620

42

359

The Institute of Chartered Accountants of Pakistan

Answers

2.3

BARRY Barry Statement of profit or loss For the year ended 31st August 2015 Rs. in million 30,000 (19,650) 10,350 (1,370) (1,930) 7,050 (350) 6,700 (2,500)

Revenue Cost of sales (W1) Gross profit Distribution costs (W1) Administrative expenses (W1) Profit from operations Finance costs Profit before tax Tax (W2) Profit after tax Barry Statement of financial position As at 31st August 2015

4,200

Rs. in million ASSETS Non-current assets Property, plant and equipment Current assets Inventory Trade and other receivables (7,400 + 200) Cash and cash equivalents Total assets

4,600 7,600 700 12,900 52,500

EQUITY AND LIABILITIES Capital and reserves Equity shares Share premium Accumulated profits (W3) Total equity Revaluation reserve (W4) Non current liabilities Borrowings Current liabilities Trade and other payables Taxation (2,100 + 400)

21,000 2,000 11,800 34,800 4,700 5,200 5,300 2,500 7,800

Total equity and liabilities

© Emile Woolf International

39,600

52,500

97

The Institute of Chartered Accountants of Pakistan

Financial accounting and reporting II

Reconciliation of opening and closing property, plant and equipment Rs. in ‘000 Assets Fixtures under & construct fittings ion

Buildings

Plant & machine ry

10,000

9,000

20,100

10,000

400

49,500

Additions

-

-

-

-

50

50

Reclassification

-

-

450

-

(450)

-

1,000

1,000

-

-

-

2,000

11,000

10,000

20,550

10,000

-

51,550

At 1 Sept 2014

-

3,000

4,000

3,700

-

10,700

Revaluation

-

(3,000)

-

-

-

(3,000)

Charge for year

-

1,000

2,550

700

-

4,250

At 31 Aug 2015

-

1,000

6,550

4,400

-

11,950

At 31 Aug 2015

11,000

9,000

14,000

5,600

-

39,600

At 1 Sept 2014

10,000

6,000

16,100

6,300

400

38,800

Land

Total

Cost/ Valuation At 1 Sept 2014

Revaluation At 31 Aug 2015 Depreciation

Net book value

Workings 1

Rs.in ‘000

Allocation of expenses

Raw materials consumed Manufacturing overheads Increase in inventories Staff costs (70%/20%/10%) Distribution costs Depreciation Building (50%/50%) Plant and machinery Fixtures and fittings (30%/70%) 2

Cost of sales 9,500 5,000 (1,400) 3,290

500 2,550 210 19,650

Admin

940

Distrib

470 900

500 490 1,930

1,370

Tax charge

Current year Under provision from previous year

© Emile Woolf International

98

Rs. in ‘000 2,100 400 2,500

The Institute of Chartered Accountants of Pakistan

Answers

3

Accumulated profits carried forward Accumulated profits carried forward per question Less tax charge - Current year estimate - Under-provision in previous year Add transfer of excess depreciation on revalued building

4

2,100 400 (2,500) 300 11,800

Revaluation reserve carried forward Revaluation reserve per question Add transfer of excess depreciation on revalued building

2.4

Rs. in ‘000 14,000

5,000 (300) 4,700

OSCAR INC (a)

Statement of profit or loss For the year ended 31 March 2015 Rs. in ‘000 Sales Operating costs (140 + 960 – 150 + 420 + 210 + 16) Operating profit before interest Income from investments Profit before taxation Income tax

Statement of financial position As at 31 March 2015 Assets Non-current assets Tangible assets Investments Current assets Inventory Receivables

530 560 ———— 1,090 150 470 ———— 620 ———— 1,710 ————

Equity and liabilities Capital and reserves Share capital Retained earnings Current liabilities Provisions for liabilities and charges

© Emile Woolf International

2,010 (1,596) ———— 414 75 ———— 489 (49) ———— 440 ————

99

600 500 ———— 1,100 414 196 ———— 1,710 ————

The Institute of Chartered Accountants of Pakistan

Financial accounting and reporting II

Workings (1)

Income tax Rs. in ‘000 Income tax (current year) Over provision for tax in the previous year

(2)

74 (25) —— 49 ——

Tangible assets – plant and machinery Rs. in ‘000 Cost at 1 April 2014 and 31 March 2015 Accumulated depreciation At 31 March 2014 Provided during the year (27 + 5)

188 32 —— 220 ——

At 31 March 2015 Net book value at 31 March 2015

(3)

750 ——

530 ——

Current liabilities Rs. in ‘000 Trade payables Mainstream corporation tax Bank overdraft

(4)

Provisions for liabilities and charges Rs. in ‘000 180 16 —— 196 ——

At 1 April 2014 Provided in the year At 31 March 2015

(5)

260 74 80 —— 414 ——

Retained earnings Retained earnings Opening retained earnings Dividends Closing retained earnings

© Emile Woolf International

100

Rs. in ‘000 440 180 (120) ——— 500 ———

The Institute of Chartered Accountants of Pakistan

Answers

2.5

CLIFTON PHARMA LIMITED (a)

Clifton Pharma Limited Statement of profit or loss for the year ended 30 September 2015

Revenue Cost of sales: see working (1) Gross profit Operating expenses: see working (2) Investment income Finance costs: Loan notes – see working (3) Finance lease – see working (2) Profit before tax Income tax expense: see working (4) Profit for the period (b)

98,000

Clifton Pharma Limited Statement of financial position as at 30 September 2015 Non-current assets Property, plant and equipment: see working (5) Investments Current assets Inventory Trade receivables Bank

358,000 92,400 450,400 23,700 76,400 12,100 112,200

Total assets

562,600

Equity and liabilities Capital and reserves Share capital Share premium Retained earnings: see working (6) Revaluation surplus Non-current liabilities 3% loan notes: see working (3) Deferred tax: see working (4) Finance lease obligation: see working (2) Current liabilities Trade payables Accrued lease finance costs: see working (2) Finance lease obligation: see working (2) Income tax payable Total equity and liabilities

© Emile Woolf International

Rs. in ‘000 338,300 (180,000) 158,300 (36,600) 2,000 (3,000) (1,700) (4,700) 119,000 (21,000)

280,000 20,000 117,300 417,300 20,000 51,500 23,000 11,700 86,200 14,100 1,700 5,300 18,000 39,100 562,600

101

The Institute of Chartered Accountants of Pakistan

Financial accounting and reporting II

Workings Rs. in ‘000

(1) Cost of sales As given in the trial balance

134,000

Depreciation of plant and equipment: 20%  (197,000 – 47,000) Depreciation of leased vehicles: 24,000/4 years

30,000

Amortisation of leasehold property: 250,000/25 years

10,000

6,000 180,000

(2) Vehicle rentals and finance lease. Operating expenses Rental costs given in the trial balance Relating to finance lease

8,600 (7,000)

Balance: relating to operating lease – operating expense

1,600

Other operating expenses (trial balance in question)

35,000

Total operating expenses

36,600

Finance lease Fair value of leased assets

24,000

Less: First rental payment, paid in advance 1 October 2014

(7,000)

Remaining obligation, 1 October 2014

17,000

Interest at 10% to 30 September 2015 (current liability)

1,700

Lease payment due 1 October 2015

7,000

Capital repayment due (= balance, current liability)

(5,300)

Remaining lease obligation = non-current liability

11,700

(3)

Loan notes The effective interest rate is 6%. Actual interest paid was Rs.1,500,000 (in trial balance); therefore the balancing Rs.1,500,000 should be added to the loan notes obligation, to make the total loan notes liability Rs.50 million + Rs.1,500,000 = Rs.51.5 million.

(4)

Taxation Deferred tax liability b/f

20,000

Deferred tax: credit in the statement of profit or loss Deferred tax liability c/f (92,000  25%)

2,000 23,000

Tax expense Income tax on profits for the year

18,000

Deferred tax movement

3,000

Tax charge in the statement of profit or loss

© Emile Woolf International

102

21,000

The Institute of Chartered Accountants of Pakistan

Answers

(5) Non-current assets and depreciation Rs. in ‘000

Leasehold property Carrying value in the trial balance (250,000 – 40,000)

210,000

Amortisation charge for the year to 30 September 2015

(10,000) 200,000

Re-valued amount

220,000

Transfer to revaluation reserve

20,000

The annual depreciation charges for plant and equipment and the leased vehicles are shown in workings (1) Rs. in ’000 Cost or valuation

Accumulated depreciation

Carrying amount

Leasehold property

220,000

0

220,000

Plant and equipment (non-leased)

197,000

77,000

120,000

24,000

6,000

18,000

441,000

83,000

358,000

Leased vehicles

(6) Retained profits At 1 October 2014 (trial balance)

19,300

Profit for the year

98,000

Retained profits at 30 September 2015

2.6

117,300

SARHAD SUGAR LIMITED (a)

Sarhad Sugar Limited – Statement of profit or loss for the year ended 30 September 2015 Revenue (300,000 – 2,500) Cost of sales (w (i)) Gross profit Distribution costs Administrative expenses (22,200 – 400 + 100 see note below) Finance costs Profit before tax (Income tax expense (11,400 + (6,000 – 5,800 deferred tax)) Profit for the year

(b)

297,500 (225,400) 72,100 (14,500) (21,900) (1,000) 34,700 (11,600) 23,100

Sarhad Sugar Limited Statement of financial position as at 30 September 2015 Assets Non-current assets (w (ii)) Property, plant and equipment (43,000 + 38,400) Development costs Current assets Inventory Trade receivables

© Emile Woolf International

81,400 14,800 96,200 20,000 43,100

103

The Institute of Chartered Accountants of Pakistan

Financial accounting and reporting II

63,100 Total assets Equity and liabilities: Equity Share capital Retained earnings (w (iii))

159,300

70,000 41,600 117,100 5,500

Revaluation reserve (w (iii)) Non-current liabilities Deferred tax

6,000

Current liabilities Trade payables (23,800 – 400 + 100 – re legal action) Bank overdraft Current tax payable Total equity and liabilities

23,500 1,300 11,400 36,200 159,300

Note: As it is considered that the outcome of the legal action against Sarhad Sugar Limited is unlikely to succeed (only a 20% chance) it is inappropriate to provide for any damages. The potential damages are an example of a contingent liability which should be disclosed (at Rs.2 million) as a note to the financial statements. The unrecoverable legal costs are a liability (the start of the legal action is a past event) and should be provided for in full. Workings (figures in brackets in Rs.000) (i) Cost of sales:

(ii)

Rs. in ‘000

Per trial balance 204,000 Depreciation (w (iii)) – leasehold property 2,500 – plant and equipment 9,600 Loss on disposal of plant (4,000 – 2,500) 1,500 Amortisation of development costs (w (iii)) 4,000 Research and development expensed (1,400 + 2,400 (w (iii)) 3,800 –––––––– 225,400 ══════ Non-current assets: Leasehold property Valuation at 1 October 2014 Depreciation for year (20 year life) Carrying amount at date of revaluation Valuation at 30 September 2015 Revaluation deficit Plant and equipment per trial balance (76,600 – 24,600) Disposal (8,000 – 4,000) Depreciation for year (20%) Carrying amount at 30 September 2015

© Emile Woolf International

104

50,000 (2,500) –––––––– 47,500 (43,000) –––––––– 4,500 ══════ 52,000 (4,000) –––––––– 48,000 (9,600) –––––––– 38,400 ══════

The Institute of Chartered Accountants of Pakistan

Answers

Rs. in ‘000

Capitalised/deferred development costs Carrying amount at 1 October 2014 (20,000 – 6,000) Amortised for year (20,000 x 20%) Capitalised during year (800 x 6 months) Carrying amount at 30 September 2015

14,000 (4,000) 4,800 –––––––– 14,800 ══════

Note: development costs can only be treated as an asset from the point where they meet the recognition criteria in IAS 38 Intangible assets. Thus development costs from 1 April to 30 September 2015 of Rs.4·8 million (800 x 6 months) can be capitalised. These will not be amortised as the project is still in development. The research costs of Rs.1·4 million plus three months’ development costs of Rs.2·4 million (800 x 3 months) (i.e. those incurred before 1 April 2015) are treated as an expense. (iii)

Movements on reserves Revaluation Retained surplus earnings Rs. in ‘000 10,000 24,500 (6,000) 23,100 (4,500)

Balances at 1 October 2014 Dividend Comprehensive income Revaluation loss Balances at 30 September 2015

2.7

5,500

41,600

BSZ LIMITED BSZ Limited Statement of financial position as at June 30, 2015

ASSETS Fixed Assets Property, plant & equipment Intangible assets Long term advances – considered good Current assets Stocks in trade Accounts receivable Advances, deposits, prepayments and other receivables Cash at banks

Note

Rs. in million

1 2

576 8 584 4

3 4 5

90 57 45 29 221 809

© Emile Woolf International

105

The Institute of Chartered Accountants of Pakistan

Financial accounting and reporting II

Rs. in million EQUITY AND LIABILITIES Share capital and reserves Authorized share capital 50,000,000 shares of Rs. 10 each

500

Issued, subscribed and paid up capital 40,000,000 shares of Rs. 10 each Unappropriated profit

400 65 465 120

Surplus on revaluation of fixed assets Non-current liabilities Deferred taxation

40

Current liabilities Short term loan Account and other payables Provision for taxation

85 82 17 184

6

809 Rs. in million

Notes 1. Property, plant and equipment Operating assets Capital work in progress – building

556 20 576

1.1 Operating assets Cost/revalued amount As of July 01 2014 Additions Disposals As at June 30 2015 Accumulated depreciation As of July 01 2014 For the year (105 × 85) + 10% × 15 × 8 /12) (105 × 19) + 10% × 8 × 3/12) Disposals

Rs. in million Freehold land 375.0 -

Building 130.0 -

Machines 100.0 (15.0)

Fixtures 19.0 8.0 -

Total 624.0 8.0 (15.0)

375.0

130.0

85.0

27.0

617.0

-

19.5 6.5

22.5

5.9

47.9 18.1

9.5

-

-

(5.0)

2.1 -

(5.0)

-

26.0

27.0

8.0

61.0

Carrying amount

375.0

104.0

58.0

19.0

556.0

Depreciation rate

-

5%

10%

10%

As at June 30 2015

© Emile Woolf International

106

The Institute of Chartered Accountants of Pakistan

Answers

1.2

Revaluation During the year 2011, the first revaluation of freehold land was carried out. The valuation was carried out under market value basis by an independent valuer, Mr. Dee, Chartered Civil Engineer of M/s SSS Consultants (Pvt.) Ltd., Islamabad. It resulted in a surplus of Rs. 120 million over book values which was credited to surplus on revaluation of fixed assets. Had there been no revaluation, the value of freehold land would be Rs. 255 million.

1.3

Disposal of machine Rs. in million 13.0 15.0 (5.0) (10.0)

Proceeds Cost Accumulated depreciation Carrying amount Profit on disposal

3.0 Note

2.

3.

Intangible Assets Cost of computer software/license Accumulated Amortization as of July 1, 2014 Amortization for the year Accumulated Amortization as of June 30, 2015 Carrying value as at June 30, 2015 Amortization rate

10.0 1.0 1.0 2.0 8.0 10%

Accounts Receivable Considered good - Secured - Unsecured Considered doubtful Less: Provision for bad debts

3.1

3.1 Provision for bad debts Balance as at July 1, 2014 Provision made during the year Amount written off during the year Balance as at June 30, 2015 (Rs. 30 million x 10%) 4

Advances, Deposits, Prepayments and Other Receivables Advances - suppliers - considered good - staffs Deposits Prepayments Sales tax receivable

© Emile Woolf International

107

2015 Rs. in million

30 27 57 3 60 3 57 3.4 1.0 (1.4) 3.0

12 6 18 11 4 12 45

The Institute of Chartered Accountants of Pakistan

Financial accounting and reporting II

5

Cash at banks Cash at banks - current accounts saving accounts

5.1

7 22 29

5.1: It carries interest / mark up ranging from 3% to 7% per annum. 6

2.8

Accounts and other payables Accounts payable Accrued liabilities

75 7 82

YASIR INDUSTRIES LIMITED Yasir Industries Limited Statement of Financial Position as at June 30, 2015 Assets Non-current assets Property, plant and equipment (W2) Intangible assets (20 – 12) Current assets Inventories (W6) Trade receivables (W5)

Rs. in million 351.00 8.00 359.00 64.50 39.00 103.50 462.50

Equity and Liabilities Equity Issued, subscribed and paid up capital Retained earnings (W4)

Revaluation surplus

41.25

Non-current liabilities Redeemable preference shares Debentures Deferred taxation (W 10)

40.00 80.00 9.00 129.00

Current liabilities Trade payables Accrued expenses (W3) Taxation Bank overdraft

30.40 25.00 16.50 13.25 85.15

Total equity and liabilities

© Emile Woolf International

120.00 87.10 207.10

462.50

108

The Institute of Chartered Accountants of Pakistan

Answers

Yasir Industries Limited Statement of profit or loss for the year ended June 30, 2015 Rs. in million 445.40 (250.72) 194.68 (20.05) (40.38) (9.10) 125.15 (30.00) 95.15 (19.50)

Sales revenue (W5) Cost of sales (W7) Gross profit Distribution costs (W8) Administrative expenses (W8) Financial charges (W9) Loss due to fraud Profit before tax Income tax expense (W10) Profit for the year

75.65

Workings (W1)

Leasehold property Annual depreciation before the revaluation (230 ÷ 40 years) = Rs. 5.75 million per annum. Depreciation this year has been charged incorrectly on cost (whereas it should have been on the revalued amount). This year’s charge must be added back Dr Cr Accumulated depreciation 5.75 Cost of sales (50%) 2.88 Administrative expenses (30%) 1.72 Distribution costs (20%) 1.15

Carrying amount at the 30 June (as per trial balance)(230.00 – 40.25) Add back depreciation incorrectly charged (see above) Carrying amount of property at the start of the year Revaluation surplus Revalued amount of leasehold property Less: WDV of leasehold property at revaluation Revaluation surplus arising in the year Transfer to retained earnings in respect of incremental depreciation (Rs. 7 million – Rs. 5.75 million)

Rs. in million 189.75 5.75 195.5 Rs. in million 238.00 195.50 42.50 (1.25) 41.25

Depreciation of revalued property Number of years depreciation by the year end: (40.25 ÷ 5.75) = 7 years. Therefore, remaining useful life as at the year-end = 33 years Revaluation was at the start of the year Remaining useful life at the start of the year = 34 years Depreciation charge based on the revalued amount (238/34 years) = Rs. 7 million

© Emile Woolf International

109

The Institute of Chartered Accountants of Pakistan

Financial accounting and reporting II

Dr 3.5 2.1 1.4

Cost of sales (50%) Administrative expenses (30%) Distribution costs (20%) Accumulated depreciation (W2)

Cr

7.00

Property, plant and equipment Rs. in million 231 120 351

Leasehold property (Rs. 238m – 7) Machines (Rs. 168.6 – Rs. 48.6m) (W3) Accrued Expenses

Rs. in million 15.00 4.80 4.00 23.80

As per trial balance Accrued interest on debentures (Rs. 80m × 12% × 6/12) Dividend on preference shares (Rs. 40m × 10%) (W4)

Retained earnings Rs. in million 10.20 75.65 1.25 87.10

Balance as per trial balance Profit for the year Transfer from revalution surplus (W5)

Sales and receivables

Given in the trial balance Deduct revenue incorrectly recognised (sale or return) Cost of sales (W6)

Sales. Rs. in million 478.40 (27.00) 451.40

66.00 (27.00) 39.00

Closing inventory

Given in the question Add back inventory held by customer on sale or return (100/120  27) Cost of sales (W7)

Rec. Rs. m

Rs. in million 42.00 22.50 64.50

Cost of sales

Opening inventory as of July 1, 2014 Purchases Direct labour Manufacturing overheads excluding incremental depreciation Less: Closing inventory Deduct depreciation incorrectly charged on cost Add depreciation charged on revalued amount Cost of sales

© Emile Woolf International

110

Rs. in million 38.90 175.70 61.00 39.00 (64.50) (2.88) 3.50 250.10

The Institute of Chartered Accountants of Pakistan

Answers

(W8)

Administrative expenses and distribution costs

Given in the trial balance Deduct depreciation incorrectly charged on cost Add depreciation charged on revalued amount Cost of sales (W9)

Admin. Rs. in million 40.00 (1.72) 2.10 40.38

DIst/ Rs. m 19.80 (1.15) 1.40 20.05

Financial charges

Balance as per trial balance Accrued interest on debentures (Rs. 80m × 12% × 6/12) Preference dividend for the year (Rs. 40m × 10%) (W10) Taxation Deferred taxation Balance b/f Charge for the year (balancing figure) Balance c/f (30%  Rs. 30 million temporary difference) Tax expense

Rs. in million 6.00 3.00 9.00 Rs. in million 16.50 3.00 19.50

Current tax Deferred tax (see above)

© Emile Woolf International

Rs. in million 0.30 4.80 4.00 9.10

111

The Institute of Chartered Accountants of Pakistan

Financial accounting and reporting II

2.9

SHAHEEN LIMITED Shaheen Limited Statement of financial position As of June 30, 2015 Assets Non-current assets Property, plant and equipment (86,000  12,000  4,500) Intangible assets (6,000  600) Current assets Stock in trade Trade receivables (37,800  10,000) Other receivables and prepayments (14,000 + 6,000) Cash and bank balances

Equity and liabilities Share capital and reserves issued, subscribed and paid up capital Unappropriated profit

12,000 6,000 5,000 9,998 32,998 157,425

Shaheen Limited Statement of profit or loss and other comprehensive income As of June 30, 2015 Sales revenue

© Emile Woolf International

112

30,000 27,800 20,000 4,725 82,525 157,425

25,525 3,530 29,055

Current liabilities Trade payables Current portion of long term borrowings Provision for litigation Provision for taxation (2,000 + 9,988  2,000)

Financial charges Profit before taxation Taxation (W3) Profit after taxation Other comprehensive income – net of tax Total comprehensive income

69,500 5,400 74,900

60,000 35,372 95,372

Non-current liabilities Long term borrowings (31,525  6,000) Deferred taxation (5,000  1,470)

Cost of sales (W2) Gross profit Selling and distribution expenses (W2) Administrative expenses (W2)

Rs. in ‘000

Rs. in ‘000 200,000 (104,708 ) 95,292 (36,275) (30,450) (66,725) (5,000) 23,567 (6,528) 17,039 17,039

The Institute of Chartered Accountants of Pakistan

Answers

Shaheen Limited Statement of changes in equity As of June 30, 2015

2015 Rs.000 Issued, Retained subscribed & earnings paid up capital 32,000* 60,000 (1,667) 60,000 30,333 17,039

Balance July 1, 2014 Correction of prior year error (10,000  20/120) Balance July 1, 2014 (restated) Comprehensive income for the year Dividend for the year ended June 30, 2014 (60,000*0.20)

(12,000)

Balance June 30, 2015

60,000

35,372

*Retained earnings as at 01-07-09 = 20,000+ (20% of 60,000)=32,000 Workings W1 Depreciation for the year On building (36,000/20) On plant and equipment (30,000  3,000)/10

1,800 2,700

Total

4,500

W2 Costs

Cost of sales

Opening inventory Costs as per Trial balance Closing inventory Depreciation (75%, 15%, and 10% of Rs. 4,500) Adjustment for goods sent on sale or return, erroneously booked as sales last year now returned during the year. (10,000/1.2) Amortization of export license (6,000/5*0.5)

23,000 100,000 (30,000) 3,375

Selling and Administrative distribution costs costs 35,000

30,000

675

450

8,333 600 104,708

36,275

30,450

W3:Taxation profit before tax Disallowances and add backs

23,567 5,000

Taxable income

28,567

Current

9,998 (2,000) (1,470)

Deferred

For the year (28,567*0.35) For prior years (7,000  5,000) For the year (5,000  800)*0.35

6,528

© Emile Woolf International

113

The Institute of Chartered Accountants of Pakistan

Financial accounting and reporting II

2.10

MOONLIGHT PAKISTAN LIMITED (a)

Moonlight Pakistan Limited Statement of Financial Position As at December 31, 2015 Rs. in million ASSETS Non-current assets Property, plant and equipment (W2)

3,472

Current assets Stocks in trade Trade receivables Cash and bank

758 702 354 1,814 5,286

EQUITY Issued, subscribed and paid-up capital (W3) Share premium (420 x 2/12) Retained earnings (W3)

Surplus on revaluation of fixed assets

1,750 70 876 2,696 240

LIABILITIES Non-current liabilities Long term loan Deferred tax (22 + 80 x 35%) Provision for gratuity

1,600 50 23 1,673

Current liabilities Creditor and other liabilities (544 + 96) Income tax payable

640 37 677 5,286

(b)

Moonlight Pakistan Limited Statement of profit or loss For the year ended December 31, 2015

Sales Cost of sales (W1) Gross profit Selling expenses (W1) Administrative expenses (W1)

Financial charges (210 + 1,600 x 12% x 6/12) Profit before taxation Taxation (37 + 80 x 35%) Profit after taxation

© Emile Woolf International

Rs. in million 3,608 (2,149) 1,459 252 270 522 937 306 631 65 566

114

The Institute of Chartered Accountants of Pakistan

Answers

W1: Cost of sales/selling expenses/admin expenses Cost of sales As per trial balance Depreciation – building (60% : 25% : 15%) (W2) Depreciation – plant Provision for gratuity (23-8) x 60%:20%:20%

1,784 69 287 9

Selling Admin. expenses expenses Rs. in million 220 250 29 17 3 3

2,149

252

270

W2: Property, plant and equipment Land

Cost as at January 1, 2015 Accumulated depreciation

Building Plant Rs. in million

600 -

Revaluation (1,840 - (2,000 - 400 )) Current year depreciation (1,840/16)

-

Total

2,000 (400)

2,104 (670)

4,704 (1,070)

240

(287)

240 (402)

1,147

3,472

(115) 600

1,725

W3: Share Capital/Retained Earnings Share capital Retained earnings Rs. in million 1,200 510 200 (200) 350 566

As per trial balance Bonus issue (1200 ÷ 6) Right issue (420 x 10/12) Profit for the year

1,750

2.11

876

FIGS PAKISTAN LIMITED Figs Pakistan Limited Statement of profit or loss and other comprehensive income For the year ended 31 December 2015

Note 1 2

Sales Cost of sales Gross profit Distribution costs Administrative expenses Other operating expenses Other operating income Profit from operations Finance costs Profit before tax Taxation Profit after tax Other comprehensive income

3 4 5 6 7 8

Total comprehensive income for the year

© Emile Woolf International

115

2015 Rs. in million 44,758 (26,203) 18,555 (6,431) (752) (399) 30 11,003 (166) 10,837 (2,532) 8,305 8,305

The Institute of Chartered Accountants of Pakistan

Financial accounting and reporting II

Figs Pakistan Limited Notes to the financial statements For the year ended 31 December 2015

1

Sales Manufactured goods Gross sales Sales tax

Note

56,528 (10,201) 46,327

Imported goods Gross sales Sales tax

1,078 (53) 1,025 (2,594) 44,758

Sales discounts

2

Rs. in million

Cost of sales Raw material consumed (1,751 + 22,603 - 2,125) Stores and spares consumed Salaries, wages and benefits (2,367 × 55%) Utilities (734 × 85%) Depreciation and amortizations (1.287 × 70%) Stationery and office expenses (230 × 25%) Repairs and maintenance (315 × 85%) Opening work in process Closing work in process Opening finished goods (manufactured) Closing finished goods (manufactured) Finished goods (imported) Opening stock Purchases

2.1

22,229 180 1,302 624 901 58 268 25,562 73 (125) 25,510 1,210 (1,153) 25,567 44 658 702 (66) 636

Closing stock

26,203 2.1

Salaries, wages and benefits include Rs. 30 million (54 × 55%) and Rs. 24 million (44 × 55%) in respect of defined contribution plan and defined benefit plan respectively.

© Emile Woolf International

116

The Institute of Chartered Accountants of Pakistan

Answers

3

Distribution costs Advertisement and sales promotion Outward freight and handling Salaries, wages and benefits (2,367 × 30%) Utilities (734 × 5%) Depreciation and amortization (1,287 × 20%) Stationery and office expenses (230 × 40%) Repairs and maintenance (315 × 5%)

3.2

Rs. in million 4,040 1,279 710 37 257 92 16 6,431

3.1

4

4.1

Salaries, wages and benefits include Rs. 16 million (54 × 30%) and Rs. 13 million (44×30%) in respect of defined contribution plan and defined benefit plan respectively. Administrative expenses Salaries, wages and benefits (2,367 × 15%) Utilities (734 × 10%) Depreciation and amortization (1,287 × 10%) Stationery and office expenses (230 × 35%) Repairs and maintenance (315 × 10%) Legal and professional charges Auditor's remuneration

4.2

Salaries, wages and benefits include Rs. 8 million (54 × 15%) and Rs. 7 million (44×15%) in respect of defined contribution plan and defined benefit plan respectively. Rs. in million 8 4 1 13

4.2

Auditor's remuneration Audit fees Taxation services Out of pocket expenses

5

Other operating expenses Donation Worker's Profit Participation Fund Worker Welfare Fund Loss on disposal of property, plant and equipment

5.1

4.1

Rs. in million 355 73 129 80 31 71 13 752

5.1

34 257 98 10 399

Donations Donations include Rs. 5 million given to Dates Cancer Foundation (DCF). One of the company’s directors, Mr. Peanut is a trustee of DCH. Donations other than that mentioned above were not made to any donee in which a director or his spouse had any interest at any time during the year.

© Emile Woolf International

117

The Institute of Chartered Accountants of Pakistan

Financial accounting and reporting II

6

7

8

Other operating income Income from financial assets Dividend income Return on savings account Income from non-financial assets Scrap sales Finance costs Finance charges on short term borrowings Exchange loss Finance charges on lease Taxation Current - for the year Deferred (3,120 × 35%)

© Emile Woolf International

Rs. in million 12 2 16 30 133 22 11 166 1,440 1,092 2,532

118

The Institute of Chartered Accountants of Pakistan

Answers

CHAPTER 3 – IAS 7: STATEMENTS OF CASH FLOWS 3.1

KLEA Statement of cash flows for the year ended 31st March 2015 Rs. in ‘000 Cash flows from operating activities Profit before taxation Adjustments for: Depreciation (W4) Finance income Interest expense

1,606 800 (50) 320

––––––

2,676 (400) (1,200) 334

Increase in trade receivables Increase in inventories Increase in trade payables

––––––

Cash generated from operations Interest paid Income taxes paid (W1)

1,410 (320) (630)

––––––

Net cash from operating activities Cash flows from investing activities Purchase of intangible assets (W2) Purchase of property, plant and equipment (W3) Proceeds from sale of equipment Purchase of long-term investments Finance income received

460 (300) (1,600) 150 (200) 50 ––––––

Net cash used in investing activities

(1,900)

Cash flows from financing activities Proceeds from issue of share capital (1,000 + 278) Payments to redeem debentures Dividends paid

1,278 (400) (400)

––––––

Net cash used in financing activities

478

–––––

Net decrease in cash and cash equivalents Cash and cash equivalents at 1 April 2014

(962) 580

–––––

Cash and cash equivalents at 31 March 2015 (32 - 414)

(382)

(Note: Alternative classifications of the cash flows in accordance with IAS 7 should receive full credit – i.e. interest and dividends received as investing activities or operating cash flows, interest and dividends paid as financing or operating cash flows.) Notes (1)

Rs. in ‘000

Analysis of cash and cash equivalents Cash on hand and balances with bank Bank overdraft

2015 32 (414)

Cash and cash equivalents

(382)

© Emile Woolf International

–––– ––––

119

2014 580 -

––––

580

––––

The Institute of Chartered Accountants of Pakistan

Financial accounting and reporting II

(2)

Material non-cash transactions During the year land was re-valued upwards by Rs.1million Rs. in ‘000

Workings (W1) Taxation paid Taxation creditor brought forward Taxation expense for period

400 650

––––––

1,050 (420)

Taxation creditor carried forward

––––––

Taxation paid in the year

630

––––––

(W2) Intangible assets Net book value brought forward Capitalised in the year (from (i))

200 300

––––

500 (200)

Amortisation charged in year (from (i))

––––

Intangibles acquired in the year

300

––––

(W3) Property, plant and equipment Cost brought forward Revaluation in year (from (ii)) Disposals (from (iii)) Additions (balancing figure)

3,000 1,000 (600) 1,600

––––––

Cost carried forward

5,000

––––––

(W4) Depreciation and amortisation Depreciation (150 movement + 500 on disposal) Amortisation Profit on disposal (W5) Charge shown in statement of profit or loss

650 200 (50)

––––

800

––––

Hence add back of depreciation and amortisation also takes account of the profit on disposal of the plant and machinery. (W5) Disposal Cost of disposal Accumulated depreciation

600 (500) ––––

Net book value Proceeds of sale

100 150

––––

Profit on sale

© Emile Woolf International

50

––––

120

The Institute of Chartered Accountants of Pakistan

Answers

3.2

STANDARD INC Statement of cash flows for the year ended 31 December 2015 Rs. in ‘000

Rs. in ‘000

Cash flows from operating activities Net profit before tax (W7) Adjustments for: Depreciation, loss on sale (W1-5) Interest receivable Interest and premium payable

64,000 20,000 (450) 8,400 ———— 91,950

Operating profit Increase in inventories Increase in receivables Increase in payables

(14,000) (1,200) 14,440 ———— 91,190 (6,840) (10,500) ————

Cash generated from operations Interest paid Tax paid (W6) Net cash from operating activities Cash flows from investing activities Acquisition of long-term investment Purchase of property plant and equipment Receipt from sale of long-term investment Interest received

73,850

(4,600) (69,000) 4,000 450 ————

Net cash used in investing activities

(69,150)

Cash flows from financing activities Proceeds from issuance of shares Redemption of long term loan Dividends paid

70,000 (42,000) (7,500) ————

Net cash used in financing activities

20,500 ———— 25,200 ————

Net increase in cash and cash equivalents WORKINGS (1)

Balance b/d Additions

© Emile Woolf International

Plant and machinery account – at cost Rs.000 120,000 39,000 ———— 159,000 ————

121

Disposals account Balance c/d

Rs.000 8,000 151,000 ———— 159,000 ————

The Institute of Chartered Accountants of Pakistan

Financial accounting and reporting II

Fixtures and fittings account – at cost

(2)

Rs.000 24,000 10,000 ——— 34,000 ———

Balance b/d Additions

Disposals account Balance c/d

Rs.000 5,000 29,000 ——— 34,000 ———

Fixed assets – additions summary Rs.000 Freehold property Rs.000(130,000 - 110,000) Plant and machinery Fixtures and fittings

(3)

Plant and machinery account – depreciation

Disposals account Balance c/d

(4)

Plant cost Fittings cost

Rs.000 6,000 54,000 ——— 60,000 ———

Rs.000 2,000 15,000 ——— 17,000 ———

Balance b/d Charge for year

Rs.000 13,000 4,000 ——— 17,000 ———

Fixed assets disposals account Rs.000 8,000 5,000

——— 13,000 ———

© Emile Woolf International

Balance b/d Charge for year

Rs.000 45,000 15,000 ——— 60,000 ———

Fixtures and fittings account – depreciation

Disposals account Balance c/d

(5)

20,000 39,000 10,000 ——— 69,000 ———

122

Plant depreciation Fittings depreciation Cash proceeds Plant Fittings Depreciation underprovided (bal fig)

Rs.000 6,000 2,000 3,000 1,000 1,000 ——— 13,000 ———

The Institute of Chartered Accountants of Pakistan

Answers

(6)

Tax account

Cash paid (bal fig) Balance c/f – corporation tax

(7)

Rs.000 10,500 33,000 ——— 43,500 ———

Rs.000 Balance b/f – corporation tax 21,500 I&E account – corporation tax 22,000 ——— 43,500 ———

Net profit before tax Note As profit before tax is required, reconstruct the statement of profit or loss up to this figure. Rs. in ‘000 Profit before tax Taxation – Corporation tax

64,000 (22,000) ——— 42,000 (15,000) ——— 27,000 14,000 ——— 41,000 ════

Dividends Retained profit for year Balance b/f Balance c/f

(8)

Cash and cash equivalents as shown in the statement of financial position Cash and cash equivalents consist of cash on hand and balances with banks. 2015

Cash at bank Bank overdraft

© Emile Woolf International

11,400 – ——— 11,400 ════

123

Rs. in ‘000 2014 Change in year 200 11,200 (14,000) 14,000 ——— ——— (13,800) 25,200 ════ ════

The Institute of Chartered Accountants of Pakistan

Financial accounting and reporting II

3.3

FALLEN Statement of cash flows for the year ended 31 December 2015 Rs. in ‘000

Cash flows from operating activities Net profit before tax Adjustments for: Depreciation, (W1-3) Interest payable

4,625 1,472 152 ——— 6,249 186 (894) (594) 324 ——— 5,271 (152) (1,775) ———

Operating profit Increase in deferred repairs provision Increase in inventories Increase in receivables Increase in payables Cash generated from operations Interest paid Tax paid (W5) Net cash from operating activities

3,344

Cash flows from investing activities Acquisition of long-term investment Purchase of property plant and equipment Receipt from sale of long-term investment

(198) (3,800) 168 ———

Net cash used in investing activities

(3,830)

Cash flows from financing activities Proceeds from issuance of shares (W6-7) Redemption of long term loan Dividends paid

792 (560) (544) ——

Net cash used in financing activities

(312) —— (798) ═══

Net increase in cash and cash equivalents

Rs. in ‘000

WORKINGS (1) Brought forward Additions

© Emile Woolf International

Leasehold premises (net) 5,700 1,300 ——— 7,000 ════

124

Depreciation (to balance) Carried forward

400 6,600 ——— 7,000 ════

The Institute of Chartered Accountants of Pakistan

Answers

(2) Brought forward Additions

Plant (net) 3,780 2,500

Disposals Depreciation (to balance) Carried forward

——— 6,280 ════ (3) Plant

Disposals 276

Cash Loss on sale (to balance)

—–— 276 ════ (4) Cash (to balance) Carried forward

(5) Cash (to balance) Carried forward DT CT

544 390 —–— 934 ════

Brought forward I&E account

1,775 202 1,730

Brought forward DT CT

138 2,038 1,531 ——— 3,707 ════

Share capital 2,280 ——— 2,280 ════

Brought forward Cash (to balance)

(7)

Share premium

Carried forward

2,112 ——— 2,112 ════

© Emile Woolf International

234 700 —–— 934 ════

Taxation

I&E account

Carried forward

168 108 —–— 276 ════

Dividends

——— 3,707 ════ (6)

276 964 5,040 ——— 6,280 ════

125

Brought forward Cash (to balance)

1,800 480 ——— 2,280 ════

1,800 312 ——— 2,112 ════

The Institute of Chartered Accountants of Pakistan

Financial accounting and reporting II

(8)

Long term loan

Cash (to balance) Carried forward

(9)

560 1,240 ——— 1,800 ———

Brought forward

1,800 ——— 1,800 ———

Analysis of the balances of cash and cash equivalents as shown in the statement of financial position Cash and cash equivalents consist of cash on hand and balances with banks. Rs. in ‘000

Cash at bank and in hand Bank overdrafts

3.4

2015

2014

Change in year

– (222) —— (222) ═══

576 – —— 576 ═══

(576) (222) —— (798) ═══

BIN QASIM MOTORS LIMITED Note: figures in brackets are in Rs.000 Bin Qasim Motors Limited Statement of cash flows for the year to 30 September 2015 Rs.000

Rs.000

Cash flows from operating activities Net profit before interest and tax (3,198 – 1,479) Adjustments for: Depreciation – buildings (W1)

1,719 80

– plant (W1)

276

Loss on disposal of plant (W1)

86

442 442

Amortisation of government grants (W2)

(125)

Negligence claim previously provided

(120)

Operating profit before working capital changes

1,916

Increase in inventories (1,046 – 785)

(261)

Increase in accounts receivable (935 – 824)

(111)

Decrease in accounts payable (760 – 644)

(116)

Cash generated from operations

1,428

Interest paid (260 + 25 – 40)

(245)

Income tax paid (W4) Dividends paid

(368) (180)

Net cash from operating activities

635

Cash flows from investing activities

© Emile Woolf International

126

The Institute of Chartered Accountants of Pakistan

Answers

Bin Qasim Motors Limited Statement of cash flows for the year to 30 September 2015 Purchase of land and buildings (W1) Purchase of plant (W1) Purchase of non-current investments Purchase of treasury bills (120 – 50) Proceeds of sale of plant (W1) Receipt of cash on servicing contracts (W2) Investment income Net cash used in investing activities Cash flows from financing activities Issue of ordinary shares (W3) Net decrease in cash and cash equivalents Cash and cash equivalents at the beginning of the period Cash and cash equivalents at the end of the period

Rs.000 (50) (848) (690) (70) 170 175 120

Rs.000

(1,193) 300 (258) 122 (136)

Workings (W1) Non-current assets Rs.000 Land and buildings – cost/valuation Balance b/f Revaluation surplus Balance c/f Difference cash purchase

1,800 150 (2,000) (50)

Plant – cost Balance b/f 1,220 Disposal (500) Balance c/f (1,568) Difference cash purchase (848) Depreciation of non-current assets: Building (760 – 680) 80 Plant (464 – (432 – 244)) 276 The plant had a carrying value of Rs.256,000 at the date of its disposal (500 cost – 244 depreciation). As there was a loss on sale of Rs.86,000 (given in question), the sale proceeds must have been Rs.170,000 (i.e. 256 – 86). (W2) Deferred income Balances b/f

– current – non-current

(125) (200)

Amortisation credited to cost of sales Balances c/f – current – non-current

100 275 375 175

Difference cash receipt

© Emile Woolf International

(325) 125

127

The Institute of Chartered Accountants of Pakistan

Financial accounting and reporting II

(W3) Share capital and convertible loan stock A reconciliation of share capital, share premium and the revaluation reserve shows the shares issued for cash: Share capital Rs.000 Opening balance

(1,000)

Share premium

Revaluatio n reserve

Rs.000

Rs.000

(60)

Revaluation of land

(40) (150)

Bonus issue 1 for 10

(100)

Conversion of loan stock (see below)

(100)

Closing balance

1,400 ––––– 200 ════

Difference issued for cash

100 (300) 460 ––––– 100 ════

90 ––––– nil ════

The 10% convertible loan stock had a carrying value of Rs.400,000 at the date of conversion to equity shares. This would be taken as the consideration for the shares issued which would be 100,000 Rs.1 shares (i.e. 400,000/100  25). This would increase issued share capital by Rs.100,000 and share premium by Rs.300,000. (W4) Income tax Rs.000 Tax provision b/f

(367)

Deferred tax b/f Statement of profit or loss tax charge

(400) (520)

Tax provision c/f

480

Deferred tax c/f

439

Difference cash paid

© Emile Woolf International

(368)

128

The Institute of Chartered Accountants of Pakistan

Answers

3.5

ITTEHAD MANUFACTURING LTD (a) Ittehad Manufacturing Ltd Statement of cash flows for the year to 30 September 2015 Rs.m Cash flows from operating activities Net profit before interest and tax Adjustments for: Amortisation – development expenditure (W1) Depreciation – property, plant and equipment Loss on sale of plant Increase in inventory (1,420 – 940) Increase in accounts receivable (990 – 680) Increase in accounts payable (875 – 730) Decrease in deferred income (260 – 300) Cash generated from operations Interest paid (30 – (15 – 5 accrual adjustments)) Income tax paid (W2) Net cash from operating activities Cash flows from investing activities Purchase of property, plant and equipment (W3) Capitalised development costs (W1) Proceeds of sale of plant (W3) Net cash from investing activities Cash flows from financing activities Issue of ordinary shares (W4) Issue of loan notes (300 – 100) Dividends paid Net cash generated from financing activities Net increase in cash and cash equivalents Cash and cash equivalents at beginning of period Cash and cash equivalents at end of period

Rs.m 920 130 320 50 (480) (310) 145 (40) 735 (20) (130) 585

(250) (500) 20 (730)

450 200 (320) 330 185 (115) 70

Workings (W1) Development expenditure Rs.m Opening balance

100

Amount capitalised

500

Closing balance

(470)

Amortisation: balancing figure

© Emile Woolf International

129

130

The Institute of Chartered Accountants of Pakistan

Financial accounting and reporting II

(W2) Income tax Opening balance: tax provision Opening balance: deferred tax Tax charged to statement of profit or loss Closing balance: tax provision Closing balance: deferred tax Tax paid (cash payments)

Rs.m Rs.m 160 140 300 270 (130) (310) (440) 130

(W3) Property, plant and equipment Opening balance Revaluation surplus Plant acquired Depreciation Closing balance Disposal at net book value – balancing figure Disposal of plant: Disposal at net book value (see above) Loss on sale (given in the question) Difference = Sale proceeds

Rs.m 1,830 200 250 (320) 1,960 1,890 70

70 (50) 20

(W4) Share capital Rs.m Opening balance, ordinary shares

500

Bonus issue 1 for 10 (from retained earnings)

(b)

50

Closing balance, ordinary shares

550 750

Difference: shares issued for cash (nominal value)

200

Plus increase in share premium (350 – 100)

250

Total cash proceeds of issue of ordinary shares

450

The cash flows generated from operations were Rs.685 million and are more than enough to pay the interest costs and taxation, but these cash flows are not as large as the equivalent profit figure. For most companies the operating cash flows are higher than the profit before interest and tax due to the effects of depreciation/amortisation charges (which are not cash flows). In the case of Ittehad Manufacturing Ltd the depreciation/amortisation effect has been more than offset by a much higher investment in working capital of Rs.645 million. Inventory has increased by over 50% and accounts receivable by 45%. This may be an indication of expanding activity, but it could also be an indication of poor inventory management policy and poor credit control, or even the presence of some obsolete inventory or unprovided bad accounts receivable.

© Emile Woolf International

130

The Institute of Chartered Accountants of Pakistan

Answers

A cause of concern is the size of the dividends, which seem high at Rs.320 million. This is a very high distribution ratio, and it seems odd that the company is returning such large amounts to shareholders at the same time as they are raising finance. Rs.450 million has been received from the issue of new shares and Rs.200 million from a further issue of loan notes. The company has invested considerably in new plant (Rs.250 million) and even more so in development expenditure (Rs.500 million). If management has properly applied the capitalisation criteria in IAS 38 Intangible Assets, then this indicates that they expect good future returns from the investment in new products or processes. The net investment in non-current assets is Rs.680 million which closely correlates to the proceeds from financing of Rs.650 million. In general it is acceptable to finance increases in the capacity of noncurrent assets by raising additional finance, however operating cash flows should finance replacement of consumed non-current assets.

3.6

WASEEM INDUSTRIES LIMITED Waseem Industries Limited Statement of cash flows for the year ended December 31, 2015 2015 Rs.m Cash flows from operating activities Profit before taxation Adjustments for: Depreciation

64

W1

17

Gain on sale of fixed assets Provision for gratuity Interest expense

(3) 10 16 104

Increase/decrease in working capital Increase in stocks-in-trade Increase in trade debts Decrease in advance, prepayments and other receivables Increase in trade and other payables Cash generated from operations Gratuity paid Interest paid Income taxes paid Net cash from operating activities

© Emile Woolf International

Workings

131

W2

(7) (13)

(55-48) (51-38)

6 22 8 112 (6) (18) (22) 66

W3 W4

W5 W6

The Institute of Chartered Accountants of Pakistan

Financial accounting and reporting II

Cash flows from investing activities Sale proceeds from sale of property, plant and equipment Purchase of property, plant and equipment Increase in capital work in progress Sale of long term investments Net cash used in investing activities Cash flows from financing activities Payment of long term finances Increase in short term finances *Dividend paid Net cash used in financing activities Net increase in cash and cash equivalents Cash and cash equivalent at the beginning of the year Cash and cash equivalent at the end of the year W1: Profit before taxation Unappropriated profit – closing Income tax expenses for the year 2015 Dividend (Rs. 125 million x 8%)

W2: Provision for gratuity Provision for gratuity: Closing Paid during the year 2015

W7 (20 -18) (75-100)

(21) 7 (10) (24)

W8 (13 - 6)

(9) 20 11

Rs.m 16 6 22 12 10

Less: Provision for gratuity - opening Provision for the year W3: Advances, prepayments and other receivables Advances, payments and other receivables – closing Advance Tax - closing Advances, payments and other receivables – opening Advance Tax – opening Decrease in advances, prepayments and other receivables

132

(100) (2) 25 (51)

Rs.m 85 19 10 114 (50) 64

Less: Unappropriated profit - opening

© Emile Woolf International

26

Rs.m 37 (10) 27 40 (7) 33 (6)

The Institute of Chartered Accountants of Pakistan

Answers

W4: Trade and other payables Trade and other payables – closing Accrued mark-up – closing

Rs.m 66 (7) 59

Trade and other payables – opening Accrued mark-up – opening

46 (9) 37 22

Increase in trade and other payables W5: Interest paid Accrued mark up – opening Expense for the year

Rs.m 9 16 25 (7) 18

Less: Accrued mark-up – closing Interest paid during the year W6: Income taxes paid Advances taxes – closing Provision for the year

Rs.m 10 19 29 (7) 22

Less: advance taxes - opening Income taxes paid during the year W7: Fixed assets purchase Closing fixed assets Depreciation for the year Carrying amount of disposed off assets

Rs.m 242 17 23 302 (182) 100

Less: opening fixed assets Purchase of fixed assets W8: Payment of long term finances Long term finance including current portion – closing (118 + 22) Long term finance including current portion – Opening (94 + 25) Payment during the year

© Emile Woolf International

133

Rs.m 140 (119) 21

The Institute of Chartered Accountants of Pakistan

Financial accounting and reporting II

3.7

JALIB INDUSTRIES LIMITED Jalib Industries Limited Statement of cash flow for the year ended December 31, 2015 Rs. in million CASH FLOW FROM OPERATING ACTIVITIES Net profit before tax Adjustments for: Depreciation Loss on sale of fixed assets Provision for gratuity Financial charges Bad debt expense Working Capital Changes Increase in creditors, accrued and other liabilities ([36.2 - 5] - [34.4 - 6]) Increase in stock in trade Increase in trade debts Decrease in advances and other receivables [(42-2.2)-(37.4-3.6)] Cash generated from operations Gratuity paid Income tax paid [3.6 + 104.6 - 2.2] Financial charges paid (6 + 10.5 - 5) Net cash from operating activities

191.40 27.70 4.60 15.50 10.50 1.20 250.90

2.80 (80.80) (5.00) 6.00 173.90 (4.40) (106.00) (11.50) 52.00

Working 3

Working 4

Working 2

CASH FLOW FROM INVESTING ACTIVITIES Capital expenditure incurred Proceeds on sale of fixed assets Net cash used in investing activities

(57.00) 10.40 (46.60)

Working 1

CASH FLOW FROM FINANCING ACTIVITIES Issue of share capital Repayment of long term loans (120 - 98) Dividend paid (1.4 + 75 - 3) Net cash from financing activities Net increase in cash and cash equivalents Cash and cash equivalent at the beginning of year Cash and cash equivalent at the end of year

99.00 (22.00) (73.40) 3.60 9.00 3.00 12.00

Working 5

WORKING 1 Rs. in million Capital expenditure incurred Book value of PPE - Closing Book value of CWIP - Closing Add: Book value of assets sold during the year Add: Depreciation for the year Less: Book value of PPE - Opening Less: Book value of CWIP - Opening

© Emile Woolf International

134

129.40 22.50 15.00 27.70 (100.60) (37.00) 57.00

The Institute of Chartered Accountants of Pakistan

Answers

WORKING 2 Rs. in million Gratuity paid during the year Opening balance Provision for gratuity

27.50 15.50 43.00 (38.60) 4.40

Less: Closing balance Gratuity paid during the year WORKING 3 Bad debts expense for the year Closing balance (28.5 ÷ 0.95) - 28.5 Less: Opening balance (24.7 ÷ 0.95) - 24.7 Add: Bad debts written off Bad debts expense for the year WORKING 4 Increase in trade debts Closing balance (28.5 ÷ 0.95) Less: Opening balance (24.7 / 0.95) Add: Bad debts written off WORKING 5 Issue of share capital Closing balance of paid up capital Closing balance of share premium Less: Opening balance of paid up capital Opening balance of share premium Issue of bonus shares (300 x 10%)

© Emile Woolf International

135

1.50 (1.30) 1.00 1.20

30.00 (26.00) 1.00 5.00

396.00 45.00 (300.00) (12.00) (30.00) 99.00

The Institute of Chartered Accountants of Pakistan

Financial accounting and reporting II

3.8

APOLLO INDUSTRY LIMITED Apollo Industry Limited Statement of cash flows for the year ended December 31, 2015 Rs. in ‘000 Cash used in operating activities Profit before taxation Adjustment for: (non cash items / separately disclosed items) Depreciation for the year (7,000-90-1,000) Amortization for the year (1140+50-1100) Provision for staff gratuity (1,400+300-1,190) Profit on sale of fixed assets (2,800-1,000) Mark-up on short term placement Operating profit before working capital changes Increase in working capital (12,125 – 15,700 + 4,200 – 6,250) Cash generated from operations Payment for staff gratuity Payment for taxation (950 + 4,660 – 800) Cash used in investing activities Capital expenditure incurred Proceeds from sale of PPE (1,200 + 1,800) Acquisition of intangible assets Mark-up received on short term placement Long term deposits (400-300)

Note 1

Cash used in financing activities Issue of ordinary share capital (25,000-2,000-20,000) Net decrease in cash and cash equivalents Opening balance: cash and cash equivalents

5,910 90 510 (1,800) (1,000) 10,210 (5,625) 4,585 (300) (4,810) (525) (13,110) 3,000 (50) 1,000 (100) (9,260) 3,000 (6,785) 7,225 440

Closing balance: cash and cash equivalents Note 1

6,500

Capital expenditure incurred: Opening book value for PPE Opening book value for CWIP Book value of assets sold during the year Depreciation for the year (7,000-901,000) Revaluation reserve adjustment Closing book value for PPE Closing book value for CWIP

Rs.000 25,500 10,000 (1,200) (5,910) (1,000) (35,000) (5,500) (13,110)

© Emile Woolf International

136

The Institute of Chartered Accountants of Pakistan

Answers

3.9

MARVEL ENGINEERING LIMITED Marvel Engineering Limited Cash Flow Statement For the year ended 30 June 2015 Workings Cash flows from operating activities Profit before taxation Adjustment for non-cash charges and other items: Depreciation Impairment of plant and machinery Financial charges Gain on sale of fixed assets Gain on sale of investments Dividend income Provision for gratuity payable (55 - 50 + 6) Working capital changes Decrease / (increase) in current assets: Increase in stock-in-trade (97 - 68) Increase in trade debts (see tutorial note) Other current assets (100 - 120) Increase / (decrease) in current liabilities: Trade and other payables ([73 - 7] - [56 - 3]) Cash generated from operations Financial charges paid (3 + 75 - 7) Income tax paid (5 + 21 + 21 - 12 - 15) Gratuity paid Net cash generated from operating activities Cash flows from investing activities Capital expenditure Proceeds from sale of property, plant and equipment (5+2) Proceeds from sale of investments (10+3) Purchase of long term investments (130-100+10) Dividend received Net cash used in investing activities Cash flows from financing activities Insurance of ordinary shares Proceeds from long term loan (330 - 110) Payment of dividend (2 + (440 × 5%) - 4) Net cash from financing activities Net decrease in cash and cash equivalents Cash and cash equivalent at the beginning of the year Cash and cash equivalent at the end of the year

© Emile Woolf International

137

2015 Rs.m 88.00 50.00 11.00 75.00 (2.00) (3.00) (30.00) 11.00

(29.00) (76.00) 20.00 13.00 128.00 (71.00) (20.00) (6.00) 31.00 1

(289.00) 7.00 13.00 (40.00) 30.00 (279.00)

2

40.00 220.00 (20.00) 240.00 (8.00) 39.00 31.00

The Institute of Chartered Accountants of Pakistan

Financial accounting and reporting II

W1: Capital expenditure Closing balance Add: Depreciation for the year Add: Impairment against plant Add: Disposal during the year Less: Opening balance

Rs.m 633.00 50.00 11.00 5.00 (410.00) 289.00

W2: Issuance of ordinary shares Closing balance of share capital Closing balance of share premium Less: Bonus shares issued (440 × 5%) Less: Opening balance of share capital

494.00 8.00 (22.00) (440.00) 40.00

Tutorial note: The original ICAP answer did not simply adjust for the movement in trade debts but added back the write off for bad debts (Rs. 6 million) and movement in the doubtful debt provision (Rs. 4 million) and then adjusted for the movement in trade debt before these write offs (Rs. 86 million). As the trade debt contains the credit for the write off and the profit for the year contains the debit it is easier to leave the expense in and adjust for the net movement. The following working was provided in the official answers. WORKINGS (All amount in million rupees) W1: Closing balance Add: Bad debts written off Less : Opening balance

© Emile Woolf International

(133 ÷ 0.95) - 133 (57÷ 0.95) - 57

138

Provision for bad debts 7.00 6.00 (3.00) 10.00

Trade debtors (133 ÷ 0.95) (57÷ 0.95)

140.00 6.00 (60.00) 86.00

The Institute of Chartered Accountants of Pakistan

Answers

CHAPTER 4 – CONSOLIDATED ACCOUNTS: STATEMENTS OF FINANCIAL POSITION– BASIC APPROACH 4.1

HALL Consolidated statement of financial position as at 31 December 2015 Rs.000 Assets Non-current assets Property, plant and equipment (35,000 + 20,000) Goodwill Current assets (16,000 + 14,000)

Equity and liabilities Capital and reserves Share capital Retained earnings (W5)

55,000 3,000 ———— 58,000 30,000 ———— 88,000 ———— 10,000 16,000 ———— 26,000

Non-controlling interest (W4) Long-term liabilities 8% Debenture loans (20,000 + 9,000) Current liabilities (20,000 + 9,000)

4,000 29,000 29,000 ———— 88,000 ————

WORKINGS (1)

Group structure Hall

75%

Stand

(2)

Net assets of Stand Reporting date Rs.000 4,000 12,000 16,000

Share capital Retained earnings

© Emile Woolf International

139

Post Date of acquisition acquisition Rs.000 4,000 8,000 4,000 12,000

The Institute of Chartered Accountants of Pakistan

Financial accounting and reporting II

(3)

Goodwill Rs.000 12,000 (9,000) ——— 3,000 ———

Cost of shares Less Net assets acquired (75%  12,000 (W2))

(4)

Non-controlling interest (25%  16,000 (W2))

Rs.000 4,000 ——–

(5)

Retained earnings Rs.000 13,000 3,000 ——– 16,000 ——–

Hall Inc Stand Inc (75%  4,000 (W2))

4.2

HASSLE Consolidated statement of financial position as at 31 December 2015 Sundry net assets (207,500 + 226,600)

Rs. 474,100 ———— 474,100 ————

Equity capital Retained earnings (W5)

Non-controlling interests (W4) Sundry liabilities (100,000 + 106,600)

120,000 123,500 ———— 243,500 24,000 206,600 ———— 474,100 ————

WORKINGS (1)

Group structure

Hassle

80%

Strife

© Emile Woolf International

140

The Institute of Chartered Accountants of Pakistan

Answers

(2)

Net assets of Strife Reporting date Rs. 50,000 70,000

Share capital Retained earnings

120,000 (3)

Goodwill

Non-controlling interest

24,000

Retained earnings

Rs.

Hassle Strife (80%  (70,000 − 50,000) (W2)) “Negative goodwill” (W3)

4.3

60,000 (80,000) ———— (20,000) ———— Rs.

20%  120,000) (W2) (5)

100,000 Rs.

Cost Net assets acquired (80%  100,000) (W2)

(4)

Post Date of acquisition acquisition Rs. 50,000 50,000 20,000

87,500 16,000 20,000 ———– 123,500 ———–

HYMN Consolidated statement of financial position as at 31 December 2015 Rs. Assets Non-current assets Property, plant and equipment Goodwill Current assets

170,000 29,000 275,000 ———— 474,000 ————

Equity and liabilities Shareholders’ equity Share capital Retained earnings (W5)

100,000 178,200 ———– 278,200 19,800 176,000 ———— 474,000 ————

Non-controlling interest (W4) Current liabilities

© Emile Woolf International

141

The Institute of Chartered Accountants of Pakistan

Financial accounting and reporting II

WORKINGS (1)

Group structure Hymn

80%

Psalm

(2)

Net assets of Psalm Reporting date Rs. 50,000 49,000

Share capital Retained earnings

99,000 (3)

Goodwill

Non-controlling interest

(56,000) ———— (29,000) ————

19,800

Retained earnings

Rs.

Hymn Psalm (80%  29,000 (W2))

© Emile Woolf International

85,000

Rs.

20%  99,000 (W2) (5)

70,000 Rs.

Cost of shares Net assets acquired Psalm Inc (80%  70,000) (W2)

(4)

Post Date of acquisition acquisition Rs. 50,000 20,000 29,000

155,000 23,200 ————— 178,200 —————

142

The Institute of Chartered Accountants of Pakistan

Answers

4.4

HANG Consolidated statement of financial position as at 31 December 2015 Rs. Assets Non-current assets Property, plant and equipment (240 + 180) Goodwill Current assets (250 + 196)

420,000 26,600 446,000 ————– 892,600 ————–

Equity and liabilities Shareholders’ equity Share capital Share premium account Retained earnings (W5)

200,000 25,000 198,000 ————– 423,000 87,600 382,000 ————– 892,600 ————–

Non-controlling interest (W4) Current liabilities (225 + 157)

WORKINGS (1)

Group structure Hang

60%

Swing

(2)

Net assets of Swing Inc 31 Dec 31 Dec 2015 2014 Rs. Rs. Ordinary shares of Rs.1 each Share premium account Retained earnings

© Emile Woolf International

90,000 90,000 49,000 49,000 80,000 50,000 ———– ———– 219,000 189,000 ———– ———–

143

The Institute of Chartered Accountants of Pakistan

Financial accounting and reporting II

Reporting date Rs. 90,000 49,000 80,000

Share capital Share premium Retained earnings

219,000 (3)

Goodwill

(5)

140,000 (113,400) ———— 26,600 ————

Non-controlling interest

Rs.

40%  219,000 (W2)

87,600

Retained earnings

Rs.

Hang Swing (60%  30,000 (W2))

4.5

189,000 Rs.

Cost Net assets acquired (60%  189,000) (W2)

(4)

Post Date of acquisition acquisition Rs. 90,000 49,000 50,000 30,000

180,000 18,000 ————– 198,000 ————–

HASH Consolidated statement of financial position as at 31 December 2015 Sundry net assets (207,500 + 226,600) Goodwill (W2)

Share capital Retained earnings (W5)

Non-controlling interests (W4) Sundry liabilities (100,000 + 106,600)

© Emile Woolf International

144

Rs.000 434,100 8,800 ———— 442,900 ———— 120,000 92,300 ———— 212,300 24,000 206,600 ———— 442,900 ————

The Institute of Chartered Accountants of Pakistan

Answers

WORKINGS (1)

Group structure Hash

80%

Stash

(2)

Net assets of Stash Reporting date Rs.000 50,000

Share capital Retained earnings: At the start of the year (70,000 – 24,000) Profit for the first 9m (24,000 ×9/12)

(3)

46,000

70,000

18,000 64,000

120,000

114,000

Goodwill

(5)

6,000

Rs.000

Cost Net assets acquired (80%  114,000) (W2)

(4)

Post Date of acquisition acquisition Rs.000 50,000

Non-controlling interest

100,000 (91,200) ——— 8,800 ——— Rs.000

20%  120,000) (W2)

24,000

Retained earnings

Rs.000

Hash Stash (80%  (70,000 − 64,000) (W2))

87,500 4,800 ———– 92,300 ———–

© Emile Woolf International

145

The Institute of Chartered Accountants of Pakistan

Financial accounting and reporting II

CHAPTER 5 – CONSOLIDATED ACCOUNTS: STATEMENTS OF FINANCIAL POSITION– COMPLICATIONS 5.1

HAIL Consolidated statement of financial position as at 31 December 2015 Rs.000 Rs.000 Assets Non-current assets Property, plant and equipment Investments (68,000 – 65,000) Goodwill (W3) Current assets Cash at bank and in hand Trade receivables Inventories

246,000 3,000 6,500 39,900 138,300 92,400 ———– 526,100 ———–

Equity and liabilities Capital and reserves Share capital Capital reserve (W6) Retained earnings (W5)

100,000 18,000 210,480 ———– 328,480 11,420

Non-controlling interest (W4) Current liabilities Trade payables Proposed dividend

183,000 – parent company – non controlling interest

3,000 200 ———–

3,200 ———– 526,100 ———–

WORKINGS (1)

Group structure Hail

90%

Snow

© Emile Woolf International

146

The Institute of Chartered Accountants of Pakistan

Answers

(2)

Net assets of Snow Reporting date Rs.000 50,000 5,000 20,000

Share capital Share premium account Revaluation reserve Retained earnings Per question Proposed dividend

(3)

41,200 (2,000) 39,200

10,000

114,200

65,000

Rs.000 11,420 ————

Retained earnings Hail Proposed dividend Dividend receivable from Snow Snow (90%  29,200 (W2))

(6)

Rs.000 65,000 (58,500) ———— 6,500 ————

Non-controlling interest 10%  114,200 (W2)

(5)

29,200

Goodwill Cost of shares Net assets acquired (90%  65,000) (W2)

(4)

Post Date of acquisition acquisition Rs.000 50,000 5,000 

Rs.000 185,400 (3,000) 1,800 26,280 ———— 210,480 ————

Capital reserve Rs.000 18,000 ————

Snow (90%  20,000 (W2))

© Emile Woolf International

147

The Institute of Chartered Accountants of Pakistan

Financial accounting and reporting II

5.2

HAIRY Consolidated statement of financial position as at 31 December 2015 Rs.000 Assets Non-current assets Property, plant and equipment

180,000

Current assets Cash at bank and in hand Investments Receivables Inventory (17,000 + 11,000 – 800)

Equity and liabilities Capital and reserves Share capital Share premium account Capital reserve Retained earnings (W5)

15,500 3,000 91,700 27,200 ———– 317,400 ———–

100,000 20,000 23,000 102,900 ———– 245,900 16,500

Non-controlling interest (W4) Current liabilities

55,000 ———– 317,400 ———–

WORKINGS (1)

Group structure

Hairy

80%

Spider

© Emile Woolf International

148

The Institute of Chartered Accountants of Pakistan

Answers

(2)

Net assets of Spider Reporting date Rs.000 Share capital Share premium account Retained earnings Per question Unrealised profit

(3)

(4)

(5)

Post Date of acquisition acquisition Rs.000

60,000 16,000

60,000 16,000

7,300 (800) 6,500

2,300

82,500

78,300

4,200

Goodwill

Rs.000

Cost of shares Less Net assets acquired (80%  78,300 (W2))

55,000 (62,640) ———— (7,640) ————

Non-controlling interest

Rs.000

Share of net assets (20%  82,500 (W2))

16,500 ————

Retained earnings

Rs.000

Hairy Spider (80%  4,200 (W2)) Negative goodwill (W4)

91,900 3,360 7,640 ———— 102,900 ————

© Emile Woolf International

149

The Institute of Chartered Accountants of Pakistan

Financial accounting and reporting II

5.3

HARD Consolidated statement of financial position as at 31 December 2015 Rs.000 Assets Non-current assets Property, plant and equipment (225 + 175 – 17.5 (W6)) Goodwill (W3) Current assets (271 + 157)

382,500 14,000 428,000 ———– 824,500 ———–

Equity and liabilities Shareholders’ equity Share capital Share premium account Retained earnings (W5)

100,000 15,000 260,500 ———– 375,500 76,000 373,000 ———– 824,500 ———–

Non-controlling interest (W4) Current liabilities

WORKINGS (1)

Group structure Hard

60%

Soft

(2)

Net assets of Soft Inc

Share capital Share premium account Retained earnings

© Emile Woolf International

150

31 Dec 2015 Rs.000

31 Dec 2014 Rs.000

100,000 10,000 80,000

100,000 10,000 50,000

190,000

160,000

Post acquisition

30,000

The Institute of Chartered Accountants of Pakistan

Answers

(3)

Goodwill

Rs.000

Cost

110,000 Net assets acquired 60%  160,000 (W2)

(4)

(96,000) ———— 14,000 ————

Non-controlling interest

Rs.000

40%  190,000 (W2) (5)

76,000

Retained earnings Hard Less

Rs.000

Adjustment re intra group transfer

Soft (60%  (80,000 – 50,000 (W2))

(6)

260,000 (17,500) ———— 242,500 18,000 ———— 260,500 ————

PURP on non current assets IS

Rs.000 50,000 (12,500) ———— 37,500 ————

Cost Accumulated depreciation

SHOULD BE Cost Accumulated depreciation

100,000 (80,000) ———— 20,000 ————

Dr Retained earnings Cr Non current assets

© Emile Woolf International

151

17,500 17,500

The Institute of Chartered Accountants of Pakistan

Financial accounting and reporting II

5.4

HALE (a)

Consolidated statement of financial position as at 31 December 2015 Rs.000 Assets Non-current assets Property, plant and equipment (152,000 + 129,600 + 28,000 (W2)) Goodwill (W3) Current assets Bank (41,000 + 8,000) Receivables (104,000 + 84,000) Inventory (112,000 + 74,400 – 3,200 (W6))

309,600 61,400 49,000 188,000 183,200 ————– 791,200 ————–

Equity and liabilities Capital and reserves Share capital Retained earnings (W5)

100,000 555,200 ————– 655,200 60,000 76,000 ————– 791,200 ————–

Non-controlling interest (W3) Current liabilities (52,000 + 24,000)

WORKINGS (1)

Group structure

Hale

128 160

= 80% ord ordords ords

Sowen

(2)

Net assets of Sowen Reporting date Rs.000 160,000

Share capital Fair value adjustment on non-current assets Retained earnings

© Emile Woolf International

152

Post Date of acquisition acquisition Rs.000 160,000

28,000 112,000

28,000 (11,000)

300,000

177,000

123,000

The Institute of Chartered Accountants of Pakistan

Answers

(3)

Goodwill Rs.000 203,000 (141,600) ————– 61,400 ————–

Cost of shares Less Net assets acquired (80%  177,000 (W2))

(4)

Non-controlling interest Share of net assets (20%  300,000 (W2))

(5)

Rs.000 60,000 ————

Retained earnings Rs.000 460,000 (3,200) 98,400 ————– 555,200 ————–

Hale PURP (W6) Sowen (80%  123,000 (W2))

(6)

Unrealised profits % 125 (100) —— 25 ——

SP Cost GP

5.5

Rs.000 16,000 (12,800) ——— 3,200 ———

HELLO Consolidated statement of financial position as at 31 December 2015 Rs. Assets Non-current assets Property, plant and equipment (225 + 175 + 10 – 2) Goodwill (W3) Current assets (271 + 157)

428,000 ———– 844,000 ———–

Equity and liabilities Shareholders’ equity Called up share capital Retained earnings (W5)

100,000 291,800 ———– 391,800 79,200 373,000 ———– 844,000 ———–

Non-controlling interest (W4) Current liabilities

© Emile Woolf International

408,000 8,000

153

The Institute of Chartered Accountants of Pakistan

Financial accounting and reporting II

WORKINGS (1)

Group structure Hello

60%

Solong

(2)

Net assets of Solong Inc

Share capital Retained earnings Per the question Less: Fair value adjustment for depreciation (2/10 × 10,000) Fair value adjustment

(3)

Reporting date Rs. 100,000 90,000 (2,000) 88.000 10,000

60,000 10,000

198,000

170,000

Goodwill

Rs.

Cost

(5)

Rs. 110,000

Net assets acquired 60%  170,000 (W2)

(4)

Post Date of acquisition acquisition Rs. 100,000

(102,000) ———— 8,000 ————

Non-controlling interest

Rs.

40%  198,000 (W2)

79,200

Retained earnings

Rs.

Hello Solong (60%  (88,000 – 60,000 (W2))

© Emile Woolf International

154

275,000 16,800 ———– 291,800 ———–

The Institute of Chartered Accountants of Pakistan

Answers

5.6

HASAN LIMITED Hasan Limited Consolidated statement of financial position as at 31 March 2015 Rs.000 Rs.000 Assets Non-current assets Property, plant and equipment (W1) Goodwill (W4) Software (W1) Investments (65 + 210)

4,020 480 1,440 275 –––––––––––––

Current assets Inventories (W2) Trade receivables (524 + 328) Cash and bank (20 + 55 cash in transit)

6,215

1,274 852 75 –––––––––––––

2,201 –––––––––––––

Total assets

8,416 –––––––––––––

Equity and liabilities Capital and reserves Equity capital Reserves Share premium Retained earnings (W3)

2,000 2,000 2,420 –––––––––––––

4,420 –––––––––––––

6,420 350

Non-controlling interest (W5) Non-current liabilities Government grants (230 + 40) Current liabilities Trade payables (475 + 472) Operating overdraft Income tax liability (228 + 174)

270 947 27 402 –––––––––––––

1,376 –––––––––––––

Total equity and liabilities

8,416 –––––––––––––

© Emile Woolf International

155

The Institute of Chartered Accountants of Pakistan

Financial accounting and reporting II

Workings (W1) Property, plant and equipment Rs.000 Balance from question – Hasan Limited

2,120

Balance from question – Shakeel Limited

1,990

Fair value adjustment on acquisition (see below)

(120)

Over-depreciation re fair value adjustment year to 31 March 2015

30 –––––––––––––

4,020 –––––––––––––

A fair value of the leasehold based on the present value of the future rentals (receivable in advance) would be the next (non-discounted) payment of the rental plus the final three years as an annuity at 10%: Rs.000 PV of rental receipts: Rs.80,000 + (Rs.80,000  2.50)

280

Carrying value on acquisition is

(400) –––––––––––––

Fair value reduction of leasehold

(120) –––––––––––––

The depreciation of the leasehold in Shakeel Limited’s accounts would be Rs.100,000 per annum. However in the consolidated accounts it should be Rs.70,000 (Rs.280,000/4). This would require a reduction in depreciation of Rs.30,000 in the consolidated accounts for the next four years. Software: Shakeel Limited’s accounts Capitalised amount Depreciation to 31 March 2014 Value at date of acquisition Depreciation to 31 March 2015 Carrying value 31 March 2015

Consolidated figures

Rs.000

Rs.000

2,400

2,400

(300)

–––––

8 year life

2,100

(480)

––––– 1,920

(300)

(480)

–––––

–––––

1,800

Difference

5 year life 180 fair value adjustment 180 additional amortisation

1,440

–––––

–––––

(W2) Inventories Rs.000 Amounts given in the question (719 + 560) Unrealised profit in inventories (25  25/125)

1,279 (5) –––––––––––––

1,274 –––––––––––––

© Emile Woolf International

156

The Institute of Chartered Accountants of Pakistan

Answers

(W3) Retained earnings Rs.000 Retained profits of Shakeel Limited, 31 March 2015

1,955

Adjustments: Excess charge for leasehold depreciation Insufficient charge for Software amortisation Unrealised profit in inventory (W2)

30 (180) (5) –––––––––––––

Adjusted retained profits at 31 March 2015 Retained earnings of Shakeel Limited at 1 April 2014

1,800 2,200 –––––––––––––

Shakeel Limited: loss for the year (post-acquisition loss)

(400) –––––––––––––

Rs.000 Parent company share of post-acquisition loss (90%) Hasan Limited reserves at 31 March 2015 Goodwill impairment

(360) 2,900 (120) –––––––––––––

Consolidated retained profits at 31 March 2015

2,420 –––––––––––––

(W4) Goodwill Rs.000 At acquisition date Shares of Shakeel Limited

1,500

Share premium of Shakeel Limited Retained earnings of Shakeel Limited Fair value adjustments: Leasehold (W1)

500 2,200 (120)

Software (W1)

(180) –––––––––––––

3,900 –––––––––––––

Acquired by Hasan Limited (90%) Cost of investment

3,510 4,110 –––––––––––––

Goodwill at acquisition

600

Impairment

120 –––––––––––––

Goodwill at 31 March 2015

480 –––––––––––––

© Emile Woolf International

157

The Institute of Chartered Accountants of Pakistan

Financial accounting and reporting II

(W5) Non-controlling interests Rs.000 Share capital of Shakeel Limited

1,500

Share premium of Shakeel Limited

500

Adjusted retained earnings of Shakeel Limited, 31 March 2015 (W3) Fair value adjustments: Leasehold Software

1,800

(120) (180) –––––––––––––

Total net assets at 31 March 2015

3,500 –––––––––––––

Non-controlling interests (10%)

350 –––––––––––––

(W6) Elimination of current accounts: Rs.000 Shakeel Limited’s current account with Hasan Limited per question Deduct cash in transit regarding this balance

75 (15) –––––––––––––

Adjusted figure to cancel

60 –––––––––––––

(W7) Elimination of intra-group loan: Rs.000 Investment in Hasan Limited’s books

200

Deduct repayment in transit

(40) –––––––––––––

Non-current liability in Shakeel Limited’s books

160 –––––––––––––

© Emile Woolf International

158

The Institute of Chartered Accountants of Pakistan

Answers

CHAPTER 6 – CONSOLIDATED ACCOUNTS: STATEMENTS OF COMPREHENSIVE INCOME 6.1

HARRY Consolidated statement of profit or loss for the year ended 31 December 2015 Rs.000 Revenue Cost of sales

1,410 (733) ——– 677 (90) (100) ——– 487 9 (22) ——– 474 (165) ——– 309 (15) ——– 294 ——–

Gross profit Distribution costs Administrative expenses Operating profit Investment income Finance costs Profit before tax Income tax expense Profit after tax Non-controlling interest (W3) Profit

Movement on consolidated retained earnings for the year ended 31 December 2015 Retained earnings at 1 January 2014 (W4) Retained earnings for the year Dividends Retained earnings at 31 December 2015 (W5)

127 294 (50) ——– 371 ——–

WORKINGS (1)

Group structure

Harry

75%

Sally

© Emile Woolf International

159

The Institute of Chartered Accountants of Pakistan

Financial accounting and reporting II

(2)

Consolidated statement of profit or loss

Revenue C of S

– per Q – PURP Distribution costs Administrative expenses Investment income (20 – 15) Interest payable Tax PAT (3)

Harry Rs.000

Sally Adj Rs.000 Rs.000

Consol Rs.000

1,120 (610) (3) (50) (55) 5 (18) (140)

390 (220) – (40) (45) 4 (4) (25) —– 60 —–

1,410

(100) 100 –

(733) (90) (100) 9 (22) (165)

Non-controlling interest Rs.000

25%  60,000 (W1) or as per PAT in question

15 ——

(4)

Reserves brought forward Rs.000 Harry Sally (75%  (45 – 9))

(5)

100 27 —— 127 ——

Reserves carried forward (proof) Rs.000 Harry PURP Sally (75%  (85 – 9))

(6)

317 (3) 57 —— 371 ——

Inter-company dividend Rs.000 Payable by Sally Receivable by Harry (75%  20)

© Emile Woolf International

160

20 —– 15 —–

The Institute of Chartered Accountants of Pakistan

Answers

6.2

HORNY Consolidated statement of profit or loss for the year ended 31 December 2015 Rs.000 Revenue Cost of sales

362,000 (169,050) ———– 192,950 (93,817) ———– 99,133 13,100 3,800 ———– 116,033 (48,400) ———– 67,633 (2,996) ———– 64,637 ———–

Gross profit Operating costs Operating profit Investment income Negative goodwill Profit before tax Income tax Profit after tax Non-controlling interest (W3) Profit Movement on consolidated retained earnings for the year ended 31 December 2015

Rs.000 Retained earnings at 1 January 2015 Retained profit for the year Dividend Retained earnings at 31 December 2015

80,200 64,637 (20,000) ———– 124,837 ———–

WORKINGS (1)

Group structure Horny

75% (acq 31 August 2005)

Smooth

© Emile Woolf International

161

The Institute of Chartered Accountants of Pakistan

Financial accounting and reporting II

(2)

Consolidation schedule Horny Smooth Adj Consol 4 12 Rs.000 Rs.000 Rs.000 Rs.000 Revenue Cost of sales

304,900 65,100 (8,000) 362,000 (144,200) (32,850) 8,000 (169,050)

Operating costs Investment income of H of S (all of it)

(76,450) (17,367)

Tax

(42,900)

(93,817)

10,500

PAT

2,600

13,100

(5,500) ——— 11,983

(48,400)

(3)

Non-controlling interest

@ 25% ——– = 2,996 ——–

(4)

Consolidated retained earnings carried forward - proof Rs.000 Horny Simpson (11,983 – 2,996) Negative goodwill

© Emile Woolf International

112,050 8,987 3,800 ——— 124,837 ———

162

The Institute of Chartered Accountants of Pakistan

Answers

6.3

HERON Consolidated statement of financial position as at 30 June 2015 Rs.000 Assets Non-current assets Property, plant and equipment (31,000 + 15,000) Current assets (23,000 + 11,000)

34,000 ——— 80,000 ════

Equity and liabilities Shareholders’ equity Share capital Share premium account Retained earnings (20,000 + (

46,000

10,000 5,000 2  18,500)) 3

32,333 ——— 47,333

1 Non-controlling interest (3  20,000)

6,667

Non-current liabilities Current liabilities (5,000 + 6,000)

15,000 11,000 ——— 80,000 ════

Consolidated statement of profit or loss for the year ended 30 June 2015 Rs.000 Revenue (30,000 + 25,000) Cost of sales (9,000 + 10,000)

55,000 (19,000) ——— 36,000 (4,200) (3,800) (2,000) ——— 26,000 (6,000) ———

Gross profit Distribution costs (3,000 + 1,200) Administrative expenses (1,000 + 2,800) Finance costs Profit before tax Income tax expense (3,000 + 3,000) Profit for the period 20,000 1 Non-controlling interest (3  8,000)

(2,667) ——— Profit for the financial year attributable to the members of Heron Inc 17,333 ——— Consolidated statement of changes in equity for the year ended 30 June 2015 (extract) 2 Retained earnings brought forward (8,000 + (3  10,500)) Profit for the financial year attributable to the members of Heron Inc Retained earnings carried forward

© Emile Woolf International

163

15,000 17,333 ——— 32,333 ———

The Institute of Chartered Accountants of Pakistan

Financial accounting and reporting II

6.4

HANKS Consolidated statement of financial position as at 31 December 2015 Rs.000 Rs.000 Assets Non-current assets Property, plant and equipment (32,000 + 25,000 + 20,000 + 6,000) Goodwill

83,000 4,500 ———– 87,500

Current assets Cash at bank and in hand (9,500 + 2,000 + 4,000) Receivables (20,000 + 8,000 + 17,000) Inventory (30,000 + 18,000 + 18,000 – 2,100)

15,500 45,000 63,900 ———– 124,400 ———– 211,900 ═════

Total assets Equity and liabilities Share capital Share premium account Retained earnings (W5)

40,000 6,500 88,300 ———– 134,800

Non-controlling interest (W4)

28,100

Current liabilities Trade payables (23,500 + 6,000 + 17,000) 46,500 Proposed dividends – to minority shareholders (2,500 – 2,000) 500 – to Hanks’s shareholders 2,000 ———– Total equity and liabilities

49,000 ———– 211,900 ═════

Consolidated statement of profit or loss for the year ended 31 December 2015 Rs.000 Revenue (W6) Cost of sales (W6)

310,000 (159,100) ———– 150,900 (51,000) (29,500) ———– 70,400 (24,000) ———– 46,400 (9,200) ———– 37,200 ═════

Gross profit Distribution costs (W6) Administrative expenses (W6) Profit before taxation Tax (W6) Profit after taxation Non-controlling interest (W6) Profit

© Emile Woolf International

164

The Institute of Chartered Accountants of Pakistan

Answers

Statement of movements on reserves for the year ended 31 December 2015 Share premium account Rs.000

Share Capital At 1 January 2015 Profit for the year Dividends (proposed) At 31 December 2015

40,000

6,500 –

——–— 40,000 ═════

–––––– 6,500 ═════

Retained earnings Rs.000

Total Rs.000

53,100 (W7) 99,600 37,200 37,200 (2,000) (2,000) ––––––– ––––––– 88,300 134,800 ═════ ═════

WORKINGS (1)

Group structure Hanks 80%

Streep

(2)

60%

Scott

Net assets Streep Reporting date Rs.000 10,000

Share capital Retained earnings Per question Proposed dividend

Post Date of acquisition acquisition Rs.000 10,000

37,000 (2,500) 34,500

7,500

44,500

17,500

27,000

Scott Reporting date Rs.000 15,000 27,000 6,000

Share capital Retained earnings Revaluation reserve

48,000

© Emile Woolf International

165

Post Date of acquisition acquisition Rs.000 15,000 3,000 24,000 6,000 24,000

The Institute of Chartered Accountants of Pakistan

Financial accounting and reporting II

(3)

Goodwill on Streep Rs.000 20,500 (14,000) ——— 6,500 ———

Cost of shares Net assets acquired (80%  17,500) (W2)

Of which: Written off by start of the year (6,500 – 5,000) Written off by end of the year (6,500 – 4,500) Recognised as impairment during the year (balancing figure)

1,500 2,000 ——— 500 ———

Goodwill on Scott Rs.000 13,000 (14,400) ——— (1,400) ———

Cost of shares Net assets acquired (60%  24,000 (W2))

(4)

Non-controlling interest Rs.000 8,900 19,200 ——— 28,100 ———

Streep (20%  44,500 (W2)) Scott (40%  48,000 (W2))

(5)

Consolidated retained earnings c/f Rs.000 Hanks Dividend receivable from Streep (80% of 2,500) Proposed dividend Streep (80%  27,000 (W2)) Scott (60%  24,000 (W2)) 30 PURP ((5,200 + 3,900)  130 ) Goodwill impairment – Streep Negative goodwill – Scott

© Emile Woolf International

166

55,000 2,000 (2,000) 21,600 14,400 (2,100) (2,000) 1,400 ——— 88,300 ———

The Institute of Chartered Accountants of Pakistan

Answers

(6)

Consolidation schedule Hanks Streep Rs.000 Rs.000 Sales revenue C of S – per Q – PURP (W5) Distrib (51,000) Admin (29,500) Tax PAT Non-controlling interest in profit after tax

(7)

Scott Adj Consol Rs.000 Rs.000 Rs.000

125,000 117,000 82,000 (14,000) 310,000 (65,000) (64,000) (42,000) 14,000 (2,100) (159,100) (21,000) (14,000) (16,000) (14,000)

(8,000) (7,000)

(10,000) (9,000) (5,000) ——— ——— 22,000 12,000 @20% ——– 4,400 + ——–

@40% ——– 4,800 ——–

(500) (24,000)

=

9,200 ——–

Consolidated retained earnings b/f Hanks Share of post acquisition profits of Streep (80% (15,000 – 7,500)) Share of post acquisition profits of Scott (60% (15,000 – 3,000)) Goodwill impairment - Streep Negative goodwill credited

© Emile Woolf International

167

Rs.000 40,000 6,000 7,200 (1,500) 1,400 ——— 53,100 ———

The Institute of Chartered Accountants of Pakistan

Financial accounting and reporting II

CHAPTER 7 – TANGIBLE NON-CURRENT ASSETS (IAS 16: PROPERTY, PLANT AND EQUIPMENT AND IAS 23: BORROWING COSTS) 7.1

ROONEY (a)

Borrowing costs IAS 23 should be applied in accounting for borrowing costs. Borrowing costs are recognised as an expense in the period in which they are incurred unless they are capitalised in accordance with IAS 23 which says that borrowing costs that are directly attributable to the acquisition, construction or production of a qualifying asset can be capitalised as part of the cost of that asset. 

A qualifying asset is an asset that necessarily takes a substantial period of time to get ready for its intended use or sale.



Borrowing costs that are directly attributable to acquisition, construction or production are taken to mean those borrowing costs that would have been avoided if the expenditure on the qualifying asset had not been made.

When an enterprise borrows specifically for the purpose of funding an asset, the identification of the borrowing costs presents no problem as the amount capitalised is the actual borrowing costs net of any income earned on the temporary investment of those borrowings. If funds are borrowed, generally, the amount of borrowing costs eligible for capitalisation is determined by applying a capitalisation rate to the expenditures on that asset calculated as the weighted average of the borrowing costs applicable to general borrowings. IAS 23 also contains rules on commencement of capitalisation, suspension of capitalisation and cessation of capitalisation. Amount capitalised

Rs.000

Cost of manufacture Interest capitalised (Rs.20m × 5% × 2 years)

28,000 2,000

–––––––

30,000

–––––––

(b)

Accounting Rule IAS 16 requires that each part of an item (that has a cost that is significant in relation to the total cost) is depreciated separately. Therefore the cost recognised at initial recognition must be allocated to each part accordingly.

© Emile Woolf International

168

The Institute of Chartered Accountants of Pakistan

Answers

Accounting 31st March 2016

(i)

Hydraulic system “Frame”

Carrying value 1.4.2015

Depreciation

Carrying value 31.3.2016

Rs.000 9,000 21,000

Rs.000 3,000 2,625

Rs.000 6,000 18,375

30,000

5,625

–––––– ––––––

–––––– ––––––

––––––

24,375 ––––––

Revaluation loss (to profit and loss)

(3,375)

Fair value.

21,000

–––––– ––––––

The carrying value of the assets should be written down by a factor of 21,000/24,375. This gives a carrying value for the hydraulic system (in Rs.000) of 5,169 and for the ‘frame’ 15,831. The hydraulic plant should be depreciated over two more years and the ‘frame’ over 7 more years. 31st March 2017

(ii)

Carrying Depreciation value charge 1.4.2016 Hydraulic system “Frame”

Carrying value 31.3.2017

Rs.000 5,169 15,831

Rs.000 2,585 2,262

Rs.000 2,584 13,569

21,000

4,847

16,153 19,600

–––––––

–––––––

Revalued amount

––––––– –––––––

Total gain

3,447

–––––––

To statement of profit or loss Other comprehensive income

3,375 72 ––––––– 19,600 –––––––

Fair value

The total revaluation gain is 3,447. Of this total amount, 3,375 reverses the loss in the previous year and is therefore reported in profit and loss for the year. The remaining 72 is reported as other comprehensive income. (Tutorial note: Deferred tax is ignored by this question.)

© Emile Woolf International

169

The Institute of Chartered Accountants of Pakistan

Financial accounting and reporting II

7.2

EHTISHAM IAS 16 permits assets to be carried at cost or revaluation. Where the latter is chosen, the asset must be stated at its fair value. The original depreciation was Rs. 40,000 (Rs. 1,000,000/25 years) per annum. On 31st March 2014 the asset is two years old. Its carrying value before revaluation was therefore Rs.1million less accumulated depreciation of Rs.80,000 (2/25 × Rs. 1 million). Rs. 1,000,000

Cost/valuation Accumulated depreciation

(80,000)

Net book value

920,000

In order to effect the revaluation, the cost is uplifted to fair value of Rs.1.15m, the accumulated depreciation is eliminated, and the uplift to the net book value is credited to a revaluation surplus account. Debit 150,000 80,000

Cost/valuation Accumulated depreciation

Credit

230,000

Revaluation surplus The impact of the journal is as follows: Before 1,000,000

Cost/valuation Accumulated depreciation

(80,000)

Net book value

920,000

Adjustment 150,000

After 1,150,000

80,000

nil 1,150,000

The asset is depreciated over its remaining useful economic life of 23 years giving a charge of Rs. 50,000 (Rs. 1,150,000/23 years) per annum in the year to 31st March 2015. Debit 50,000

Statement of profit or loss

Credit 50,000

Accumulated depreciation This results in a carrying value as at 31st March 2015 of: Rs. 1,150,000

Cost/valuation

(50,000)

Accumulated depreciation

1,100,000

Net book value

Transfer from revaluation surplus to retained earnings As a result of the revaluation, the annual depreciation has increased from Rs.40,000 to Rs.50,000. This extra depreciation of Rs.10,000 is transferred from the revaluation reserve to accumulated profits each year. Debit 10,000

Revaluation surplus

10,000

Accumulated profits

© Emile Woolf International

Credit

170

The Institute of Chartered Accountants of Pakistan

Answers

By the 31st March 2015, the balance remaining on the revaluation reserve will be Rs.220,000. Surplus recognised at 31 March 2014

Rs. 230,000

Transfer to accumulated profits

(10,000)

Net book value

220,000

The fall in property values at the year-end. The asset must be revalued downwards to Rs.0.8million, a write-down of Rs.300,000. Rs.220,000 of this is charged against the revaluation reserve relating to this asset, and the remaining Rs.80,000 must be charged against profits. The reduction of the carrying amount of the asset is achieved by removing the accumulated depreciation and adjusting the asset account by the balance. Debit 220,000

Revaluation surplus Statement of profit or loss

Credit

80,000 350,000

Asset at valuation Accumulated depreciation

50,000

The impact of the journal is as follows: Before 1,150,000

Cost/valuation

(50,000)

Accumulated depreciation

1,100,000

Net book value

Adjustment 350,000 50,000

After 800,000 nil 800,000

This balance is depreciated over the remaining useful life of the asset (22 years).

© Emile Woolf International

171

The Institute of Chartered Accountants of Pakistan

Financial accounting and reporting II

7.3

CARLY Financial statements for the year ended 31 December 2015 (extract) Property, plant and equipment

Cost/valuation At 1 January 2015 Revaluation Additions (W2) Disposals At 31 December 2015 Accumulated depreciation At 1 January 2015 Charge for the year (W1) Revaluation Disposals At 31 December 2015 Carrying amount At 31 December 2014 At 31 December 2015

Land and buildings Rs.

Plant and machinery Rs.

1,500,000 250,000 -

340,500 17,550 (80,000)

Computer equipment Rs.

Total Rs.

617,800 -

2,458,300 250,000 17,550 (80,000)

––––––––––

––––––––

––––––––

1,750,000

278,050

617,800

––––––––––

600,000 20,000 (620,000) -

––––––––

––––––––––

125,900 51,191 (57,000)

505,800 44,800 -

1,231,700 115,991 (620,000) (57,000)

––––––––

––––––––

nil

120,091

550,600

900,000

––––––––––

1,750,000

––––––––––

2,645,850

––––––––

–––––––––– ––––––––––

––––––––––

––––––––

214,600

––––––––

157,959

––––––––

––––––––

112,000

––––––––

67,200

––––––––

––––––––––

670,691

––––––––––

1,226,600

––––––––––

1,975,159

––––––––––

Workings (1) Depreciation charges Buildings = (1,500,000 – 500,000)  2% = 20,000. Plant and machinery: Rs. New machine (17,550  25%  /12) Existing plant (((340,500 – 80,000) – (125,900 – 57,000))  25%) 9

3,291 47,900

–––––––

51,191

Computer equipment = 112,000  40% = Rs.44,800 (2)

–––––––

Cost of new machine Rs. Purchase price (20,000 – 3,000 – 1,000) Delivery costs Installation costs Interest on loan taken out to finance the purchase

16,000 500 750 300

–––––––

17,550

–––––––

© Emile Woolf International

172

The Institute of Chartered Accountants of Pakistan

Answers

7.4

ADJUSTMENTS LIMITED (a)

Lathe The lathe was purchased in 2009 and was originally being written off over an estimated useful life of twelve years. As at 1 January 2015 six of the years have elapsed with a further six years remaining. It was decided that the machine will now only be usable for a further four years. IAS 16 Property, plant and equipment requires that where the original estimate of useful life is revised, adjustments should be made in current and future periods (not in prior periods). The unamortised cost of the asset should be charged to revenue over the remaining useful life of the asset. The net book value of Rs.75,000 should therefore be charged over the remaining four years of useful life, giving an annual depreciation charge of Rs.18,750. The revision is not a change in accounting policy, or a fundamental error but a change in accounting estimate. It is therefore not appropriate to deal with any excess depreciation by adjusting opening retained earnings.

(b)

Grinder The grinder was purchased in 2012 and was originally being depreciated on a straight line basis. It has now been decided to depreciate this on the sum of digits basis. IAS 16 requires that depreciation methods be reviewed periodically and if there is a significant change in the expected pattern of economic benefits, the method should be changed. Depreciation adjustments should be made in current and future periods. This change might be appropriate if, for instance, usage of the machine is greater in the early years of an asset’s life when it is still new and consequently it is appropriate to have a higher depreciation charge. If the change is implemented, the unamortised cost (the net book value) of the asset should be written off over the remaining useful life commencing with the period in which the change is made. The depreciation charge for the remaining life of the asset will therefore be as follows. Year

No of digits

2015 2016 2017 2018 2019 2020 2021

7 6 5 4 3 2 1 —— 28 ——

1/2  7 (7 + 1)

7/28  Rs.70,000 6/28  Rs.70,000

Depreciation Rs. 17,500 15,000 12,500 10,000 7,500 5,000 2,500 ———– Rs. 70,000 ———–

Disclosure will need to be made in the accounts of the details of the change, including the effect on the charge in the year.

© Emile Woolf International

173

The Institute of Chartered Accountants of Pakistan

Financial accounting and reporting II

(c)

Leasehold land IAS 16’s allowed alternative treatment in respect of measurement of property plant and equipment (subsequent to initial recognition), is that of revaluation. Revaluation is made at fair value. Where any item of property plant or equipment is revalued, the entire class to which the asset belongs should be revalued. Revaluations must be kept up to date. Where there are volatile movements in fair value, the revaluation should be performed annually. Where there are no such movements, revaluations every three to five years may be appropriate. Accumulated depreciation at the date of revaluation is either (i)

restated proportionately with the change in the gross carrying amount so that the carrying amount after the revaluation equals the revalued amount (e.g. where revaluations are made to depreciated replacement cost using indices)

(ii)

eliminated against the gross carrying amount of the assets and the net amount restated to the revalued amount of the asset (e.g. where buildings are revalued to their market value).

IAS 16 requires that the subsequent charge for depreciation should be based on the revalued amount. The annual depreciation will therefore be Rs.62,500, i.e. Rs.1,500,000 divided by the 24 years of remaining life. There will then be a difference between the revalued depreciation charge and the historical depreciation charge. The resulting excess depreciation may be dealt with by a movement in reserves, i.e. by transferring from the revaluation reserve to retained earnings a figure equal to the depreciation charged on the revaluation surplus each year.

© Emile Woolf International

174

The Institute of Chartered Accountants of Pakistan

Answers

7.5

FAM Accounting policies (a)

Property, plant and equipment is stated at historical cost less depreciation, or at valuation.

(b)

Depreciation is provided on all assets, except land, and is calculated to write down the cost or valuation over the estimated useful life of the asset. The principal rates are as follows. Buildings Plant and machinery Fixtures and fittings

Fixed asset movements

Cost/valuation

2% pa straight line 20% pa straight line 25% pa reducing balance

Land Plant Fixtures, Payments on and and fittings, account and buildings machinery tools and assets in the Total equipment course of construction

Rs.000 Rs.000

Cost at 1 January 2015 Revaluation adjustment Additions Reclassifications Disposals

900 1,613 600 – – 154 100 – – (277) ——— ——— Cost at 31 December 2015 100 1,490 2015 valuation 1,500 ——— ——— Depreciation At 1 January 2015 80 458 Revaluation adjustment (80) – Provisions for year (W2) 17 298 Disposals – (195) —— —— At 31 December 2015 17 561 —— —— Net book value At 31 December 2015

At 31 December 2014

Rs.000 Rs.000 390 – 40 – (41) —— 389

Rs.000

——

91 2,994 – 600 73 (W1) 267 (100) – – (318) —— ——— 64 2,043 1,500 —— ———

140 – 70 (31) —— 179 ——

– – – – —— – ——

1,583 929 ——— ———

210 ——

64 ——

2,786 ———

820 1,155 ——— ———

250 ——

91 ——

2,316 ———

678 (80) 385 (226) —— 757 ——

Land and buildings have been revalued during the year by Messrs Jackson & Co on the basis of an existing use value on the open market.

© Emile Woolf International

175

The Institute of Chartered Accountants of Pakistan

Financial accounting and reporting II

The corresponding historical cost information is as follows. Land and buildings Rs.000 Cost Brought forward Reclassification

900 100 ——— 1,000 ———

Carried forward Depreciation Brought forward Provided in year

80 10 ——— 90 ——— 910 ———

Carried forward Net book value

WORKINGS (1)

Additions to assets under construction Deposit on computer

Rs.000 53 20 —— 73 —— Rs.000

(2)

600 Depreciation on buildings 40 + (100  2%) 2% straight line depreciation is equivalent to a 50 year life. The buildings are ten years old at valuation and therefore have 40 years remaining. Depreciation on plant (1,613 + 154 – 277)  20% Depreciation on fixtures (390 + 40 – 41 – 140 + 31)  25%

© Emile Woolf International

176

17

298 70

The Institute of Chartered Accountants of Pakistan

Answers

7.6

IMRAN LIMITED (a)

Specific borrowings Rs. Borrowing costs incurred: 13% bank loan outstanding for 10 months (Rs. 32 million x 306/365 x 13%) 11% bank loan outstanding for 5 months (Rs. 10 million x 153/365 x 11%) Borrowing costs Less: Interest that relates to suspension 13% bank loan: (Rs. 32 million x 61/365 x 13%) 11% bank loan (Rs. 10 million x 61/365 x 11%)

3,487,562 461,096 3,948,658 695,233 183,836 (879,068) 3,069,590

Less: Investment income on temporary investment of the borrowings

(b)

(500,000) 2,569,590

General borrowings Phase 1

Building cost capitalised Financed out of rights issue Financed from borrowing

20,000,000 (15,000,000) 5,000,000

Period to the year end March 1 to December 31 April 1 to December 31 October 1 to December 31 Period of suspension Number of days for which borrowing should be capitalised

Phase 3

18,000,000

16,000,000

18,000,000

16,000,000

306 275

Weighted average borrowing rate (W3) Fraction of the year for which the rate should be applied to costs incurred Capitalised borrowing

(61)

(61)

92 

245

214

92

 12.73%

 12.73%

 214/365 1,343,451

 92/365 513,385

 12.73%  245/365 427,240

Total

© Emile Woolf International

Phase 2

2,284,076

177

The Institute of Chartered Accountants of Pakistan

Financial accounting and reporting II

Workings W1: Average borrowings

Rs.m

13% bank loan outstanding for 10 months (Rs. 32 million x 306/365 days) 11% bank loan outstanding for 5 months (Rs. 10 million x 153/365 days) Average outstanding for the year

26,827,397 4,191,781 31,019,178

W2: Borrowing costs incurred (or from part a) 13% bank loan outstanding for 10 months (Rs. 32 million x 306/365 x 13%) 11% bank loan outstanding for 5 months (Rs. 10 million x 153/365 x 11%) Borrowing costs

Rs.m 3,487,562 461,096 3,948,658

W3: Weighted average rate Borrowing costs

© Emile Woolf International

/ Average outstanding for the year = 3,948,658 (W2)/31,019,178 (W1) = 12.73%

178

The Institute of Chartered Accountants of Pakistan

Answers

7.7

HUMAYUN CHEMICALS LIMITED (a)

If review is performed on June 30, 2015 Cost of machine Depreciation charged @ 20% for the year ended June 30, 2013 and June 30, 2014 (Rs. 10,000,000  Rs. 3,000,000) x 20% x 2 WDV as at June 30, 2014 Residual value (10% of the cost of machine) Depreciable amount - on July 1, 2014

Rs. 10,000,000

Rs. 2,800,000 Rs. 7,200,000 Rs. 1,000,000 Rs. 6,200,000

Remaining useful lives

6 years

Depreciation charge for the year ended June 30, 2015

Rs. 1,033,333

If review is performed on June 30, 2014 Cost of machine Depreciation for the year ended June 30, 2013 (Rs. 10,000,000 - Rs. 3,000,000) x 20% WDV as at June 30, 2013 Residual value (10% of the cost of machine)

(b)

Rs. 10,000,000 Rs. 1,400,000 Rs. 8,600,000 Rs. 1,000,000

Depreciable amount - on July 1, 2013 Remaining useful lives

Rs. 7,600,000 6 years

Depreciation charge for the year ended June 30, 2015 Depreciation charged in the financial statement for the year ended June 30, 2014 Effect of change in estimate to be incorporated (Reversal) [Rs. 1,400,000  1,266,667)

Rs. 1,266,667 Rs. 1,400,000 (Rs. 133,333)

According to IAS-16, the following factors should be considered when estimating the useful life of a depreciable asset: (i)

Expected usage

(ii)

Expected physical wear and tear

(iii)

Obsolescence

(iv)

Legal or other limits on the use of the assets.

Once the useful life of a depreciable asset is determined, it shall be reviewed at least at each financial year-end. If expectations vary from the previous estimates, then change should be adjusted for current and future periods in accordance with the requirements of IAS 8.

© Emile Woolf International

179

The Institute of Chartered Accountants of Pakistan

Financial accounting and reporting II

7.8

FARADAY PHARMACEUTICAL LIMITED Date 01.07.2011

30.06.2012

01.07.2012

01.07.2012

30.06.2013

30.06.2013

01.07.2013

01.07.2013

30.06.2014

01.07.2014

© Emile Woolf International

Debit Rs.000

Particulars Building Bank (Record purchase of plant)

200,000 200,000

Depreciation Accumulated depreciation – Building (Record depreciation for the year 2012) Working: Rs. 200,000 ÷ 20 = Rs. 10,000

10,000

Accumulated depreciation – Building Building (Reversal of prior year depreciation)

10,000

Building Surplus on revaluation of fixed assets (Increase in value through revaluation) Working: Rs. 230,000 – Rs. 190,000 = Rs. 40,000

40,000

Depreciation Accumulated depreciation – Building (Record depreciation for the year 2013) Working: Rs. 230,000 ÷ 19 = Rs. 12,105

12,105

Surplus on revaluation of fixed assets Retained earnings/Profit & loss account (transfer of surplus through retained earning to the extent of excess depreciation) Working: Rs. 40,000 ÷ 19 = Rs. 2,105 Accumulated depreciation – Building Building (Reversal of prior year depreciation) Surplus on revaluation of fixed assets Revaluation expense Building (Decrease in value through revaluation) Working: Reversal of Surplus balance (Rs. 40,000 – Rs. 2,105) Rs. 37,895. Balancing figure of Rs. 10,000 charged to Profit and Loss Building value decline: (Rs. 230,000 – Rs. 12,105) – Rs. 170,000 =Rs. 47,895 Depreciation Accumulated depreciation – Building (Record depreciation for the year 2014) Working: Rs. 170,000 ÷ 18 = Rs. 9,444 Accumulated depreciation – Building Building (Reversal of prior year depreciation)

180

Credit Rs.000

10,000

10,000

40,000

12,105

2,105 2,105

12,105 12,105 37,895 10,000 47,895

9,444 9,444

9,444 9,444

The Institute of Chartered Accountants of Pakistan

Answers

Debit Rs.000

Date

Particulars

01.07.2014

Building Revaluation income Surplus on revaluation of fixed assets (balancing) (Reversal of prior year impairment) Working: Revaluation income = Rs. 10,000 – [ Rs. 10,000 – Rs. 9,444] = Rs. 9,444 Building: [Rs. 170,000 – Rs. 9,444] – Rs. 180,000 =Rs. 19,444

19,444

Depreciation Accumulated depreciation – Building (Record depreciation for the year 2015) Working: Rs. 180,000 ÷ 17 = Rs. 10,588

10,588

30.06.2015

30.06.2015

© Emile Woolf International

Surplus on revaluation of fixed assets Retained earnings (Reverse the excess depreciation) Working: Rs. 10,000 ÷ 17 = Rs. 588

181

Credit Rs.000 9,444 10,000

10,588

588 588

The Institute of Chartered Accountants of Pakistan

Financial accounting and reporting II

7.9

SPIN INDUSTRIES LIMITED Commitment fee Actual borrowing costs of specific loan General borrowing costs Less: Investment income Interest costs to be capitalised

(W1) (W1) (W2)

Rupees 125,000 2,050,000 1,175,283 (137,500) 3,212,783

W1 Outstanding amount (Rs.) Specific loan Utilised till first repayment Utilised after the first repayment

Outstanding month up to Rate of Months outstanding completion interest

Borrowing cost (Rs.)

25,000,000

1-Sep-14

31-Jan-15

5

12%

1,250,000

20,000,000

1-Feb-15 31-May-15

4

12%

800,000 2,050,000

General Borrowings Utilised after specific loan nd exhausted on 2 payment to contractor (W3) Principal payment of specific loan 3rd payment to contractor 4rd payment to contractor

(W4)

8,125,000

1-Dec-14

5,000,000 12,000,000 9,000,000

31-May-15

6

1-Feb-15 31-May-15 1-Feb-15 31-May-15 1-Jun-15 31-May-15

4 4 0

12.08% 12.08%

490,750 201,333 483,200 -

12.08% 12.08%

1,175,283

W2: Investment income Surplus fund available from 1-Sep-14 to 30-Nov-14 (Rs. 25m – Rs. 0.125m – Rs. 8m – Rs. 10m) × 8% × 3/12 W3: Specific loan utilization Commitment fee Payment for obtaining permit 1st payment to contractor 2nd payment to contractor (balancing)

Rs.137,500

125,000 8,000,000 10,000,000 6,875,000 25,000,000

2nd payment to contractor (total) Less: paid out of specific loan (as worked out above) Paid from general borrowing

15,000,000 6,875,000 8,125,000

W4: Weighted average rate of borrowing

From Bank A From Bank B

Weighted average amount of loan (Rs.) 25,000,000 20,000,000 45,000,000

Rs. 25,000,000 × 13% × 9/12

Weighted average rate of borrowing (Rs. 5,437,500 / 45,000,000)

© Emile Woolf International

182

Interest (Rs.) 2,437,500 3,000,000 5,437,500

12.08%

The Institute of Chartered Accountants of Pakistan

Answers

7.10

SCIENTIFIC PHARMA LIMITED Scientific Pharma Limited Journal entries for the year ended June 30, 2015

30.06.2015

30.06.2015

30.06.2015

30.06.2015

30.06.2015

Repair and maintenance expenses Account payable / Bank (Repair cost of major break down of the plant) Depreciation expense (45,000-2,000)/10.5 years Accumulated depreciation (Depreciation expense for the year) Revaluation surplus (10,380/10.5) Retained earnings (Incremental depreciation credited to retained earnings) W1 Impairment loss Property, plant and equipment (Impairment of plant due to break down) Revaluation surplus W1 Impairment loss (Impairment loss adjusted against revaluation)

Debit Rs.000 1,500

1,500

4,095 4,095 989 989

5,296 5,296

5,296 5,296

W1: Impairment loss Recoverable amount WDV of the plant on impairment date W2 Impairment loss as on 30.06.2015

19,227 (24,523) (5,296)

W2: WDV of the plant on impairment date FOB price (US$ 800,000 at Rs. 52) Other charges including installation cost Accumulated depreciation (1-1-2006 to 30-6-2010) WDV as on 30-6-2010 Revaluation surplus (45,000-34,620) Revalued amount as of July 1, 2010 Accumulated depreciation (1-7-2010 to 30-6-2015)

Credit Rs.000

Rs.000 41,600 7,000 48,600 {(48,600-2,000)/15*4.5}

(13,980) 34,620 10,380 45,000

{(45,000-2,000)/10.5*5)

(20,476) 24,523

WDV as on 30-6-2015 W3: Revaluation surplus on impairment date Revaluation surplus Transferred to retained earnings (01.07.2010 to 30.06.2015) Revaluation surplus balance on impairment date

W2

10,380

(10,380/10.5*5)

(4,943) 5,437

Since impairment loss is less than the revaluation surplus on impairment date, the full amount of impairment would be adjusted against the revaluation surplus.

© Emile Woolf International

183

The Institute of Chartered Accountants of Pakistan

Financial accounting and reporting II

7.11

QURESHI STEEL LIMITED Capital work in progress – Factory building Progress invoices received from the contractor (30,000+20,000+10,000+15,000) (Rain damages paid would be chargeable to profit and loss account/ insurance claim) Borrowing costs to be capitalised: Loan processing charges Interest on bank loan Interest on running finance Interest income from surplus loan amount Capital work in progress – June 30, 2015

Rs.000 75,000.00

500.00 1,841.67 2,730.00 (395.00) 79,676.67

W1 W2 W4

W1: Interest on bank loan: Interest amount To 31-05-2015 30-06 -2015

From 01-14-2014 01-06-2015

Outstanding loan amount 25,000 20,000

Months 6 1

Rs.000 Interest at 13% 1625.00 216.67 1,841.67

W2: Interest on running finance Rs.000 Payments from Payment Description s date

01-07-14 Advanced payment st 15-10-14 1 progress bill nd 15-01-15 2 progress bill rd 15-04-15 3 progress bill 31-05-15 Loan interest 31-05-15 Loan instalment

Invoice amount

Payments net of deductions

10,000

10,000

30,000

25,500

20,000

17,000

17,000

10,000

8,500

7,500

Right issue

Bank loan

15,000

1,625 5,000 15,000

*24,500

Months outstanding up to 30-6-10

Interest at 15% per annum (W3)

10,000

12.00

1,500

10,500

8.50

1,116

Running finance

-

-

-

1,000

2.50

31

1,625

1.00

20

5,000 29,125

1.00

63 2,730

*Loan amount of Rs. 25,000,000 less processing charges of Rs. 500,000 W3: Average rate of interest for running finance facility (9,000/60,000)

15%

W4: Interest income from surplus loan amounts: Interest income From 01-14-14 16-01-15

© Emile Woolf International

To 15-01-15 15-04-15

Months 1.5 3.0

184

Surplus loan amounts 24,500 7,500

Rs.000 Interest income at 8% (245) (150) (395)

The Institute of Chartered Accountants of Pakistan

Answers

7.12

GRANITE CORPORATION Borrowing costs to be capitalised Workings Commitment fee @ 1% Borrowing costs on specific loan Borrowing costs on running finance

1 3

Less: Investment income

2

2015 6,987,500 1,381,625

2014 700,000 3,033,333 -

(2,099,001) 6,720,124

(1,381,334) 2,351,999

Suspension

70,000,000

4

0

From June 30 to first principal repayment After the 1st principal repayment After the 2nd principal repayment to completion Amount to be capitalised as on 30-Jun-2015

70,000,000 65,000,000 60,000,000

2 6 3

0 1 0

Borrowing cost to be capitalised (Rs.) @ 13%

Outstanding month

From commencement on to June 30 Amount to be capitalised as on 30-Jun-2014

Net outstanding months

Outstanding amount (Rs.)

W1 : Actual borrowing costs on specific loan

3,033,333 3,033,333 2 5 3

1,516,667 3,520,833 1,950,000 6,987,500

W2 : Investment income (All amounts in Rupees) Available Funds

O/s amount up to completion

Used to reduce running finance (14%) Amount

Rs.(70m 25m - 0.7m) 44,300,000 Investment income – 2014 Rs. (70m 25m 0.7m) 44,300,000 Rs.(44.3 5m 4.55m) 34,750,000 Investment income – 2015

Income

Invested in saving account @ 8% Amount

Total Income

Income

4

10,000,000

466,667

34,300,000

914,667

1,381,334 1,381,334

2

10,000,000

233,333

34,300,000

457,333

690,666

5

10,000,000

583,335

24,750,000

825,000

1,408,335 2,099,001

Net outstanding months

30,250,000

4

1

3

1,058,750

5,000,000 4,225,000 10,000,000 49,475,000

3 3 0

0 0 0

3 3 0

175,000 147,875 1,381,625

Amount

2nd payment to contractor (Rs. 65m - 34.75m) Payment of 2nd instalment Principal Interest (Rs. 65m x 13% x 6/12) 3rd payment to contractor

© Emile Woolf International

Suspension

Description

No. of months outstanding

2015

185

Borrowing cost to be capitalised (Rs.) @ 14%

W3 : Interest on running finance

The Institute of Chartered Accountants of Pakistan

Financial accounting and reporting II

CHAPTER 8 – IAS 38: INTANGIBLE ASSETS 8.1

FAZAL In accordance with IAS 38, expenditure on intangible assets must be expensed unless it meets the recognition criteria for capitalisation. These criteria require the demonstration that future benefits will arise from the incurred costs. It would be difficult to prove that this is the case in relation to training costs and IAS 38 specifically states that training costs should always be expensed as they are incurred and not treated as an intangible asset. Hence the treatment adopted by Fazal is not correct and the costs being carried forward must be expensed to the year’s profits.

8.2

HENRY Property, plant and equipment Plant and machinery Cost On 1 January 2015 Additions

Rs. X 30,000

–––––––

On 31 December 2015

X

–––––––

Accumulated depreciation On 1 January 2015 Charge for the year (30,000  9/12 ÷ 5)

X 4,500

–––––––

On 31 December 2015

X

–––––––

Carrying amount On 31 December 2014

X

–––––––

On 31 December 2015

25,500

–––––––

Intangible assets Internally generated research and development expenditure Cost On 1 January 2015 Additions

Rs. 412,500 45,000

––––––––

On 31 December 2015

457,500

––––––––

Accumulated amortisation On 1 January 2015 Charge for the year (W)

68,750

––––––––

On 31 December 2015

68,750

––––––––

Carrying amount On 31 December 2014

412,500

––––––––

On 31 December 2015

© Emile Woolf International

388,750

––––––––

186

The Institute of Chartered Accountants of Pakistan

Answers

Working Amortisation charge (Project A) Rs. Total savings (100,000 + 300,000 + 200,000) 2015 amortisation charge (100,000/600,000  412,500)

600,000 68,750

Tutorial notes The costs in respect of Project B cannot be capitalised as there are uncertainties surrounding the successful outcome of the project – but the machine bought may be capitalised in accordance with IAS16. The 2015 costs in respect of Project C can be capitalised as the uncertainties have now been resolved. However, the 2014 costs cannot be reinstated.

8.3

TOBY Intangible assets

Cost On 1 January 2015 Additions (W1) On 31 December 2015 Accumulated amortisation/impairment On 1 January 2015 Written off/amortised during the year (W1 and W2) On 31 December 2015

Goodwill

Patents

Brands

Total

Rs.

Rs.

Rs.

Rs.

10,000

20,000

50,000

80,000

–––––––

–––––––

–––––––

10,000

20,000

50,000

–––––––

–––––––

–––––––

-

-

-

-

3,000

–––––––

3,000 -

–––––––

On 31 December 2015

80,000

–––––––

–––––––

Carrying amount On 31 December Year 0

–––––––

7,000

–––––––

2,500

–––––––

2,500

–––––––

-

–––––––

17,500

–––––––

7,500

–––––––

7,500

–––––––

-

–––––––

42,500

–––––––

13,000

–––––––

13,000

–––––––

-

–––––––

67,000

–––––––

Workings (1) Goodwill on acquisition of George Rs. Cost of acquisition Minus fair value of net assets acquired (100,000 – 5,000) Goodwill Recoverable value Amortisation of patent

(3)

20,000 ÷ 8 = Rs.2,500 Amortisation of brand

––––––––

10,000 (7,000)

––––––––

Impairment write off (2)

105,000 (95,000)

3,000

––––––––

50,000 ÷ 5 × 9/12 = Rs.7,500 Tutorial note IAS38 Intangible assets prohibits the recognition of internally generated brands (3) or internally-generated goodwill (4).

© Emile Woolf International

187

The Institute of Chartered Accountants of Pakistan

Financial accounting and reporting II

8.4

BROOKLYN 1

Development expenditure IAS 38 on intangibles requires that research and development be considered separately: 

research – which must be expensed as incurred



development – which must be capitalised where certain criteria are met.

It must first be clarified how much of the Rs.3 million incurred to date (10 months at Rs.300,000) is simply research and how much is development. The development element will only be capitalised where the IAS 38 criteria are met. The criteria are listed below together with the extent to which they appear to be met. 

The project must be believed to be technically feasible. This appears to be so as the feasibility has been acknowledged.



There must be an intention to complete and use/sell the intangible. Completion is scheduled for June 2016



The entity must be able to use or sell the intangible. Interest has been expressed in purchasing the knoWhow on completion It must be considered that the asset will generate probable future benefits. Confirmation is required from Brooklyn as to the extent of interest shown by the pharmaceutical companies and whether this is of a sufficient level to generate orders and to cover the deferred costs. Availability of adequate financial and technical resources must exist to complete the project. The financial position of Brooklyn must be investigated. A grant is being obtained to fund further work and the terms of the grant, together with any conditions, must be discussed further.







Able to identify and measure the expenditure incurred. A separate nominal ledger account has been set up to track the expenditure.

If all of the above criteria are met, then the development element of the Rs.3m incurred to date must be capitalised as an intangible asset. Amortisation will not begin until commercial production commences. 2

Provision Although the claim was made after the reporting period, IAS 10 considers this to be an adjusting event after the reporting period. The employment of the individual dates back to 20X2 and so the lawsuit constitutes a current obligation for the payment of damages as a result of this past event (the employment). The amount and the timing are not precisely known but the likelihood of payment of damages by Brooklyn is probable and so a provision should be made for the estimated amount of the liability, as advised by the lawyer. Disclosure, rather than provision, would only be appropriate if the expected settlement was possible or remote, and the lawyer’s view is that a payment is more likely than not. It is not appropriate to calculate an expected value where there is only one event, instead a provision should be made for the most likely outcome. The lawyer has various views on the possible payout, but the most likely payout is Rs.500,000 as this has a 50% probability. As settlement of the provision is not anticipated until 2018, the provision should be discounted back at 8% to give a liability of Rs.476,280.

© Emile Woolf International

188

The Institute of Chartered Accountants of Pakistan

Answers

Provided that the payment from the insurance company is virtually certain, this should be shown as an asset, also at its discounted value of Rs.47,628, being 10% of the provision. In both cases the discounting should be unwound over the coming three years through profit or loss. 3

Revaluation IAS 16 on Property, Plant and Equipment does not impose a frequency for updating revaluations. It simply requires a revaluation where it is believed that the fair value of the asset has materially changed. Hence, if in the past there have been material differences between the carrying amount and fair value at the 5 yearly review then Brooklyn should consider having more frequent valuations following on from this year’s valuation. Revaluations should be regular and not timed simply when property prices are at a peak. It is not acceptable for Brooklyn to defer its next revaluation while values are low. If property prices do fall in 2016, then it may be necessary to perform an impairment test in accordance with IAS 36 Impairment of assets. If it is believed that an asset value has moved materially, then all assets in that class must be revalued. Hence it is not sufficient for Brooklyn to just revalue the London property. IAS 16 does not require the valuation to be performed by an external party, and so the use of the property manager to conduct the valuations is acceptable. Notes to the financial statements will disclose that he is not independent of the company.

8.5

ZOUQ INC (a)

(i) (ii) (iii) (iv)

(v)

The depreciable amount of an intangible asset with a finite useful life shall be allocated on a systematic basis over its useful life. Amortization shall begin when the asset is available for use Amortization shall cease at the earlier of the date that the asset is classified as held for sale and the date that the asset is derecognised. The amortization method used shall reflect the pattern in which the asset's future economic benefits are expected to be consumed by the entity. The amortization charge for each period shall be recognised in statement of profit or loss. Goodwill Account

(b)

Rupees 01.01.2014

Goodwill recognised (W1)

01.01.2015

Balance b/d

Rupees

270,000,000 31.12.2014 31.12.2014 270,000,000 220,000,000 220,000,000

189

50,000,000 220,000,000 270,000,000

31.12.2015

© Emile Woolf International

Impairment of goodwill Balance b/d

Balance b/d

220,000,000 220,000,000

The Institute of Chartered Accountants of Pakistan

Financial accounting and reporting II

Brand Account Rupees 01.01.2014

Brand recognised

Rupees

100,000,000

31.12.2014 31.12.2014

Amortization Balance c/d

100,000,000 01.01.2015

Balance b/d

10,000,000 90,000,000 100,000,000

90,000,000

31.12.2015

-

31.12.2015 31.12.2015

Amortization Impairment of Brand Balance c/d

90,000,000

10,000,000 13,500,000 68,000,000 90,000,000

W1: Value of goodwill Rupees 1,350,000,000

Purchase price (50,000,000 x Rs. 30 x 90%) Less: Fair value of net identifiable assets and liabilities (Rs. 1,100,000,000 x 90%) Less: Value of brand (Rs. 100,000,000 x 90%)

(990,000,000) (90,000,000)

Goodwill recognised

8.6

270,000,000

STAR-BRIGHT PHARMACEUTICAL LIMITED Star-Bright Pharmaceutical Limited Statement of financial position As at December 31, 2015 2014 2015 Restated Rs. in million Non-current assets Intangible asset – brand [Note 8] Shareholders’ equity Retained earnings (W5 and 6)

274

285

2,071

1,879

498 43 541

460 38 498

Star-Bright Pharmaceutical Limited Statement of Financial Position As at December 31, 2015 8- Intangible assets – Brand Cost At beginning of the year (2015: 382+24+54+38, 2014: 382+ 24+54) Capitalised during the year Amortization At beginning of the year (W1 and 2) During the year (W3 and 4)

(213) (54) (267)

*3

274

(163) (50) (213)

*4

285

W1 : 382 x 50% + 24 x 30% + 54 x 20% + 38 x 10% = 213 W2 : 382 x 40% + 24 x 20% + 54 x 10% = 163 W3 : 541 x 10% = 54 W4 : 498 x 10% = 50 W5 : 1,950 + 24 + 54 + 38 + 43 – [267 – (382 x 60%)] = 2,071 W6 : 1,785 + 24 + 54 + 38 – [213 – (382 x 50%)] = 1,879

© Emile Woolf International

190

The Institute of Chartered Accountants of Pakistan

Answers

8.7

RAISIN INTERNATIONAL (a)

Following are the criteria that should be used while recognizing intangible assets from research and development work. (i) No intangible asset arising from research shall be recognised. (ii)

An intangible arising from development shall be recognised if, and only if , an entity can demonstrate all of the following:  the technical feasibility of completing the intangible asset so that it will be available for use or sale.  its intention to complete the intangible asset and use or sell it. 

its ability to use or sell the intangible asset.



  (b)

how the intangible asset will generate probable future economic benefits. Among other things, the entity can demonstrate the existence of a market for the output of the intangible asset or the intangible asset itself or, if it is to be used internally, the usefulness of the intangible asset. the availability of adequate technical, financial and other resources to complete the development and to use or sell the intangible asset. its ability to measure reliably the expenditure attributable to the intangible asset during its development.

(i)

Since the product met all the criteria for the development of the product, it should be recognised as an intangible in the statement of financial position (SOFP) of the company. However, RI should capitalise only the development work (i.e. Rs. 9 million) as intangible asset. IAS-38 does not allow capitalization of cost relating to the research work, training of staff and cost of trial run. Since the product has a useful life of 7 years, the amortization expense amounting to Rs. 0.32 million (Rs. 9 million × 3/12 ÷ 7 years) should be recorded in the statement of profit or loss.

(ii)

This purchasing of right to manufacture should be recognised as an intangible in the SOFP because:  it is for an established product which would generate future economic benefits.  cost of the patent can be measured reliably. Since there is a finite life, the patent must be amortised over its useful life. The useful life will be shorter of its actual life (i.e. 10 years) and its legal life (i.e. 5 years. The amortization to be recorded in SOCI is Rs. 2.83 million (Rs. 17 million × 10/12 ÷ 5).

(iii) The acquired brand should be recognised as an intangible in the SOFP because acquisition price is a reliable measure of its value. The amortization to be recorded in SOCI is Rs. 0.12 million (Rs. 2 million ÷ 10 years x 7/12). (iv) The carrying value of the intangible asset should be increased to Rs. 10 million in the SOFP. Since there is an indefinite useful life of the intangible assets, it should not be amortised. Instead, RI should test the intangible asset for impairment by comparing its recoverable amount with its carrying amount.

© Emile Woolf International

191

The Institute of Chartered Accountants of Pakistan

Financial accounting and reporting II

CHAPTER 9 – IAS 17: LEASES 9.1

DAWOOD The lease has been correctly classified as a finance lease as it is being leased for its entire useful economic life which indicates that the risks and rewards of ownership have been transferred to Dawood. The leased asset should be capitalised as a non-current asset and depreciated over the 5 year lease period/useful life. By 31st March 2015, the net book value of the asset will be Rs.225,000, being the cost of Rs.250,000 less 6 months depreciation. A finance lease creditor should be established initially for Rs.250,000. During the year, finance costs will be added and the first payment of Rs.29,500 will be deducted. The finance costs on the lease of Rs.45,000 (Being total payments of (10 × Rs.29,500) – cash price Rs.250,000) will be spread over the lease term using the sum-of-digits method. Sum of digits =

n  (n  1) 9  10 = 45  2 2

(Note: n = number of periods of borrowing, and as the payments are in ADVANCE not arrears, Dawood is only financing the asset of 4 1/2 years (9 six-monthly periods)). Therefore, the finance cost relating to the first 6 months through to 31st March 2015 is Rs.9,000 (9/45  Rs.45,000). This will result in a movement on the finance lease creditor as follows: 6 months to Brought forward

Payment

Finance cost

Carried forward

31 March 13

250,000

(29,500)

9,000

229,500

30 Sept 13

229,500

(29,500)

8,000

208,000

31 March 14

208,000

(29,500)

7,000

185,500

The year end liability of Rs.229,500 will be split between current liabilities Rs.51,000 (29,500 + (29,500–8,000)), and the balance of Rs.178,500 as non-current liabilities.

9.2

FINLEY Financial statements for the year ended 31 December 2015 (extracts) Statement of financial position Non-current assets Property, plant and equipment (36,000 – 9,000)

Rs. 27,000

Current liabilities Finance lease obligations (W1)

10,000

Non-current liabilities Finance lease obligations (W1)

17,950

Statement of profit or loss Depreciation on leased assets ((36,000 ÷ 4) Finance lease charges (W1)

© Emile Woolf International

192

9,000 1,950

The Institute of Chartered Accountants of Pakistan

Answers

Workings (1)

Finance lease obligations (boat) Opening balance

Lease payment

Capital outstanding

Interest at 7.5%

Closing balance

Rs.

Rs.

Rs.

Rs.

Rs.

31 December 2015

36,000

(10,000)

26,000

1,950

27,950

31 December Year 5

27,950

(10,000)

17,950

1,346

19,296

Year ended

Rs. Current (balancing figure)

10,000

Non-current

17,950 –––––––

27,950 –––––––

9.3

FABIAN Financial statements for the year ended 31 December 2015 (extracts) Statement of financial position Non-current assets

Rs.

Property, plant and equipment (126,760 – 31,690)

95,070

Current assets Trade and other receivables (W1)

6,250

Current liabilities Finance lease obligations (W2) Non-current liabilities

30,056

Finance lease obligations (W2) Statement of profit or loss

69,380

Operating expenses Operating lease rentals (W1)

5,450

Depreciation on leased assets (126,760 ÷ 4)

31,690

Finance costs Finance lease charges (W2)

12,676

Tutorial note The notes to the financial statements would disclose the fact that included in trade and other receivables is Rs.3,750 (W1) due in more than one year.

© Emile Woolf International

193

The Institute of Chartered Accountants of Pakistan

Financial accounting and reporting II

Workings (1)

Operating lease (car) Rs. statement of profit or loss charge = ((7,500 + (36  700))  6/36) = Cash paid in 2015 (7,500 + (700  6)) Minus charged to statement of profit or loss in 2015

–––––––

11,700 (5,450)

–––––––

Prepayment at end of 2015

6,250

–––––––

Prepayment at end of 2015 Cash paid in 2016 (12  700) Minus charged to statement of profit or loss in 2016 (5,450  2) Prepayment at end of 2016 (2)

5,450

6,250 8,400 (10,900) –––––––

3,750

–––––––

Finance lease obligations (machine) Date

Opening balance

Interest (10%)

Lease payment

Closing balance

2015 2016

Rs. 126,760 99,436

Rs. 12,676 9,944

Rs. (40,000) (40,000)

Rs. 99,436 69,380 Rs.

Current (balancing figure) Non-current

30,056 69,380

–––––––

99,436

–––––––

9.4

XYZ INC (a)

Extracts from the financial statements of XYZ Inc at 31 December 2015 Statement of financial position (i)

Tangible fixed assets held under finance leases Plant and machinery Rs.000 Cost At 1 January 2015 Additions

x 4,400 ——— 4,400 ———

At 31 December 2015 Accumulated depreciation At 1 January 2015 Charge for the year

x 629 ——— 629 ———

At 31 December 2015 Net book value At 31 December 2015

3,771 ———

At 1 January 2015

© Emile Woolf International

x ———

194

The Institute of Chartered Accountants of Pakistan

Answers

(ii)

Finance lease payables Amounts payable: Rs.000 4,516 996 ——— 3,520 ———

Within one to five years (600  8 – 284) Less future finance charges

Accruals Rs.000 951

Finance leases (667 + 284) Statement of profit or loss Profit is stated after charging

Rs.000 Finance charges Depreciation 4,400 ÷ 7 (b)

604 (W2) 629

Table Period ended

30 June 2015 31 December 2015 30 June 2016 31 December 2016 30 June 2017 31 December 2017 30 June 2018 31 December 2018 30 June 2019 31 December 2019

Amount borrowed Rs.000 4,400 4,092 3,760 3,403 3,018 2,604 2,158 1,677 1,160 600

Repaid Capital due 7.68% Amount due for period interest at period end Rs.000 Rs.000 Rs.000 Rs.000 (600) (600) (600) (600) (600) (600) (600) (600) (600) (600)

3,800 3,492 3,160 2,803 2,418 2,004 1,558 1,077 560 –

292 268 243 215 186 154 119 83 40 –

4,092 3,760 3,403 3,018 2,604 2,158 1,677 1,160 600 –

Comparison Period 1 2 3 4 5 6 7 8 9 10

© Emile Woolf International

Sum of digits (W2) Rs.000 320 284 249 213 178 142 107 71 36 – ——— 1,600 ———

195

Actuarial (as above) Rs.000 292 268 243 215 186 154 119 83 40 – ——— 1,600 ———

The Institute of Chartered Accountants of Pakistan

Financial accounting and reporting II

WORKINGS (1)

Calculation of finance charge Rs.000 6,000 (4,400) ——— 1,600 ———

Minimum lease payments 5  600  2 Fair value of asset Finance charge (2)

Allocation of finance charge Period ended 30 June 2015 31 December 2015

30 June 2016 31 December 2016 30 June 2017 31 December 2017 30 June 2018 31 December 2018 30 June 2019 31 December 2019 n(n + 1) 9(9 + 1) = 2 2

(3)

Digits

Finance charge Rs.000 9/45  1,600 8/45  1,600

9 8

7/45  1,600 6/45  1,600 5/45  1,600 4/45  1,600 3/45  1,600 2/45  1,600 1/45  1,600

7 6 5 4 3 2 1 – ——

320 284 ——— 604 249 213 178 142 107 71 36 ———

45

1,600

——

———

Lease obligation Period ended

30 June 2015 31 December 2015 30 June 2016 31 December 2016

© Emile Woolf International

Amount borrowed

Repaid

Capital due for period

Interest

Rs.000

Rs.000

Rs.000

Rs.000

Amount due at period end Rs.000

4,400 4,120 3,804 3,453

(600) (600) (600) (600)

3,800 3,520 3,204 2,853

320 284 249 213

4,120 3,804 3,453 3,066

196

The Institute of Chartered Accountants of Pakistan

Answers

9.5

SNOW INC Extracts from the financial statements of Snow Inc for year ended 31 December 2015 Statement of profit or loss Profit is stated after charging Rs.000 Finance charges

(1,714 + 1,429 + 9,614) (W1 and 2)

Depreciation

12,757 41,667

Statement of financial position Tangible fixed assets held under finance leases Plant and machinery Rs.000 – 185,000 ———— 185,000 ————

Cost At 1 January 2015 Additions (35,000 + 150,000) At 31 December 2015 Accumulated depreciation At 1 January 2015 35,000 15,000 Charge for year  + 5  3



 

41,667 ——— 41,667 ———

At 31 December 2015 Net book value At 31 December 2015

143,333 ———— – ———

At 1 January 2015 Finance lease payables Amounts payable: Rs.000 Within one to five years Less future finance charges

166,000 18,243 ———— 147,757 ————

(6,500  4 + 35,000  4) (2,857 + 15,386 *)

* 35,000  5 = 175,000 – 150,000 – 9,614 = 15,386 Accruals Rs.000 Finance leases

© Emile Woolf International

46,000 (11,000 + 35,000)

197

The Institute of Chartered Accountants of Pakistan

Financial accounting and reporting II

WORKINGS (1)

Snowplough (a)

Calculation of finance charge Rs.000 2,000 39,000 (35,000) ——— 6,000 ———

Deposit MLP (6  6,500) Fair value of asset Finance charge

(b)

Allocation of finance charge Period ended

Digits

Finance charge Rs.000

30.06.2015

6 621  6,000

1,714

31.12.2015

5 521 4 421 3 321 2 221 1 121

 6,000

1,429

 6,000

1,143

 6,000

857

 6,000

571

 6,000

286 ——— 6,000 ———

30.06.2016 31.12.2016 30.06.2017 31.12.2017

—— 21 —— n (n + 1) 6 (7) = = 21 2 2 (c)

Period ended 30.6.15 31.12.15 30.6.16 31.12.16

(2)

Capital Interest Amount Repayment Capital O/S at start O/S at end O/S at end Rs.000 Rs.000 Rs.000 Rs.000 Rs.000 33,000 28,214 23,143 17,786

1,714 1,429 1,143 857

34,714 29,643 24,286 18,643

(6,500) (6,500) (6,500) (6,500)

28,214 23,143 17,786 12,143

Snow machine Period ended

Amount Repayment Capital Interest Amount O/s at start O/S at start at 8.36% O/S at end Rs.000 Rs.000 Rs.000 Rs.000 Rs.000 31.12.15 150,000 (35,000) 115,000 9,614 124,614 31.12.16 124,614 (35,000) 89,614 7,492 97,106

© Emile Woolf International

198

The Institute of Chartered Accountants of Pakistan

Answers

9.6

MIRACLE TEXTILE LIMITED Miracle Textile Limited Statement of financial position (Extracts) As at 30 June 2015 Note

2015

2014

Rs.

Rs.

4

16,000,000

18,000,000

LIABILITIES Non-current liabilities Obligation under finance lease

9

6,505,219

10,633,074

Current liabilities Current portion of obligation under finance lease

9

4,127,856

3,566,925

ASSETS Non-current assets Property, plant and equipment

Miracle Textile Limited Notes to the financial statements (Extracts) As at 30 June 2015 4- Property, plant and equipment Leased assets Cost Opening balance Addition during the year Accumulated depreciation Opening balance Depreciation for the year Balance as at 30 June

2015

2014

Rs.000

Rs.000

20,000,000 20,000,000

20,000,000 20,000,000

(2,000,000) (2,000,000) (4,000,000) 16,000,000

(2,000,000) (2,000,000) 18,000,000

9- Obligations under finance lease (W1) Minimum lease payment Not later than one year Later than one year but not later than five years Later than five years

© Emile Woolf International

30-Jun-2015 Financial charges Principal for future outstanding periods

Minimum lease payment

30-Jun-2014 Financial charges Principal for future outstanding periods

Rs.

Rs.

Rs.

Rs.

Rs.

Rs.

5,800,000

1,672,144

4,127,856

5,800,000

2,233,075

3,566,925

7,800,000

1,294,781

6,505,219

13,600,000

2,966,926

10,633,074

13,600,000

2,966,926

10,633,074

19,400,000

5,200,000

14,200,000

199

The Institute of Chartered Accountants of Pakistan

Financial accounting and reporting II

9.1

The Company has entered into a finance lease agreement with a bank in respect of a machine. The finance lease liability bears interest at the rate of 15.725879% per annum. The company has the option to purchase the machine by paying an amount of Rs. 2 million at the end of the lease term. The lease rentals are payable in annual instalments ending in June 2015. There are no financial restrictions in the lease agreement.

W1: Lease Schedule Payment date 01.07.2013 01.07.2014 01.07.2015 01.07.2016 30.06.2017

9.7

Opening principal 20,000,000 14,200,000 10,633,075 6,505,219 1,728,222

Instalment 5,800,000 5,800,000 5,800,000 5,800,000 2,000,000

Principal repayment 5,800,000 3,566,925 4,127,856 4,776,997 1,728,222 20,000,000

Interest @ 15.725879% 2,233,075 1,672,144 1,023,003 271,778 5,200,000

Closing principal 14,200,000 10,633,075 6,505,219 1,728,222 -

SHOAIB LEASING LIMITED Entries in the books of Lessor Date

Particulars

Dr.

01.07.2015

Lease payments receivable (W1) Machine Unearned finance income (W1)

2,680,000

30.06.2016

Bank

2,100,000 580,000 860,000

Lease payments receivable 30.06.2016

30.06.2017

860,000

Unearned finance lease Finance income (W2)

272,941

Bank

860,000

272,941

Lease payments receivable 30.06.2017

30.06.2018

860,000

Unearned finance lease Finance income (W2)

196,640

Bank

960,000

196,640

Lease payments receivable 30.06.2018

Unearned finance lease Finance income (W2)

W1: Total finance income Total future lease payments (Rs. 860,000 x 3) Add: Purchase bargain option Gross investment in finance lease Less: Cost of assets Total finance income

© Emile Woolf International

Cr.

200

960,000 110,419 110,422 Rupees 2,580,000 100,000 2,680,000 2,100,000 580,000

The Institute of Chartered Accountants of Pakistan

Answers

W2: Amortization schedule Instalment

Interest

Principal

Date

Principal Opening

30.06.2016 30.06.2017 30.06.2018

Rs. 2,100,000 1,512,941 849,581

Rs. 860,000 860,000 960,000

Rs. 272,941 196,640 110,419* 580,000

Rs. 587,059 663,360 849,581 2,099,997.04

Principal Closing Rs. 1,512,941.20 849,581.19 nil

Note that there is a rounding adjustment of Rs. 3 in the last interest amount. Shoaib Leasing Limited Extracts from the statement of financial position as at June 30, 2016 2016 Rupees Non-current assets Net investment in leases

Note 10 849,578

Current assets Current portion of net Investment in leases 10

10.1

10.2

663,360

Net investment in leases Minimum lease payments receivables (Note 10.1) Add: Residual value of leased assets Gross Investments in leases Less: Unearned lease income (Rs. 580,000 - Rs. 272,941 - 3) Net investment in leases (Note 10.2) Note 10.2 Less: Current portion of net investment in leases

Minimum lease payments Less than one year More than one year and less than 5 years

Net investment in leases Less than one year More than one year and less than 5 years

© Emile Woolf International

201

1,720,000 100,000 1,820,000 (307,062) 1,512,938 (663,360) 849,578

860,000 860,000 1,720,000

663,360 849,578 1,512,938

The Institute of Chartered Accountants of Pakistan

Financial accounting and reporting II

9.8

NEPTUNE LIMITED (a)

Journal entries (i) Finance Lease: Debit

Date

Particulars

1-Jan-2015

Finance lease debtors Unearned finance lease income Sale (Record sale of vehicles on finance lease) Bank Finance lease debtors (Instalment received under finance lease) Unearned finance lease income Finance lease income (Interest income earned at 15%)

1-Jan-2015

31-Dec-2015

(ii)

Credit

Rupees 12,000,000 3,295,690 8,704,310

2,000,000 2,000,000

1,005,647 1,005,647

Operating lease:

1-Jan-2015

Bank Unearned rental income (Operating lease instalment received in advance) 31-Dec-2015 Unearned rental income Rental income (11,410,000÷3)(W2) (Booking of operating lease income) Depreciation expenses 31-Dec-2015 (15,000,000÷6) Accumulated depreciation on machine. (Yearly depreciation on machine)

4,000,000 4,000,000

3,803,333 3,803,333

2,500,000 2,500,000

Reason for choice of leases: 1. 2.

© Emile Woolf International

Lease A should be accounted for as a finance lease because the lease term covers the entire economic life. Since none of the conditions specified in IAS-17 (Leases) for classification as a finance lease is being met, Lease B shall be considered as an operating lease.

202

The Institute of Chartered Accountants of Pakistan

Answers

W1

Finance lease: Opening Balance

Year

2015 2016 2017 2018 2019 2020

(A)

Rs. 8,704,310 7,709,957 6,566,450 5,251,417 3,739,130 2,000,000

(B) (A)+(B)

W2

Instalment

Income at 15%

Rs. 2,000,000 2,000,000 2,000,000 2,000,000 2,000,000 2,000,000 8,000,000 10,000,000

Rs. 1,005,647 856,493 684,967 487,713 260,870 0 1,433,550 2,290,043

Recovery of Principal Rs. 994,354 1,143,507 1,315,033 1,512,287 1,739,130 2,000,000 6,566,450 7,709,957

Closing balance Rs. 7,709,957 6,566,450 5,251,417 3,739,130 2,000,000 0

Operating lease: Rs.

Annual instalment

(b)

2015 2016 2017

(4,000,000 × 95%) (3,800,000 × 95%)

4,000,000 3,800,000 3,610,000 11,410,000

Neptune Limited Notes to the Financial Statements For the year ended December 31, 2015 (i)

Investment in finance lease 2015 Rs.

Present value of minimum lease payments Less: current maturity

Less than one year One to five years

Net investment in leases 2015

Rs.

Rs.

1,143,507 6,566,450 7,709,957

The minimum lease payment has been discounted on interest rate of 15% per annum to arrive at their present value. Rentals are paid in annual instalments. Operating lease Not later One to than one Total five years year Future minimum lease payments (W2)

© Emile Woolf International

Gross investment in finance leases 2015 2,000,000 8,000,000 10,000,000 (2,290,043) 7,709,957

Less: unearned finance income Net investment in leases

(ii)

7,709,957 (1,143,507) 6,566,450

203

Rs.

Rs.

3,800,000

3,610,000

Rs.

7,410,000

The Institute of Chartered Accountants of Pakistan

Financial accounting and reporting II

9.9

QUARTZ AUTO LIMITED (a) Entries to record the lease in books of Quartz Auto Limited Description Debit Credit Lease receivable (2,715,224 × 5) + 700,000 14,276,120 Cost of goods sold [(900,000 × 7) - (100,000 ×7 × 5,951,974 0.49718)] Inventory (900,000 x 7) 6,300,000 Sales (Note – 1) 9,101,974 Unearned finance income 4,826,120 Bank Lease receivable

2,715,224

Unearned finance income Finance income

1,417,500

2,715,224 1,417,500

Note – 1 Lower of fair value i.e. 9,450,000 (Rs. 1,350,000 x 7) and PV of minimum lease payment (2,715,227 x 3.35219 = 9,101,974)

(b)

Disclosure in the financial statements 1-

2015

Net investment in lease

Rs.

Lease receivable (2,715,227 x 4) Unguaranteed residual amount Gross investment in lease

10,860,896 700,000 11,560,896

Less: Unearned finance income (4,826,120 – 1,417,500)

(3,408,620) 8,152,276

1.1 Details of investment in finance lease

Not later than one year Later than one year but not later than five years Later than five years

Gross investment in lease 2,715,224 8,845,672 11,560,896

Net investment in lease 1,492,383 6,659,893 8,152,276

(W1) Year ended

Instalment at year end

Interest

Principal

31/06/2015 31/06/2016 31/06/2017 31/06/2018 31/06/2019

2,715,224 2,715,224 2,715,224 2,715,224 2,715,224

1,417,500 1,222,841 998,984 741,548 445,247

1,297,724 1,492,383 1,716,240 1,973,676 2,269,977

© Emile Woolf International

204

Net Investment in Lease 9,450,000 8,152,276 6,659,893 4,943,653 2,969,977 700,000

Gross Investment in Lease 14,276,120 11,560,896 8,845,672 6,130,448 3,415,224 700,000

The Institute of Chartered Accountants of Pakistan

Answers

9.10

LODHI TEXTILE MILLS LIMITED Particulars Generator A (i) Cash / Bank Accumulated depreciation – Generator

Debit

Loss on sale/ Impairment loss Property, plant and equipment - Generator

Credit

6,000,000 2,500,000 *1,500,00 0 10,000,000

*(This amount comprises of impairment loss amounted to Rs. 1 million and loss on disposal amounted to Rs. 0.5 million.)

(ii)

Assets subject to finance lease - Generator Liabilities against assets subject to finance lease

6,000,000 6,000,000

Generator B (i) Cash / Bank Accumulated depreciation – Generator Property, plant and equipment - Generator

6,000,000 6,000,000

(ii)

Assets subject to finance lease - Generator Liabilities against assets subject to finance lease

6,000,000

Impairment loss Accumulated impairment (ASFL) - Generator

1,000,000

(iii)

Generator C (i) Cash / Bank Accumulated depreciation – Generator

12,000,000

6,000,000

1,000,000

8,000,000 3,000,000 10,000,00 0

Property, plant and equipment - Generator Deferred income OR Surplus on revaluation of fixed assets (ii)

Assets subject to finance lease - Generator Liabilities against assets subject to finance lease

© Emile Woolf International

205

1,000,000 8,000,000 8,000,000

The Institute of Chartered Accountants of Pakistan

Financial accounting and reporting II

9.11

NOMAN ENGINEERING LIMITED Journal entries Debit

Credit

Rs.000

Rs.000

Date

Description

1-Jul-2014

Bank Accumulated depreciation (18,750-15,000) Property, plant and equipment Deferred gain on disposal (20,000-15,000) (Disposal of plant under sale and finance lease back)

20,000 3,750

Property, plant and equipment Long term finance lease liability (Plant acquired under sale and lease back)

20,000

1-Jul-2014

31-Dec-2014 Long term finance lease liability Interest expense Bank (Payment of 1st. Instalment of lease liability) 30-Jun-2015

30-Jun-2015

W.1 W.1

18,750 5,000

20,000

1,127 1,373 2,500

Long term finance lease liability Interest expense Bank (Payment of 2nd. Instalment of lease liability)

1,204 1,296 2,500

Deferred gain on disposal (5,000/6) Gain on disposal (Deferred gain on amortised over the life of the plant)

833 833

30-Jun-2015

Depreciation expense (20,000/6) 3,333 Accumulated depreciation 3,333 (Depreciation for the year for plant) Note: If there is no reasonable certainty that the lessee will obtain ownership by the end of the lease term, the asset shall be fully depreciated over the shorter of the lease term and its useful life.

W1: Liability against finance lease Balance Payments made on

1-Jul-2014 31-Dec-2014 30-Jun-2015

Balance 30-6-2015

© Emile Woolf International

206

Instalment payments 2,500 2,500 5,000

Interest at 13.731%

Principal balance

1,373 1,296 2,669

20,000 (1,127) (1,204) (2,331) 17,669

The Institute of Chartered Accountants of Pakistan

Answers

CHAPTER 10 – IAS 37: PROVISIONS CONTINGENT LIABILITIES AND CONTINGENT ASSETS AND IAS 10: EVENTS OCCURRING AFTER THE REPORTING DATE 10.1

BADAR Decommissioning costs IAS 37 Provisions, Contingent Liabilities and Contingent Assets only permits a provision to be made if three conditions are met: (i)

The company has a present obligation, either legally or constructively, as a result of a past event;

(ii)

Probable outflow of resources is required to settle the obligation; and

(iii)

A reliable estimate is available.

Although there is no legal requirement to restore the site, the company has established a constructive obligation by setting a valid expectation in the market, due to its published policies and past practice, from which it cannot realistically withdraw. It therefore appears probable that Badar will have to pay money to improve the site and so a provision should be created for the expected amount. As the expected payment of Rs.100,000 will not be settled for three years, the provision should be discounted and entered at its net present value of Rs.75,131 (Rs.100,000/(1.1)3). Over the three years, the discounting should be unwound and charged to profit or loss as finance costs, resulting in a provision of Rs.100,000 by the end of the third year. The cost of the construction work has been correctly capitalised. The cost of the future decommissioning work should be added to this asset so that the total costs of the site can be matched to the revenue from the copper over the period of mining. This will result in an asset of Rs.575,131 which should be depreciated over the three year life in line with anticipated revenues.

10.2

GEORGINA (1)

Litigation for damages Under IAS37, a provision should only be recognised when: 

an entity has a present obligation as a result of a past event



it is probable that an outflow of economic benefits will be required to settle the obligation



a reliable estimate can be made of the amount of the obligation.

Applying this to the facts given: 

Georgina’s legal advisors have confirmed that there is a legal obligation. This arose from the past event of the sale, on 1 September 2015 (i.e. before the year end).



Probable is defined as ‘more likely than not’. The legal advisors have confirmed that it is likely that the claim will succeed.



A reliable estimate of Rs.500,000 has been made.

© Emile Woolf International

207

The Institute of Chartered Accountants of Pakistan

Financial accounting and reporting II

Therefore a provision of Rs.500,000 should be made. Counter-claim IAS37 requires that such a reimbursement should only be recognised where receipt is ‘virtually certain’. Since the legal advisors are unsure whether this claim will succeed no asset should be recognised in respect of this claim. (2)

Claim for unfair dismissal In this case, the legal advisers believe that success is unlikely (i.e. possible rather than probable). Therefore this claim meets the IAS37 definition of a contingent liability: 

a possible obligation



arising from past events



whose existence will be confirmed only by the occurrence or nonoccurrence of one or more uncertain future events.

The liability is a possible one, which will be determined by a future court case or tribunal. It did arise from past events (the dismissal had taken place by the year end). This contingent liability should be disclosed in the financial statements (unless the legal advisors believe that the possibility of success is in fact remote, and then no disclosure is necessary). (3)

Returns Applying the IAS37 conditions in (1) to the facts given: 

Although there is no legal obligation, a constructive obligation arises from Georgina’s past actions. Georgina has created an expectation in its customers that such refunds will be given.



As at the year end, based on past experience, an outflow of economic benefits is probable.



A reliable estimate can be made. This could be 1% × 400,000 but since the returns are now all in the actual figure of Rs.3,500 can be used.

Therefore a provision of Rs.3,500 should be made. (4)

Closure of division Applying the above IAS37 conditions in (1) to the facts given: 

A present obligation exists because at the year end there is a detailed plan in place and the closure has been announced in the press.



An outflow of economic benefits is probable.



A reliable estimate of Rs.300,000 has been made.

However, IAS37 specifically states in respect of restructuring that any provision should include only direct expenses, not ongoing expenses such as staff relocation or retraining. Therefore a provision of Rs.250,000 (300,000 – 50,000) should be made.

© Emile Woolf International

208

The Institute of Chartered Accountants of Pakistan

Answers

10.3

EARLEY INC (a)

IAS 10 (revised) Events After the Statement of financial position Date states that assets and liabilities should be adjusted for events occurring after the statement of financial position date that provide additional evidence relating to conditions existing at the statement of financial position date. It specifically includes the example of bad debts, where evidence of bankruptcy of a debtor occurs after the year end. In this case, Nedengy appears to have recovered part of the debt and as such only Rs.200,000 needs to be provided. It may be argued that the receivership has occurred as a result of events occurring after the statement of financial position date, as a result of a change in legislation for example, but this is unlikely. IAS 18 Revenue states that when uncertainty arises about the collectability of an amount already included in revenue, the amount should be recognised as an expense.

(b)

It is likely that the fall in the value of the property will fit the IAS 10 (revised) definition of adjusting events noted in (a) above, unless, again, it can be argued that the decline in the property market occurred after the year-end. IAS 36 Impairment of assets and IAS 16 Property, Plant and Equipment require that the carrying amount of property, plant and equipment should be reviewed periodically in order to assess whether the recoverable amount has fallen below the carrying amount. Where it has, the property, plant and equipment should be written down to the recoverable amount, either through the statement of profit or loss as an expense, or though other comprehensive income to revaluation reserve in shareholder’s equity, but only to the extent that the balance on the revaluation reserve relates to a previous revaluation surplus on the same asset.

(c)

IAS 2 Inventories requires that inventories be stated at the lower on cost and net realisable value. Net realisable value is the estimated selling price in the ordinary course of business less the estimated costs of completion and the estimated costs necessary to make the sale. Unless Earley was making a significant margin on the tricycles, it is likely that the reduction in selling price of 30% will necessitate a write- down to net realisable value, especially considering the transportation costs to Iraq which must be included. If the Iraqi option is unlikely to proceed, it may be necessary to write the tricycles down to scrap value.

(d)

Under IAS 10, the nationalisation is likely to be regarded as a non-adjusting event that merely requires disclosure in the financial statements. IAS 27 Consolidated Financial Statements and Accounting for Investments in Subsidiaries, requires that an investment in a enterprise should be accounted for as an investment (under IAS 39: Financial Instruments: Recognition and Measurement) from the date that it ceases to fall within the definition of a subsidiary and does not become an associate. It seems here that Earley has neither control nor significant influence, nor even an investment as the assets have been in fact, expropriated. The loss of the investment should be accounted for in the year in which it occurred, but disclosed in the current year. If the loss of the subsidiary results in Earley no longer being a going concern, then the event becomes an adjusting event.

(e) & (f)

© Emile Woolf International

Both of the events described are non-adjusting event which should be disclosed, but not adjusted for in the current year financial statements.

209

The Institute of Chartered Accountants of Pakistan

Financial accounting and reporting II

10.4

ACCOUNTING TREATMENTS (a)

IAS 37 Provisions contingent liabilities and contingent assets states that contingent gains should not be recognised as income in the financial statements. The company has a debit balance already in its books which indicates that it must be reasonably certain that at least part of the claim will be paid. This element of the claim then is probably not a contingency at all. The remaining part (the difference between the Rs.15,000 and the Rs.18,600) is, and should be disclosed and not accrued.

(b)

IAS 16 Property, Plant and Equipment requires that the carrying amount of property, plant and equipment should be reviewed periodically in order to assess whether the recoverable amount has fallen below the carrying amount. Where it has, the property, plant and equipment should be written down to the recoverable amount through the statement of profit or loss as an expense. In this case this would result in the recognition of an expense of Rs.200,000. (280,000 – 80,000). It may be the case that the amounts involved are so significant as to warrant separate disclosure in the statement of profit or loss under IAS 8 Net Profit of Loss for the Period, Fundamental Errors and Changes in Accounting Policies.

(c)

IAS 37 states that contingent liabilities should not be recognised. Though a provision should be made for amounts where the company has an obligation to pay them. The question in this case is whether or there is an obligating event within the context of IAS 37. On balance it seems inappropriate to recognise a provision in respect of this amount but the possible liability should be disclosed as a contingent liability.

(d)

(i)

the nature of the contingency

(ii)

the uncertainties surrounding the ultimate outcome

(iii)

the likely effect, ie Rs.500,000 loss less likely tax relief.

IAS 2 Inventories requires that inventories be stated at the lower on cost and net realisable value. Net realisable value is the estimated selling price in the ordinary course of business less the estimated costs of completion and the estimated costs necessary to make the sale. In this case, cost is Rs.1,800 and net realisable value is Rs.1,600

(e)

The company should set up a provision for Rs.100,040, ie should accrue for the 10% probable liability. It should disclose the possible liability under contingent liabilities. The disclosure is as noted in (c) except that the financial effect is Rs.300,120 (30%  Rs.1,000,400). The balance should be ignored as it is a remote contingent liability.

Tutorial note In (c) above it is not appropriate to provide for 20%receivableRs.500,000, ie Rs.100,000. This would only be appropriate where the event is recurring many times over. In (e) it is appropriate to use the percentages provided, as warranty work is provided for.

© Emile Woolf International

210

The Institute of Chartered Accountants of Pakistan

Answers

10.5

J-MART LIMITED (a)

Adjusting events: Adjusting events are events that provide further evidence of conditions that existed at the reporting date. Examples of adjusting events include: (i)

The subsequent determination of the purchase price or of the proceeds of sale of non-current assets purchased or sold before the year end.

(ii)

The renegotiation of amounts owing by customers or the insolvency of a customer

(iii)

Amounts received or receivable in respect of insurance or the insolvency of a customer.

(iv)

The settlement after the reporting date of a court case that confirms that the entity had a present obligation at the reporting date.

(v)

The receipt of the information after the reporting date indicating that an asset was impaired at the reporting date.

(vi)

The discovery of fraud or errors that show that the financial statements are incorrect.

Non-adjusting events: Non-adjusting events are indicative of conditions that arose subsequent to the reporting date. Examples of non-adjusting events might be:

(b)

(i)

Losses of non-current assets or inventories as a result of a catastrophe such as fire or flood

(ii)

Closing a significant part of the trading activities if this was not begun before the year end

(iii)

The value of an investment falls between the reporting date and the accounts are authorised

(iv)

Announcement of dividend after year end.

(i)

The conditions attached to the sale give rise to a constructive obligation on the reporting date. A provision for the sales return should be recognised for 5% of June 2015 sales. The related cost should also be reversed.

(ii)

Since the law suit was already in progress at year-end and the amount of compensation can also be estimated, it is an adjusting event. A provision of Rs. 400,000 should be made.

(iii)

There is no obligating event at the year end either for the costs of fitting the smoke detectors or for fines under the legislation. No provision should be recognised in this regard.

(iv)

The obligating event is the communication of decision to the customers and employees, which gives rise to a constructive obligation from that date, because it creates a valid expectation that the division will be closed. Since no communication has yet been made, no provision is required in this regard.

© Emile Woolf International

211

The Institute of Chartered Accountants of Pakistan

Financial accounting and reporting II

(v)

The obligating event is the signing of the lease contract, which gives rise to a legal obligation. A provision is required for the unavoidable rent payments.

(vi)

Since the declaration was announced after year-end, there is no past event and no obligation at year-end and is therefore non-adjusting event. Details of the dividend declaration must, however, be disclosed.

10.6

AKBER CHEMICALS LIMITED (a)

The event is an accident, and since it happened before the year end, it is a past event. However, there is no present obligation since: (i)

there is no law requiring the company to clean the canal.

(ii)

there is no constructive obligation to clean the river since: 

a public statement has not been made;



there is no established pattern of past practice as this was the first time the company faced such a situation.

Although the company has decided to clean up the river and even has a reliable estimate of the costs thereof, no liability or provision should be recognised in the current year because:

(b)



the decision was taken after year end; and



the decision was not yet made public.

It is a non-adjustable event because the event due to which the net realizable value (NRV) of stock has fallen, arose after the reporting date. However, if this event is material, the company should disclose the decline in NRV in its financial statement for the year ended June 30, 2015.

(c)

The company should make the provision because: (i)

the company has a present obligation because of past event

(ii)

the claim of the customer is valid and is confirmed by the company's inspection team which shows that an outflow will be required to settle the obligation.

(iii)

the amount of outflow is reliably estimated i.e. Rs. 2 million.

Since the company is certain of recovery from the vendor, it should: (i)

disclose it as a separate asset.

(ii)

recognise a receivable but the same should not exceed the amount of the related provision i.e. rs. 2.0 million.

© Emile Woolf International

212

The Institute of Chartered Accountants of Pakistan

Answers

10.7

QALLAT INDUSTRIES LIMITED (i)

Provision must be made for estimated future claims by customers for goods already sold. The expected value i.e. Rs. 10 million ([Rs. 150m x 2%] + [Rs. 70m x 10%]) is the best estimate of the provision.

(ii)

Warehouse A: It is an onerous contract. as the warehouse has been sublet at a loss of Rs. 200,000 per month. QIT should therefore create a provision for the onerous contract that arises on vacating the warehouse. This is calculated as the excess of unavoidable costs of the contract over the economic benefits to be received from it. Therefore, QIL should immediately provide for the amount of Rs. 13.2 million. [5.5 years x 12 month x Rs. 200,000] in its financial statements i.e. for the year ended June 30, 2015. Warehouse B: It is not an onerous contract because the warehouse has been sublet at profit. Hence this would require no adjustment.

(iii)

A provision is to be made by QIL against a contingent liability as: (i)

There is a present obligation (legal or constructive) as a result of a past event; i.e. accident occurred on June 15, 2015.

(ii)

It is probable that outflow of resources will be required to settle the obligation; and

(iii)

A reliable estimate can be made of the amount of the obligation.

The amount of provision shall be Rs. 2.0 million i.e. the most probable amount as determined by the lawyer.

(iv)

A provision of Rs. 0.4 million is required in relation to penalty for March 1 to June 30, 2015 because at the reporting date there is a present obligation in respect of a past event. The reimbursement of penalty amount from the vendor shall be recognised when and only when it is virtually certain that reimbursement will be received if the entity settles the obligation. The reimbursement should be treated as a separate asset in the statement of financial position. However, in profit and loss statement, the expense relating to a provision may be netted off with the amount recognised as recoverable, if any.

10.8

SKYLINE LIMITED (i)

Although the debt owing by the customer existed at the reporting date, the customer’s inability to pay did not exist at that point. This condition only arose in January 2016 after the fire. Thus, this is a non-adjusting event. However, if it is material for the financial statements, the following disclosure should be made.

(ii)



Nature of the event



An estimate of its financial effect

The amount withdrawn before year end i.e. Rs. 1.5 million is an adjusting event as although it was discovered after year end it existed at the year end. However, since 60% has been recovered subsequently, Rs. 0.6 million would be provided.

© Emile Woolf International

213

The Institute of Chartered Accountants of Pakistan

Financial accounting and reporting II

The further withdrawal of Rs. 6.0 million is a non-adjusting event as it occurred after year end. However, if the events are considered material the following disclosures should be made:

 Nature of the event  The gross amount of contingency  The amount recovered subsequently (iii)

SL should not recognise the contingent gain until it is realised. However, if recovery of damages is probable and material to the financial statements, SL should disclose the following facts in the financial statements:

 Brief description of the nature of the contingent asset  An estimate of the financial effect. (iv)

SL should make a provision of the expected amount i.e. Rs. 1.2 million (Rs. 1.0 million x 60% + Rs. 1.5 million x 40%) because

 it is a present obligation as a result of past event;  it is probable that an outflow of resources embodying economic benefits will be required to settle the obligations; and  a reliable estimate can be made of the amount. In addition, SL should disclose the following in the notes to the financial statements:

 Brief nature of the contingent liability  The amount of contingency  An indication of the uncertainties relating to the amount or timing of any outflow.

10.9

WALNUT LIMITED (i)

This is an adjusting post reporting event as it provides evidence of conditions that existed at the end of the reporting period. The reasons for the competitor’s price reduction will not have arisen overnight, but will normally have occurred over a period of time, may be due to superior investment in technology. An inventory write down of Rs. 2.5 million should be recognised and the amount included as inventory on the Statement of Financial Position reduced to Rs. 12.5 million.

(ii)

The provision should be recognised because the obligating event is the communication of event to the public which creates a valid expectation that the division will be closed. However, the provision should only be recognised to the extent of redundancy costs. IAS prohibits the recognition of future operating losses, staff training and profits on sale of assets.

(iii)

This is a non-adjusting event because the burglary and theft of consumable stores occurred after reporting date. However, if the event is material, it should be disclosed in the financial statements unless the loss is recoverable from the insurance company.

© Emile Woolf International

214

The Institute of Chartered Accountants of Pakistan

Answers

(iv) The drop in value of investment in shares is a non-adjusting event. Since the legislation was announced after the reporting date, the event is not a past event. However, if the amount is material, it should be disclosed in the financial statements. (v)

This is an adjusting event as it provides evidence of conditions that existed at the end of the reporting period. The insolvency of a debtor and the inability to pay usually builds up over a period of time and it can therefore be assumed that it was facing financial difficulty at year-end. A bad debts expense of Rs. 1.5 million should be recognised in SOCI.

(vi) It is a non-adjusting event because the declaration was announced after the yearend and there was no obligation at year end. Details of the bonus shares declaration must, however, be disclosed.

10.10 ATTOCK TECHNOLOGIES LIMITED (i)

Since the event which caused the inventory to be sold at a loss occurred after the year end, it is non-adjusting event. However, the effect of the event should be disclosed in the financial statements for the year ended June 30, 2015.

(ii)

It is an adjusting event in accordance with the requirement of IAS-10. The debtor’s balance should be written down by 80% amount.

(iii)

It is non-adjusting event as the subsequent reduction in price is due to an event, introduction of competitive product, occurred after the reporting period.

(iv) Since this change was not enacted before the reporting date, it is a non-adjusting event. However, a disclosure should be made for this change. (v)

Since the declaration was announced after the year-end and there was no obligation at year-end it is a non-adjusting event. Details of the dividend declaration must, however, be disclosed.

© Emile Woolf International

215

The Institute of Chartered Accountants of Pakistan

Financial accounting and reporting II

CHAPTER 11 – IAS 8: ACCOUNTING POLICIES, CHANGES IN ACCOUNTING ESTIMATES AND ERRORS 11.1

WONDER LIMITED 2014 2015

(Restated)

Rs.m

Rs.m

Wonder Limited Extracts of Statement of financial position For the year ended 30 June 2015 Property, plant and equipment

178.50

111.50

Retained earnings

158.65

95.05

41.85

21.45

Deferred tax liability PPE:

Year 2015: 189 - [20 - (20 × 10% × 1.75)] + [56/4 – 56/7]

DTL: Year 2015: [(21.45 + (45 - 27) + {(6+2) × 30%}]

PPE: Year 2014: 130 - 18.5(Note X) DTL: Year 2014: 27 - 5.55 (Note X)

Wonder Limited Extracts from the Statement of profit or loss for the year ended 30 June 2015 Profit before taxation Taxation Profit after taxation

98.00

101.50

(34.40)

(36.45)

63.60

65.05

PBT : Year 2015 : 90 + (20 × 10% ) + [(56/4) - (56/7)]

PBT : Year 2014 : 120 - 18.5 (Note X)

Tax : Year 2015: 32 + [(6+2) × 30%]

Tax : Year 2014 : 42 - 5.55 (Note X)

Wonder Limited Extracts of statement of changes in equity for the year ended 30 June 2015 Retained earnings Rs.m

Balance as on 1 July 2013 (108-78)

30.00

Profit for the year ended 30 June 2014 (78 - 12.95 (Note X))restated

65.05

Balance as at 30 June 2014 - restated

95.05

Profit for the year ended 30 June 2015

63.60

Balance as at 30-June 2015

© Emile Woolf International

158.65

216

The Institute of Chartered Accountants of Pakistan

Answers

Wonder Limited Notes to the financial statements For the year ended 31 December 2015 X Correction of error During the year ended 30 June 2013, the repair works was erroneously debited to machinery account. The effect of this error is as follows: 2014 Rs.m

Effect on the statement of profit or loss (Increase) / decrease in expenses or losses Repairs and maintenance Depreciation (20 × 10% × 9 ÷ 12) Tax expenses (30% × (20-1.5)) Decrease in profit for the year

(20.00) 1.50 5.55 (12.95)

Effect on the statement of financial position Increase / (decrease) in assets Property, plant and equipment (20 – 1.5)

(18.50)

(Increase) / decrease in liabilities Deferred tax liability (Rs. 18.5 × 30%)

5.55

(Increase) / decrease in equity Retained earnings (18.50 - 5.55)

11.2

(12.95)

DUNCAN Statement of changes in equity (extract) Retained earnings

Retained earnings

Opening balance as reported Change in accounting policy (W2)

2015 Rs.000 23,950 450

2014 Rs.000 22,500 400

Re-stated balance Profit after tax for the period (W1) Dividends paid

24,400 4,442 (2,500)

22,900 3,250 (1,750)

–––––––

–––––––

Closing balance

26,342

–––––––

–––––––

24,400

–––––––

–––––––

2015

2014

Rs.000 4,712 600 (870)

Rs.000 3,200 500 (450)

Workings (1)

Revised profit

Per question Add back: Expenditure for the year Minus: Depreciation Revised profit

© Emile Woolf International

––––––

4,442

––––––

217

––––––

3,250

––––––

The Institute of Chartered Accountants of Pakistan

Financial accounting and reporting II

(2)

Prior period adjustment The prior period adjustment is the reinstatement of the Rs.400,000 asset on 1 January 2014 and the Rs.450,000 asset at 1 January 2015. On 31 December 2015 the closing balance above of Rs.26,342,000 can be reconciled as the original Rs.26,162,000 plus the reinstatement of the remaining asset of Rs.180,000.

11.3

MOHANI MANUFACTURING LIMITED Mohani Manufacturing (Private) Limited Statement of changes in equity For the year ended December 31, 2015 Retained Earnings Rs. in million

Balance at December 31, 2013 as previously reported (Rs. 89m – Rs. 21m) Effect of change in accounting policy (Rs. 37m - Rs. 35.5m) Balance at December 31, 2013 – restated Profit for the year ended December 31, 2014 - restated (W1) Balance at December 31, 2014 – restated Profit for the year ended December 31, 2015 (W2)

68.00 (1.50) 66.50 39.70 106.20 8.80

Balance at December 31, 2015

115.00

W1: Profit for the year ended December 31, 2014 (as restated) Profit as previously reported Incorrect recording of depreciation (Rs. 25 million – Rs. 10 million) Reversal of FIFO method Opening inventory Closing inventory

Rs. in million

21.00 15.00

37.00 (42.30) (5.30)

Application of weighted average method Opening inventory Closing inventory

(35.50) 44.50 9.00 39.70

W2: Adjusted profit for year ended June 30, 2015 Profit as per draft financial statements Adjustment in Opening Inventory FIFO Weighted average Adjustment in Closing Inventory FIFO Weighted average

42.30 (44.50) (2.20) (58.40) 54.40 (4.00)

Adjusted profit

© Emile Woolf International

15.00

8.80

218

The Institute of Chartered Accountants of Pakistan

Answers

CHAPTER 12 – IAS 12: INCOME TAXES 12.1

FRANCESCA Rs.

Rs.

Opening liability

1,340,600

Capital allowances during the year

50,000,000

Depreciation charged during the year

(45,000,000) –––––––––––

5,000,000

 30%

1,500,000

 30%

1,500

 30%

(1,200)

–––––––––––

Interest receivable in statement of profit or loss Interest received in tax computation

50,000 (45,000) –––––––––––

Receivable in statement of financial position

5,000 –––––––––––

Interest payable in statement of profit or loss Interest paid in tax computation

32,000 (28,000) –––––––––––

Payable in balance sheet

4,000 –––––––––––

Development costs as allowable expense

Revaluation

500,600

 30%

150,180

x 30%

329,850

6,000,000

Carrying value

(4,900,500) –––––––––––

Revaluation surplus

1,099,500 –––––––––––

Closing liability

––––––––––

3,320,930 ––––––––––

Rs. Charged to the revaluation reserve Charged in the statement of profit or loss (balancing figure) Total movement on the provision of (3,320,930 – 1,340,600)

© Emile Woolf International

219

329,850 1,650,480

––––––––––

1,980,330

––––––––––

The Institute of Chartered Accountants of Pakistan

Financial accounting and reporting II

12.2

SHEP (I) (a)

Corporate income tax liability - year ended 31st December 2015 Rs. Profit per accounts Add Depreciation

121,000 11,000 ———— 133,000 (15,000) ———— 117,000 ————

Less tax depreciation Taxable profits

Tax payable @ 30% (b)

35,100 ————

Deferred tax liability Rs. Carrying amount (48,000 + 12,000 = 60,000 – 11,000) Tax base (48,000 + 12,000 = 60,000 – 15,000) Temporary difference Deferred tax liability required @ 30%

(c)

49,000 45,000 ——— (4,000) ——— (1,200) ———

Movement on the deferred tax liability Rs. Balance b/f Statement of profit or loss (balancing figure) Balance c/f

(d)

 1,200 ——— 1,200 ———

Statement of profit or loss note Rs. 35,100 1,200 ———— 36,300 ————

Current tax expense Deferred tax expense Tax expense

© Emile Woolf International

220

The Institute of Chartered Accountants of Pakistan

Answers

12.3

SHEP (II) (a)

Corporate income tax liability - year ended 31st December 2016 Rs. Profit per accounts Add Depreciation Interest payable Provision Fine

125,000 14,000 500 1,200 6,000 ———— 146,700 (16,000) (150) ———— 130,550 ———— 39,165 ————

Less tax allowance (given) Interest receivable Taxable profits Tax payable @ 30%

(b)

Deferred tax liability Carrying amount Rs. Tangible assets Carrying amount (49bf – 14) Tax base (45bf – 16) Interest payable (25,000 x 8% x 3/12) Interest receivable (4,000 x 15% x 3/12) Provision

Temporary difference Rs.

29,000    ——— 29,000 ———

6,000 (500) 150 (1,200) ——— 4,450 ———

35,000 (500) 150 (1,200) ——— 33,450 ———

Deferred tax @30%

(c)

Tax base Rs.

1,335 ———

Movement on the deferred tax liability Rs. Balance b/f Statement of profit or loss (balancing figure) Balance c/f

(d)

1,200 135 ——— 1,335 ———

Statement of profit or loss note Rs. 39,165 135 ———— 39,300 ————

Current tax expense Deferred tax expense Tax expense

© Emile Woolf International

221

The Institute of Chartered Accountants of Pakistan

Financial accounting and reporting II

(e)

Tax reconciliation Rs. 125,000 ———— 37,500 1,800 ———— 39,300 ————

Accounting profit Accounting profit @ 30% Tax effect of the fine (6,000 @ 30%) Tax expense

12.4

SHEP (III) (a)

Corporate income tax liability - year ended 31st December 2017 Rs. Profit per accounts Add Depreciation Interest payable (note) Provision Entertainment

175,000 18,500  2,000 20,000 ———— 215,500 (24,700)  (17,800) (500) ———— 172,500 ————

Less tax allowance (given) Interest receivable (note) Development costs Provision Taxable profits

Tax payable @ 30%

51,750 ————

Note There is no adjustment to profit for the interest paid and the interest receivable. Consider the interest payable. The tax authority will disallow the closing accrual but will allow last year’s accrual (that has been paid in this year) as a deduction. These amounts are equal so there is no net effect. Similar comments can be made about the interest receivable.

© Emile Woolf International

222

The Institute of Chartered Accountants of Pakistan

Answers

(b)

Deferred tax liability Carrying amount Rs. Tangible assets Carrying amount (35bf – 18.5) 16,500 Tax base (29bf – 24.7) Interest payable (500) Interest receivable 150 Provision (2,700) Development expenditure 17,800 ——— 31,250 ——— Deferred tax @ 30%

(c)

Tax base Rs.

Temporary difference Rs.

4,300     —— 4,300 ——

12,200 (500) 150 (2,700) 17,800 ——— 26,950 ——— 8,085 ———

Movement on the deferred tax liability Rs. Balance b/f Statement of profit or loss (balancing figure) Balance c/f

(d)

Statement of profit or loss note Rs. 51,750 6,750 ———— 58,500 ————

Current tax expense Deferred tax expense Tax expense

(e)

1,335 6,750 ——— 8,085 ———

Tax reconciliation Accounting profit Accounting profit @ 30% Tax effect of the fine (20,000 @ 30%) Tax expense

© Emile Woolf International

223

Rs. 175,000 ———— 52,500 6,000 ———— 58,500 ————

The Institute of Chartered Accountants of Pakistan

Financial accounting and reporting II

12.5

SHEP (IV) (a)

Corporate income tax liability - year ended 31st December 2017 Rs. Taxable profits (as before)

172,500 ———— 58,650 ————

Tax payable @ 34% (b)

Deferred tax liability Rs. Temporary difference (as before)

26,950 ——— 9,163 ———

Deferred tax @ 34% (c)

Movement on the deferred tax liability Rs. Balance b/f Adjustment due to change in rate Opening balance restated to 34% (1,335 x 34/30) Statement of profit or loss (balancing figure) Balance c/f

(d)

Statement of profit or loss note Current tax expense Deferred tax expense relating to origination and reversal of temporary differences Deferred tax expense resulting from increase in tax rate Tax expense

(e)

1,335 178 ——— 1,513 7,650 ——— 9,163 ———

Rs. 58,650 7,650 178 ———— 66,478 ————

Tax reconciliation Accounting profit Accounting profit @ 34% Tax effect of the fine (20,000 @ 34%) Increase in opening deferred tax balances due to change in rate Tax expense

© Emile Woolf International

224

Rs. 175,000 ———— 59,500 6,800 178 ———— 66,478 ————

The Institute of Chartered Accountants of Pakistan

Answers

12.6

WAQAR LIMITED a)

Computation of current period income tax liability 2015

2014

Rs.m

Rs.m

Accounting profit before tax Less: Admissible deductions Capital Gain Tax depreciation on furniture and fittings Rs. 40.5 x 10% Rs. 40.5 (1-10%) x 10% Tax depreciation on Machinery Rs. 90 x 10% Rs. 90 (1-10%) x 10%

40.00

30.00

(10.00)

(8.00) (4.05)

(3.65) (9.00) (8.10)

Add: Inadmissible deductions Accounting depreciation on machinery Accounting depreciation on furniture and fittings Taxable profit Tax rate Tax payable (current tax)

25.00 5.00 48.25 30% 14.48

25.00 5.00 38.95 35% 13.63

b) Deferred taxation computation

Working 2 At December 31,2013 Machinery Furniture and fittings Deferred tax liability at December 31,2013 (35%) At December 31, 2014 Machinery Furniture and fittings Deferred tax liability at December 31,2014 (35%) WDV as at December 31, 2015 Machinery Furniture and fittings Deferred tax liability at December 31,2015 (35%)

© Emile Woolf International

NBV (W1)

Tax base (W1)

Temporary difference

Deferred tax liability

Rs.m

Rs.m

Rs.m

Rs.m

175.00 40.00

90.00 40.50

85.00 (0.50)

29.75 (0.18)

29.57

150.00 35.00

81.00 36.45

69.00 (1.45)

24.15 (0.51)

23.64

125.00 30.00

72.90 32.80

52.10 (2.80)

15.63 (0.84)

14.79

225

The Institute of Chartered Accountants of Pakistan

Financial accounting and reporting II

Working 1 Carrying amount and tax base of machinery Cost b/f Accumulated depreciation b/f At 31 December 2013 Accounting depreciation (200/8 years) Tax depreciation (10% of WDV)

NBV 200.0 (25.0) 175.0 (25.0)

d)

At 31 December 2014 Accounting depreciation (200/8 years) Tax depreciation (10% of WDV)

150.0 (25.0)

81.0

At 31 December 2015

125.0

72.9

NBV 50.0 (10.0) 40.0 (5.0)

Tax base 50.0

(8.1)

40.5 (4.05)

At 31 December 2014 Accounting depreciation (10%  50) Tax depreciation (10% of WDV)

35.0 (5.0)

At 31 December 2015

30.0

36.45 (3.65)

Movement on deferred taxation account (W2) At January 1 Change due to change in rate (23.64  5/35)

2015 23.64 (3.38) 20.26

32.8 2014 29.57 -

Change due to origination and reversal of temporary differences in the period (balancing figure)

(5.47)

(5.93)

At December 31

14.79

23.64

Tax expense Current tax Deferred tax: Due to origination and reversal of temporary differences in the period Due to change in rate

2015 14.48

2014 13.63 -

Tax expense e)

90.0 (9.0)

Carrying amount and tax base of furniture and fittings Cost b/f Accumulated depreciation b/f At 31 December 2013 Accounting depreciation (10%  50) Tax depreciation (10% of WDV)

c)

Tax base 200.0

Tax reconciliation Accounting profit Tax rate

(5.93)

5.63

7.7

2015 40.0 30% 12.0

Tax effect of untaxed gain: 30%  10.0 35%  8.0 Decrease in opening deferred tax balances due to change in rate (with rounding adjustment) Tax expense

© Emile Woolf International

(3.38) (5.47)

(3.0) (2.8) (3.37) 5.63

226

2014 30.0 35% 10.5

7.7

The Institute of Chartered Accountants of Pakistan

Answers

12.7

SHAKIR INDUSTRIES COMPUTATION OF TAX EXPENSE FOR THE YEAR ENDED DECEMBER 31, 2015 2015 Rs. in million

Profit before tax Add: Inadmissible expenses Accounting depreciation (Rs. 1.1 million + Rs. 0.7 million) Financial charges on finance lease Penalty paid to SECP Provision for gratuity

15.80 1.80 0.15 0.70 2.40 5.05

Less: Admissible expenses Tax depreciation Lease payments Payment of gratuity Borrowing cost capitalised

Rs.m

1.65 0.65 1.60 2.30 6.20

Taxable profit for the year

14.65

Current tax expense @ 35%

5.13

COMPUTATION OF DEFERRED TAX EXPENSE FOR THE YEAR ENDED DECEMBER 31, 2015 Carrying amount Fixed assets – Owned Fixed assets – Leased Capital work in progress Provision for gratuity (0.7 + 2.4 – 1.6) Obligation against assets subject to finance lease

Tax base

Temp difference

Rs.m

Rs.m

Rs.m

16.70 1.80 2.30 (1.50)

13.85 -

2.85 1.80 2.30 (1.50)

(1.20)

-

Total

(1.20) 4.25

Deferred tax expense @ 35%

1.49 Rs. in million

Deferred tax liability (Opening)

0.55

Deferred tax expense for the year (balancing figure) Deferred tax liability as at December 31, 2015 (Rs. 4.25 million x 35%)

0.94

© Emile Woolf International

227

1.49

The Institute of Chartered Accountants of Pakistan

Financial accounting and reporting II

12.8

MARS LIMITED (a)

Date

Particulars

Debit

Credit

Rupees 01.07.2014

01.07.2014

30.06.2015

Motor Vehicle - Cost Obligations under finance lease Capitalisation of the lease

1,600,000 1,600,000

Obligations under finance lease Bank First lease payment made in advance

480,000

Finance charges Accrued finance charges

153,451

480,000

153,451

Finance charge accrual for the year ended June 30, 2015 Working: (Rs. 1,600,000  480,000)  13.701% = Rs. 153,451) 30.06.2015

Depreciation Accumulated depreciation - Motor Vehicle

400,000 400,000

Depreciation charge for the year ended June 30, 2015 Working: Rs. 1,600,000 ÷ 4 = Rs. 400,000. Assuming that there is no reasonable certainty about transfer of ownership at the end of lease term. 30.06.2015

Tax expense (W1) 1,492,035 Tax payable 1,492,035 Recognition of tax expense for the year ended June 30, 2015)

30.06.2015

Tax expense Deferred tax (W2) Recognition of deferred tax asset.

W1

22,035 22,035

Tax computation Rs.

© Emile Woolf International

Accounting profit before tax Add: Depreciation on leased assets Add: Finance charges Less: Lease payment

4,900,000 400,000 153,451 (480,000)

Taxable profit

4,973,451

Tax @ 30%

1,492,035

228

The Institute of Chartered Accountants of Pakistan

Answers

W2

Deferred tax computation Carrying amount Taxable temporary difference Leased assets Deductible temporary difference Obligations under finance lease Accrued finance charges

Tax base

1,200,000

-

(1,120,000) (153,451)

Net taxable temporary difference

1,200,000

(1,120,000) (153,451) (73,451)

Deferred tax @ 30% (Asset) (b)

Difference

22,035

Liabilities against assets subject to finance lease (W3) 2015 Rs.

Present value of minimum lease payments Less: Current maturity shown under current liabilities

1,120,000 (326,549) 793,451

Minimum lease payments (W3) Not later than 1 year Later than 1 year and not later than 5 years (480,000 × 2) Less: future finance charges on finance lease

480,000 960,000 1,440,000 (320,000) 1,120,000

Present value of finance lease liabilities (W3) Not later than 1 year Later than 1 year and not later than 5 years (371,289 + 422,162)

326,549 793,451 1,120,000

The minimum lease payment has been discounted at an interest rate of 13.701% to arrive at their present value. Rentals are paid in annual instalments. W3: Repayment Schedule Opening Principal Years Balance repayment Rs.m

2015 2016 2017 2018

1,600,000 1,120,000 793,451 422,162

Rs.m

480,000 326,549 371,289 422,162

Interest 13.701%

Annual payment

Closing Balance

Rs.m

Rs.m

Rs.m

153,451 108,711 57,838

480,000 480,000 480,000 480,000

1,120,000 793,451 422,162 -

320,000

© Emile Woolf International

229

The Institute of Chartered Accountants of Pakistan

Financial accounting and reporting II

12.9

BILAL ENGINEERING LIMITED (a)

Computation of current taxation Rs.m

50.000 10.000

Tax liability for the year (52.446 × 35%) Tax liability for prior periods (0.100 × 35%)

18.356 0.035 18.391

Deferred taxation Accounting depreciation Tax depreciation

0.096 1.000 (7.000) (0.300) (1.350) 52.446

10.000 (7.000)

Financial charges on finance lease liability(1.00 – 0.3) × 13.701% Annual instalment of lease payment allowed under tax Amortization charged in accounts Amortization cost claimed in tax Excess of taxable income over accounting profit due to time differences Deferred tax credit at 35% Total tax expenses (current and deferred) (b)

Rs.m

Profit before tax Add: Accounting depreciation Financial charges on lease liability (1.00 – 0.3) × 13.701% Amortization of research and development cost for the year Less : Tax depreciation Annual instalment of lease payment Amortization of research and development cost (15 × 0.9/10) Current year taxable income

3.000

0.096 (0.300)

(0.204)

1.000 (1.350)

(0.350) 2.446 (0.856) 17.535

Bilal Engineering Limited: Notes to the financial statements for the year ended December 31, 2015 1.1

Relationship between tax expense and accounting profit

2015

Accounting profit before tax Tax on accounting profit at 35% Tax on expenses disallowed (Permanent Difference) Effective tax rate/tax charge

50.000 17.500 0.035 17.535

Rs.m

(c)

Journal entries 1

2

© Emile Woolf International

Income tax expenses Provision for taxation (Tax provision for 2015) Deferred tax asset Tax expenses – deferred (Deferred tax credit for 2015)

230

Debit

Credit

Rs.m

Rs.m

18.391 18.391 0.856 0.856

The Institute of Chartered Accountants of Pakistan

Answers

12.10 GALAXY INTERNATIONAL 28 : TAXATION

2015

2014

Rs.m

Rs.m

0.84 6.95 7.79

(0.96) (0.96)

28.1 : Relationship between tax expense and accounting profit Profit/(Loss) before taxation 23.50

(1.75)

Current - for the year (W – 1) Deferred (W – 2)

Tax at the applicable rate of 35% Tax effect of exempt income (1.25 x 35%)

W1 : Computation of Current Tax (Loss) / profit before tax as per books Add: Allowable income / Disallowed expenses Accounting depreciation Provision for gratuity Accrued expenses Less: Disallowed income / Allowable expenses Tax depreciation Interest income from SIBs (Exempt) Accrued expenses Taxable income / (loss) Tax liability (@ 35% Tax loss to be brought forward (29.05 x 35%) Tax payable W -2: Computation of Deferred Tax Timing differences (cumulative) on account of: Depreciation (2015: 30-51, 2014: 15-45) Accrued expenses Provision for gratuity Tax losses

Deferred tax @ 35% Add: Opening deferred tax (dr.) Charge/(Reversal) for the year

© Emile Woolf International

231

8.23 (0.44) 7.79

(0.61) (0.35) (0.96)

23.50

(1.75)

15.00 2.20 -

15.00 1.70 2.00

(6.00) (1.25) (2.00) 31.45 11.01 (10.17) 0.84

(45.00) (1.00)

21.00 (3.90) 17.10

30.00 (2.00) (1.70 ) (29.05) (2.75)

5.99 0.96 6.95

(0.96) (0.96)

(29.05) -

The Institute of Chartered Accountants of Pakistan

Financial accounting and reporting II

12.11 APRICOT LIMITED Taxation

2015

2014

Rs.m

Rs.m

Current (W1) Deferred (W2)

20.48 (1.58) 18.90

10.76 (21.35) (10.59)

Relationship between tax expense and accounting profit Profit before taxation Tax at the applicable rate of 35% Less: Tax effect of exempt income

2015 60.00 21.00 (2.10) 18.90

W1:

W2:

Computation of Current Tax Profit before tax as per books Add: Allowable income / Disallowed expenses Accounting depreciation Tax profit on sale of fixed assets Bad debt expense

60.00

45.00

10.00 1.00 5.00

9.00 7.00

Less: Disallowed income / Allowable expenses Tax depreciation Accounting profit on sale of fixed assets Capital gain Bad debts written off

(8.00) (0.50) (6.00) (3.00)

(7.00) (4.00)

Taxable income

58.50

50.00

Tax liability (@ 35%)

20.48

17.50

2015

2014

Rs.m

Rs.m

Computation of Deferred Tax Fixed assets (2014: 95-90, 2015: 82.5-80) (W2.1) Provision for bad debts (2014: 12×35%, 2015: 14×35%)

0.87

1.75

Closing balance of deferred tax Less: Opening balance

(4.90) (4.03) (2.45)

(4.20) (2.45) (18.90)

Charge for the year

(1.58)

(21.35)

[W2.2]

W2.1

W2.2

Movement of Fixed Assets Opening balance Disposal during the year Depreciation for the year - 2015

Accounting 95.00 (2.50) (10.00)

Tax 90.00 (2.00) (8.00)

Closing balance

82.50

80.00

Movement of provision for bad debts Opening balance Provision for the year Write off during the year

2015 12.00 5.00 (3.00)

2014 9.00 7.00 (4.00)

Closing balance

14.00

12.00

© Emile Woolf International

232

The Institute of Chartered Accountants of Pakistan

Answers

CHAPTER 13 – RATIO ANALYSIS 13.1

WASIM Ratios Year 7 Gross profit % = Gross prof it x 100 Sales Net profit % = Net prof it x 100 Sales  Return on capital employed = Prof it bef ore interest and tax



405

x 100 = 19%

2,160 9

x 100 = 0.4%



Share capital and reserv es+ Long - term debt capital

15

 x 100 = 6%

Sales

2,160

Current ratio = Current assets

422

Current liabilities Quick ratio = Current assets excluding inv entory

 

13.2

56

x 100 = 29%

x 100

 = 8.8 times

246



x 100 = 2.9%

Asset turnover =





53

190

Share capital and reserv es+ Long - term debt capital



x 100 = 20%

1,806

246



362 1,806

2,160



Year 6

422 - 106

Current liabilities Average time to collect = Trade receiv ables x 365 Sales Average time to pay = Trade pay ables x 365 Cost of purchases Inventory turnover = Inv entory x 365 Cost of sales

= 1.7 times

  1.2 times

254



316 x 365 2,160



198 x 365 1,755



106 x 365



1,755



Amir

= 9.5 times

190

254



1,806



265

= 1.8 times

147 265 - 61

 1.4 times

147

 53 day s

198 x 365



1,806

= 41 day s

  22 day s



 40 day s

142 x 365

= 36 day s

1, 444 61 x 365

= 15 day s

1, 444

AMIR AND MO





Mo

Gross profit % =

Gross prof it

90,000

x 100

x 100 = 60%

150,000

Sales

490,000

x 100 = 70%

700,000

Net profit % = Net prof it x 100 Sales



© Emile Woolf International



44,895

 x 100 = 30%

150,000



270,830

x 100 = 39%

700,000



233

The Institute of Chartered Accountants of Pakistan

Financial accounting and reporting II

Return on capital employed = Prof it bef ore interest and tax Share capital and reserv es+ Long - term debt capital Amir 61,500 + 500 x 100 = 28.5% 207, 395 +10,000



Mo

371,000 +12,000

x 100 = 47%

565,580 + 250,000



Asset turnover =



Sales

x 100

Share capital and reserv es+ Long - term debt capital

Amir

150,000

= 0.7 times

207, 395 +10,000

 Mo

700,000

= 0.85 times

565,580 + 250,000



Amir

Mo

 ratio = Current Current assets

50,000

Current liabilities

22,605

153,250

= 2.2 times

= 1.3 times

117,670

Quick ratio =



Current assets excluding inv entory 

50,000 - 12,000 22,605

Current liabilities

Average time to collect =



Trade receiv ables

x 365



= 1.7 times

37,500



150,000

153,250 - 26,250

= 1.1 times

117,670

x 365 = 91 day s



105,000

x 365 = 55 day s

700,000

Sales

Average time to pay =



Trade pay ables

 22,605

x 365

Cost of purchases



Inventory turnover = Inv entory

© Emile Woolf International

x 365 = 137 day s

60,000



117,670

x 365 = 204 day s

210,000

 12,000

x 365

x 365 = 73 day s

60,000

Cost of sales







26,250

x 365 = 46 day s

210,000



234

The Institute of Chartered Accountants of Pakistan

Answers

CHAPTER 14 – ETHICAL ISSUES IN FINANCIAL REPORTING 14.1

ETHICAL ISSUES The range of comments made by Arif raises questions over his ethical behaviour and professional standards. A chartered accountant should be unbiased when involved in preparing and reviewing financial information. A chartered accountant should prepare financial statements fairly, honestly, and in accordance with relevant professional standards and must not be influenced by considerations of the impact of reported results. Arif’s failings Arif appears to be influenced by the need to achieve a specified level of profit. This is not appropriate and calls his integrity into question. In addition Arif’s professional competence seems to be suspect. His comment on not being up to date on all of the little technicalities in IFRS s suggests that he has not maintained a level of professional competence appropriate to his professional role. ICAP members have a responsibility to engage in continuing professional development in order to ensure that their technical knowledge and professional skills are kept up to date. Arif should seek continuing professional development activities and improve his knowledge on ethical standards. Furthermore, it might be expected that as Waheed’s superior he should set an example to Waheed and guide him in his responsibilities. Clearly this is not happening. As a member of ICAP Arif should be aware of the ICAP code of ethics. Arif should know of the danger of self-interest threats and intimidation threats to himself and to others. His attempt to influence the outcome of a fellow professional by applying such a threat to that individual is very unprofessional. Waheed’s ethical issues Waheed faces a self-interest threat, in that there is the possibility of a bonus provided the earnings per share figure remains the same as last year. Arif has also suggested that she can influence the Board’s decision over employing him as a replacement finance director – another self-interest threat to Waheed. Both of these threats must be ignored. Arif’s comments imply that his application of professional responsibility is lacking. This may extend into the way in which the current financial statements have been prepared. Waheed must be very careful (as always) to carry out the review with all due care. Waheed should first discuss his recommendations with Arif and remind his of his professional responsibilities to ensure that the accounting standards are correctly followed. If the financial statements are found to contain errors or incorrect accounting treatment then they must be amended. If Arif refuses to amend the draft financial statements if necessary Waheed should discuss the matter with other board members (including non- executives and the audit committee, if possible). Further action might include consulting with ICAP.

© Emile Woolf International

235

The Institute of Chartered Accountants of Pakistan

Financial accounting and reporting II

14.2

SINDH INDUSTRIES LTD (a)

Financial reporting issues Revenue IAS 18 Revenue sets out the rules to be followed in recognising revenue. The fact that the customer cannot cancel the contract is not relevant to the recognition of revenue. Revenue from providing a service is recognised according to the stage of completion subject to satisfying criteria set out in IAS 18. In the absence of other information the revenue in this contract should be recognised over the life of the contract as time progresses. As the contract was only signed just before the year end, none of the revenue can be recognised in 2015. The credit for the amount received should be recognised as a liability. This represents the obligation that the company has to provide the service over the next two years. The fact that the customer cannot cancel the contract is not relevant to the recognition of revenue. If Sindh Industries failed to provide the service they would be sued for restitution. Therefore the revenue can only be recognised as the service is provided. New factory Borrowing costs directly attributable to construction of an asset which necessarily takes a substantial period to get ready for its intended use should be capitalised as part of the cost of that asset under IAS 23 Borrowing Costs. IAS 23 states that the capitalisation of borrowing costs should commence when three conditions are all met for the first time: borrowing costs are being incurred, expenditure is being incurred and activities to prepare the asset are being undertaken. Although borrowing costs were incurred throughout the year and expenditure was incurred from 1 February 2015 (the date the land was purchased), construction only started on 1 June 2015. Therefore this is the date on which capitalisation commences. Capitalisation ceases when substantially all of the activities required to make the asset ready for use/sale have been completed, that is on 30 September 2015. (The actual date on which the factory was brought into use is irrelevant.) Therefore the period of capitalisation should be four months. Where construction is financed from general borrowings, the calculation of the amount to be capitalised should be based on the weighted average cost of borrowings. This is: (Rs.1,000,000 × 9.75%) + (Rs.1,750,000 × 10%) + (Rs.2,500,000 × 8%)/ (Rs.1,000,000 + Rs.1,750,000 + Rs.2,500,000) = 9% Therefore the amount capitalised should be 9% × Rs.4.5 million (land Rs.1.8 million plus construction costs Rs.2.7 million) × 4/12 = Rs.135,000. The total cost of the factory should be measured at Rs.4,635,000 (Rs.1.8 million plus Rs.2.7 million, plus Rs.135,000). The amount that has been recognised in the statement of financial position should be reduced by Rs.315,000 (Rs.450,000 – Rs.135,000). Finance costs recognised in profit or loss should be increased by Rs.315,000. Land should not be depreciated because it has an indefinite life. Under IAS 16 Property, Plant and Equipment depreciation charges should start when the asset becomes available for use, from 1 October 2015 in this case.

© Emile Woolf International

236

The Institute of Chartered Accountants of Pakistan

Answers

Depreciation of Rs.35,000 ((Rs.2.7 million, plus (Rs.135,000 × 2.7/4.5) ÷ 20) × 3/12) should be recognised in profit or loss for the year ended 31 December 2015 and the carrying amount of the asset reduced by the same amount to Rs.4.6 million. Useful life of the blast furnace Depreciation of the blast furnace has been based on an estimated useful life of 20 years. This is at variance with a report by a qualified expert. The asset valuation specialist treats the furnace as being made up of two components, the main structure and the lining, which must be replaced at regular five yearly intervals over the life of the asset. This is the approach required by IAS 16. The uncertainties inherent in business mean that many items in financial statements cannot be measured with certainty, but estimates should always be made using the most up to date and reliable information. Where estimates have been prepared by professionals with relevant qualifications, then it is nearly always most appropriate to use those estimates. Therefore in accordance with the valuer’s report the main structure of the furnace should be depreciated over 15 years from 1 January 2015 and the lining should be depreciated over five years from that date. The reassessment of the estimated lives of assets is a change in accounting estimate, rather than a change in accounting policy (IAS 8 Accounting Policies, Changes in Accounting Estimates and Errors). Changes in accounting estimate should be dealt with on a prospective basis. This is achieved by including the effect of the change in profit or loss in current and future periods. The additional depreciation should be calculated as: Rs.000 Revised depreciation:

main structure ((Rs.3.5m – Rs.1.4m)/15 years)

140

lining (Rs.1.4m/5 years)

280 420

Current depreciation (Rs.3.5m/20 years)

(175)

Additional depreciation

245

IAS 8 requires the disclosure of the nature and amount of the effect of the change in the estimate of useful lives on the profit for the year. (b)

Revised financial statements Statement of profit or loss extract for the year ended 31 December 2015 Draft

Revenue

Rs.000 Profit before tax

© Emile Woolf International

Rs.000

2,500

(1,000)

237

Borrowing

Blast

costs

furnace

Rs.000 (315)+ (35)

Revised

Rs.000

Rs.000

(245)

905

The Institute of Chartered Accountants of Pakistan

Financial accounting and reporting II

Statement of financial position at 31 December 2015 Draft Revenue Rs.000 Rs.000

(c)

Non-current assets Property, plant and equipment Current assets Total assets

12,000 3,500 15,500

Share capital Retained earnings Equity Non-current liabilities Current liabilities Total equity and liabilities

2,000 6,000 8,000 5,000 2,500 15,500

(1,000) 500 500

Borrowing costs Rs.000

Blast furnace Rs.000

(315) + (35)

(245)

(315) + (35)

(245)

Revised Rs.000

11,405 3,500 14,905 2,000 4,405 6,405 5,500 3,000 15,905

Ethical issues It is noticeable that all the adjustments required reduce profit. This and the background to the previous finance director’s resignation suggest serious problems. It is not clear who actually prepared the draft financial statements. If they were prepared by more junior staff in the absence of a finance director, some of the adjustments (for example, the calculation of borrowing costs to be capitalised) could be the result of genuine errors or lack of accounting knowledge. However, it seems reasonably clear that the managing director has attempted to influence the treatment of the revenue and the estimated useful life of at least one significant non-current asset. (Note: the directors have reviewed the useful lives of several items of plant and machinery and it is possible that other assets besides the furnace are being depreciated over unrealistically long periods.) It seems almost certain that the previous finance director resigned as a result of pressure from the managing director (and possibly from other members of the Board) to present the financial statements in a favourable light. The directors intend to seek a stock market listing in the near future. Therefore they have clear motives for manipulating the profit figure and also (perhaps) for making controversial decisions before the financial statements come under much greater scrutiny as a result of the listing. The job title of financial controller is also significant. It suggests that the role has been downgraded and that the person holding it has less authority than the rest of the Board. Possible courses of action:  Discuss with the managing director the financial reporting standards that apply to the transactions and explain the implications of non-compliance. If the managing director is himself a member of a professional body then it might be worth pointing out to him that he himself is bound by an ethical code.  Advise him that as a Chartered Accountant you are bound by the ICAP code of ethics, and that you would not be prepared to compromise your views of the figures he has prepared for career advancement.  Consider speaking to the other directors (or audit committee if there is one) and seeking their support.  If all of these actions produce a negative response then it would be appropriate to consult the ICAP ethical handbook and/or the Institute.  If all else fails then consider seeking alternative employment.

© Emile Woolf International

238

The Institute of Chartered Accountants of Pakistan

Head Office-Karachi:

Chartered Accountants Avenue, Clifton, Karachi-75600 Phone: (92-21) 99251636-39, UAN: 111-000-422, Fax: (92-21) 99251626, e-mail: [email protected]

Regional Office-Lahore: 155-156, West Wood Colony, Thokar Niaz Baig, Raiwind Road, Lahore Phone: (92-42) 37515910-12, UAN: 111-000-422, e-mail: [email protected] Islamabad Office:

Sector G-10/4, Mauve Area, Islamabad UAN: 111-000-422, Fax: (92-51) 9106095, e-mail: [email protected]

Faisalabad Office:

36-Z, Commerical Center, Near Mujahid, Hospital Madina Town, Faisalabad Phone: (92-41) 8531028, Fax: (92-41) 8503227, e-mail: [email protected]

Multan Office:

3rd Floor, Parklane Tower, Officers’ Colony, Near Eid Gaah Chowk, Khanewal Road, Multan. Phone: (92-61) 6510511-6510611, Fax: (92-61) 6510411, e-mail: [email protected]

Peshawar Office:

House No. 30, Old Jamrud Road, University Town, Peshawar Phone: (92-91) 5851648, Fax: (92-91) 5851649, e-mail: [email protected]

Gujranwala Office:

2nd Floor, Gujranwala Business Center, Opp. Chamber of Commerce, Main G.T. Road, Gujranwala. Phone: (92-55) 3252710, e-mail: [email protected]

Sukkur Office:

Admin Block Sukkur IBA, Airport Road, Sukkur Phone: (92-71) 5806109, e-mail: [email protected]

Quetta Office:

Civic Business Center, Hali Road, Quetta Cantt Phone: (92-81) 2865533, e-mail: [email protected]

Mirpur AJK Office:

Basic Health Unit (BHU) Building Sector D, New City Mirpur, Azad Jammu and Kashmir e-mail: [email protected]

2015

FINANCIAL ACCOUNTING AND REPORTING II QUESTION BANK

View more...

Comments

Copyright ©2017 KUPDF Inc.
SUPPORT KUPDF